Você está na página 1de 195

The Joint Program of Family and Community Medicine in Jeddah

The Final Examination of the


Saudi Board for Family Medicine
A. Assaggaf
The final examination of the Saudi Board for Family Medicine consists of two parts:
I.

The Written examination

II.

The Clinical and Oral Examination

I.

The Written examination, consists of the following three components:


1. Multiple Choice Questions.
2. Modified Essay Questions.
3. Critical Reading Questions.

II.

The Clinical and Oral Examination, which also consist of three components:
1.
2.
3.

I.

Simulated Clinics.
Data and slides Interpretation.
Oral Examination.

The Written Examination


1.

Multiple Choice Questions Paper

Time allowed is three hours

This paper consists of around 100 questions in the best answer format, 75 questions
are in the form of patient management questions (PMQ) and 25 questions are
traditional factual knowledge questions.

The distribution of the question will be as the following:


Areas Number of Question
Pediatrics
Therapeutics
Psychiatry
Internal Medicine
Obstetrics/Gynecology
Surgery/Orthopedic
Dermatology
Accident and Emergency
Community Medicine
ENT
Ophthalmology

2.

19
8
12
13
11
9
7
8
5
4
4

Modified Essay Questions (MEQ)

Usually three MEQs will appear in the exam

Time allowed: 11/2 hours

The Joint Program of Family and Community Medicine in Jeddah

3.

Critical Reading Questions (CRQ)

Time allowed: 2 hours

The guidelines for appraisal of scientific papers recommended by the EvidenceBased Medicine working group published in JAMA are adopted

Time allowed: 11/2 hours


Marking of the written part.
1.
2.
3.

II.

MCQ..40 marks
MEQ 35 marks
CRQ...25 marks

Clinical And Oral Examination

Only those who pass the written examination will be allowed to set for the clinical
examination.
1.

Simulated Clinics:

The main objective is assessment of adequate consultation skills, which include the
following:

History-taking skills.
Communication skills
Health education and health promotion
Patient management skills
Prescribing.
Effective use of resources: the primary health care team, referral system and use of
investigations
Evidence of being a competent and safe doctor.
Categories of Cases:

The cases or the their themes may include all or many of the following:
Acute cases.
Chronic disease management.
Difficult patient
Difficult situation, e.g.: breaking bad news or patient with multiple problem
Clinical examination.
Telephone consultation / Referral.
Patient education.
2.

Data and Slides Interpretation

Format of date interpretation exam:


A group of candidates may sit in a hall or a big room and would be given a booklet
containing the data and shown slides using slide projector and are supposed to write their
answers in the booklets provided to them.
A)

The data may include some or all of the following investigations:

EBC
Biochemistry
Serology

The Joint Program of Family and Community Medicine in Jeddah

Urine/Stool
Hormonal assay
Growth Chart
Pulmonary function test
Audiogram
X-ray
ECG
Etc.

B)

The slides may cover some or all of the following board areas:

General Medicine
Dermatology
Ophthalmology
Other relevant topics
3.

The Oral Examination:

Each candidate will be examined by two panels, spending around 25-30 minute at
each one
Each panel consist of two examiners
One of the panels will have a specialist in community medicine as examiner.
The areas covered in one panel will not be repeated in the other one.

Marking of the Clinical and Oral Examinations:


The marking of the clinical and oral examinations is distributed as the following:
The mark distributed as follows:
1.
2.
3.

Simulated clinic
40%
Data and slide interpretation 30%
Oral examination
30%

According to the Saudi Council for Health Specialties Examination Regulation, the grading
of the candidates at the clinical and oral examination will be in the following categories:
1.
2.
3.
4.

Clear pass
Pass
Borderline Pass
Fail

(For more details see General Examination Rules and Regulations Revised, November
1999; Page 9-11 Saudi Council for Health Specialties)
(For more details about the contents, format and instructions see The Final Examination
of the Saudi Board for Family Medicine / Contents and Instruction For Candidate
September 2000, by Dr. Eiad Al Faris and Dr. Hamza Abdul Ghani)

The Joint Program of Family and Community Medicine in Jeddah

Arab Board Final Examination


A. Assaggaf
Arab Board Final Written Examination
The final written examination consist of a total 210 MCQs
Divided to two parts:
Part I:
Content:
150 Traditional MCQ (Factual Knowledge questions)
Time Allowed: 3 hours.
Part II:
Content:
60 Question in the form of patient management Question (P.M.Q)
Time allowed: 2 hours.

Passing Score & marks distribution:


Each one of the 210 multiple choice questions (Part I + Part II) is given an equal score &
the exam mark will be out of 210 which then will be transferred into percentage (%).
To pass the written examination, it is essential to score 60% as a minimum.
Only those who pass the written examination will be allowed to set for the Clinical
examination.

Arab Board Final Clinical Examination (OSCE)


The final clinical examination consist of two parts:
I.

Objective Structured Clinical Examination (OSCE)

II.

Oral Exam

I.

Objective Structured Clinical Examination (OSCE)

What is OSCE?
The objective structured clinical examination (OSCE) is an approach to the assessment of
clinical competence in which the components of competence are assessed in a planned or
structured way with attention being paid to the objectivity of the examination.
The student is assessed at a series of stations with one or two aspects of competence being
tested at each station. The examination can be described as a focused examination with
each station focusing on one or two aspects of competence. In a typical examination there
may be 20 such stations and students rotate round the stations at a predetermined time
interval. A 20-stations examination with 5 minutes at each station will occupy 100
minutes.

The Joint Program of Family and Community Medicine in Jeddah

One circuit of 20 stations will allow 20 students to be examined simultaneously. If the


number of students is greater than 20, this can be accommodated by running parallel
circuits of stations or by repeating the single circuit with another group of students.
Although the concept of an examination with stations round which students rotate
represents an important aspect of the OSCE, the examination is more than just a multistation examination.

Objective

OSCE
Subjective bias is removed as possible

Structured

The content of the examination are planned carefully in


advance

Clinical

It is a performance assessment, It is concerned with what


candidate can do rather with what they know

Examination

Objective:
Traditional clinical examinations have been criticized on the grounds that they lack
objectivity. In the OSCE, subjective bias is removed as far as possible.
In any clinical examination there are three variables. The patient, the examiners and the
candidate. In the OSCE attempts are made to minimize any examiner subjective bias and to
minimize any bias and to minimize any bias introduced by candidates seeing different
patients. The following contribute to the objectivity of the examination.

Candidates see a number of examiners in the course of the examination, usually


eight or more.
What is to be assessed at each station is agreed in advance and a marking schedule
is produced which lists what is expected of the candidate at each station.
Examiners use a checklist, which reflects what is to be tasted at the station. The
examiners agree this in advance.
The aim in the examination is to produce a profile for each candidate rather than a
single composite mark. A candidate, for example, may be competent in physical
examination techniques, but have an unsatisfactory attitude and may be taking in
interpersonal skills.
The standard on criteria for pass, distinction (if appropriate), fail, and dangerous
fail can be agreed.
Examiners can be trained for the task expected of them and their performance can
be assessed in advance on practice videotapes.
The examination tests a wide range of skills thus greatly reducing the sampling
error. This very significantly improves the reliability of the examination.
Students all face the same tasks.
Simulated patients help to ensure that all students are presented with a similar
challenge.

Structured
The examination is structured in such a way that the content of the examination and the
competences to be tested are planned carefully in advance. Thus the examination can
sample different subject areas, e.g. cardiovascular system, dermatology, accident and

The Joint Program of Family and Community Medicine in Jeddah

emergency medicine, geriatrics, etc. and different skills, e.g. history-taking, physical
examination, problem-solving, patient education including attitudes.
In this way the examination is designed to reflect adequately the objectives of the
course and to make the maximum use of the time available for the exam. It is structured so
that competencies in history-taking, physical examination, patient education, problemsolving, etc. are tested in a range of areas and not in one or two areas of medicine, e.g. a
patient with a myocardial infarction or a patient with chronic bronchitis
Clinical
The OSCE is a clinical or practical examination. It is a performance assessment and is
concerned with what students can do rather than with what they know. Here are some
examples of competencies assessed at stations in an OSCE.

History taking from a patient who presents with a problem, e.g. abdominal pain.
History taking to elucidate a diagnosis, e.g. hypothyroidism.
Educating a patient about management, e.g. use of inhaler for asthma.
General advice to a patient, e.g. on discharge from hospital with a myocardial
infarction.
Explanation to patient about tests and procedures, e.g. endoscopies.
Communication with other members of health care teams, e.g. brief to nurse with
regard to a terminally ill patient.
Communication with relatives, e.g. informing a wife that her husband has bronchial
carcinomas.
Physical examination of system or part of body, e.g. examination of hands.
Physical examination to follow up a problem, e.g. CCF.
Physical examination to help confirm or refute a diagnosis, e.g. thyrotoxicosis.
A diagnostic procedure, e.g. ophthalmoscopy.
Written communication, e.g. writing referral letter or discharge letter.
Interpretation of findings, e.g. charts, laboratory reports or findings documented in
patients records.
Management, e.g. writing a prescription.
Critical appraisal, e.g. review of
published article or pharmaceutical
advertisement.
Problem solving, e.g. approach adopted in a case where a patient complains that her
weight as recorded in out-patients was not her correct weight.

In the examination it is what the examinee does, when confronted with a patient or a
situation, that is assessed not what he knows and the answers he writes to a theoretical
question on the subject. A range of techniques can be employed in the OSCE to emphasize
the practical nature of the examination. These include simulated patients, videotape and
simulators: of these, simulated patients have the greatest to contribute to the OSCE. In
traditional clinical examinations all too often history-taking ability is assessed by the
examiner scoring the candidates written or verbal report of the history and no attempt is
made to watch the candidate taking the history. In the OSCE the process as well as the
product, is measured in the examination. The technique he uses taking the history and the
questions he asks are assessed as well as his findings and his conclusions based on the
findings.

The Joint Program of Family and Community Medicine in Jeddah

What About Arab Board OSCE Exam:

You will rotate round 17 stations (Number 1-17) spending 7 minutes at


Each station. A bell will ring at the beginning of the examination and at the end of each 7minute period.
1)

At each station you will be asked to perform certain tasks e.g.


a)
b)
c)
d)

Take history
Examine a patient
Interpret x-rays or other clinical materials
Describe management etc.

2)

Two (2) of the 17 stations are rest station where you will do no task.

3)

All stations have an equal value from the total mark allocated for the OSCE.

II

The Oral Examination:


a) Two panels will examine each candidate, spending around 25-30 minutes at
each one.
b) Each panel consists of two examiners.

Marking of the Clinical Examination:


The marks of clinical examination are distributed as the following:
1)
2)

OSCE
Oral Exam.

60%
40%

To pass the Clinical examination, it is essential to score a minimum of 60% of the total
mark for the clinical examination. (By adding the marks obtained in the OSCE & the Oral).

The Joint Program of Family and Community Medicine - Jeddah

1.

Approach to Common Chronic Problems in PHC


Approach to Patient With Asthma
F. Rayes, M. Alatta & A. Al Harthy

The following items may need to be considered in each follow up visit:

Doctor-patient relationship
Encouragement of patient contribution
Patients cues
Well-being and psychosocial context of the problem
Physical complaints:
E.g. Any recent exacerbations
Current level of treatment
Problems with medication
Smoking habits
Restrictions on lifestyle:
o E.g. Exercise tolerance
o Time off work
Examination:

Chest examination

Current peak flow (PF):

The percentage of best and predicted PF


Before and after bronchodilators
Consider spirometry

Inhaler technique

Review of home recordings of PF


Management:

Discuss any concerns

Provide printed information if required

Consider others tests, e.g. allergy

Agree management program:


Self-management protocol
Action in emergency

Arrange for follow-up


The frequency of follow-up depends on the severity of the asthma, medication use,
number of exacerbations, etc. Well-stabilized patients with asthma probably do not
need review more than yearly.

Common problems in follow up of asthmatic patients:


o
o
o
o

Denial and refusing to accept the diagnosis


Asthmatic child of a smoking parent
False believes about asthma medication (it is dangerous or it is addictive)
Wrong technique in using PFM or inhalers

The Joint Program of Family and Community Medicine - Jeddah

Asthma General Advice:

Establish rapport
Encourage patients contribution:
Active listening and use of open-ended questions
Explore the social & psychological context of the problem
(Is he a student? Is he a smoker? Any association with sport?)
Exploration of patients Ideas, Concerns & Expectations
Examples (Afraid to be addicted to the inhalers
Afraid from corticosteroids
Frequent absence from collage
Drop out from sport team
Expect to change to tablets)
Health education:
o Explanation of asthma
o Explanation of asthma drugs: bronchodilator , anti-inflammatory
o Stress on the importance of correct technique
o Discuss The possible triggering factors & how to avoid them:
E.g. house dust mites, animal dander and house pets, cockroaches, respiratory
infections, environmental irretant, tobacco smoke, cold air, exercise, air pollution,
chemical gases or fumes
Drugs (aspirin, NSAID, beta-blockers, food preservative (sulfates)
o Action plan: recognize deterioration, what action to take,
o Importance of PEFR
Respond to patient cues (his understanding ability, his anxiety)
Provide patient with health education material
Arrange for follow up

Inhalation Technique Instructions:


If the patient is using dilator or anti-inflammatory

Explain why it is used and what is there effect

Explain about machine and its parts


o Remove cap (hold in correct position not upside down) and shake.
o Breath out gently through your mouth.
o Put the mouthpiece into your mouth (or 4 cm) always and in front of your mouth.
o Start to breathe in slowly, then in the middle of the breath press the canister down
go on breathing (head up right and back) and keep pressing.
o When you can not breath in any more take the inhaler out and hold breath 5-1 0
seconds.
o In case of more than one puff repeat steps 4-8 after one min.
o Wipe mouthpiece of inhaler and cover it.
o Check the expiratory date of the medicine

Wash it twice a week.

Demonstrate to patient

Let him do it and observe him, in order to correct him.

Tell him about spacer and its benefits.

If patient using 2 types. Use dilator first.

If using steroids wash mouth after.

Take feedback and encourage patient to ask questions

Arrange for follow up

The Joint Program of Family and Community Medicine - Jeddah

10

How to Use the Peak Flow Meter?


As a doctor you should have your peak flow meter at your clinic to measure your patient
peak flow rate.
The educated patient should have at home to check his own peak flow rate specially if sign
is worsening to start the action plan (see later).
This is a checklist of proper use of peak flow meter:
o Explain the machine and why to use it?
o Put mark to zero
o Stand up
o Finger away of horizontal not too tight
o Breath in as much as you can
o Lip sealing tongue should not be inside
o Breath out as hard and as fast as you can
o Write down the level (aside) put marker back to zero
o Repeat three times and record the maximum on chart
o Demonstrate (change mouth pieces)
o Let him do it and observe to correct
o Monitor bid daily, it is not a flat line
o Any drop below the zone step up medications

PEFR & Severity of Acute Attack:

80-100% of the patient best


50-80% of the patient best
50% of the patient best
< 33% of the patient best

=
=
=
=

Mild attack
Moderate attack
Sever attack
Life threatening attack

Patient Education:

What is asthma?
Asthma is a chronic breathing problem, its symptoms varies from cough, (which usually
more at night and increase after exercise) shortness of breath, wheezing.
These symptoms are caused by decrease air entry to the lung due to inflammation (
narrowing) of the air way.

If the patient take care of the disease and follow proper medical advice he can have
normal activity, but asthma can be a life threatening if the case neglected & did not receive
the proper management.

The illness may start in childhood which may improve when child is getting older
or may continue to adulthood. Also it can start at adulthood in previously healthy child.

What is an asthma attack?


Asthma frequently present in attacks which mean the person can have no symptoms, then
when he is exposed to a precipitating factor. The attack start (and this cause many doctors
miss the diagnosis if they see the patient in between attacks).

What is the precipitating factors of asthma attack?

The precipitating factor can be:


cold air, dust, smoking, exercise, perfumes,
emotional stress and others.

It can vary from case to case, so the patient try his best to avoid the precipitating
factor. But for exercise it is recommended to continue activity and exercise and to take one
inhalation of the Bronchodilator (Ventolin) 20 minutes before start the exercise to prevent
the attack

The Joint Program of Family and Community Medicine - Jeddah

11

Avoidance of Irritant & Allergens:

Smoking:
Both active and passive smoking must be avoided if family member
smoke and not ready yet to quit he should not smoke at home, in the car or other closed
places where patient may stay in.

Wood smoke, household sprays, bakhoor, cooking oil, detergent, some strong small
should be avoided.

House dust mite:


Which is increase in humid areas. These measure lower it
amount:
o
Washing all the bedding sheets, blankets and covers in hot water.
o
Vacuum the carpet regularly while the patient not in the room (if you
can takeout the carpeting out).
o
Avoid having animals in the house (cats or birds).
o
Frequent cleaning and apply insecticide when asthmatic not present.
o
If the patient has occupational exposure to allergies should be referred
to specialist.

Action Plan: (as mentioned in the National Protocol)


The action plan can be symptom based and/or peak flow based to suit the persons
understanding of his/her problem. All people with asthma need to:
o Be able to recognize deterioration
o Know what action to take
o Have their action plan reviewed and, if necessary, modified
o Following an acute attack
o All people with asthma should know how to obtain prompt medical assistance:
o Provide patients and families with a written ACTION PLAN CARD

The Traffic Light Zone System:


The following zone system that puts the control asthma into one of 3 different zones each
with specific instructions. Traffic light colors indicating the various zones have been
chosen since most patients are familiar with them.

Green zone: Indicates all is clear, PEF is at 80-100% with less then 15%
variability. There are minimal symptoms (ideally none) related to asthma. The patient is to
continue maintenance therapy as previously instructed by the physician. Inhaled 2
agonists may be used if needed prior to exercise or for occasional mild symptoms.

Yellow zone: Indicates caution, PEF is 60-80% predicted with 15-25% variability.
Asthma symptoms such as nocturnal cough, shortness of breath or wheezing may occur.
This would indicate:
Either an acute exacerbation in which case guidelines in following section should be
followed,
Or a gradual deterioration in the severity of asthma where intensification or
stepping up of maintenance therapy is required, this should be done in consultation
with the physician. A doctor ought to be consulted within 48 hours.

Red zone:
This signals a medical alert. PEF is less than 60% and asthma
symptoms are present at rest and interfere with activity. Inhaled 2 agonist should be taken
immediately, if PEF remains below 60% immediate medical attention at an acute care
facility is recommended, if PEF improves to above 60% instructions for the yellow zone
should be followed. Physician has to be consulted within 4 hours or less.
These are broad guidelines. Instructions given to each patient should be individualized
taking into account factors such as reliability, level of understanding, availability of
medical care and the doctor patient relationship. In some patients instructions on when to
start sustained release theophylline or prednisolone could also be given.

The Joint Program of Family and Community Medicine - Jeddah

Approach to Patient With Diabetes Mallets


M. Alatta & F. Rayes
The following items may need to be considered in each visit:

Doctor-patient relationship
Encouragement of patient contribution
Patients cues
Well-being and psychosocial component of patient problem
Physical complaints
Dietary problems
Medication problems
Examination & investigation:

Fasting, blood glucose

Urinary protein and glucose

Body mass index

Blood pressure

Visual acuity
Management & education:

Discussion of immediate concerns

Discussion of current management

Follow-up arrangement made

The following items may need to be considered in annual visit:

Smoking habit
Pruritus
Pain and/or paraesthesia in legs
Sexual problems
Visual problems
Angina
Claudication
Examination:

Fundoscopy

Inspection of feet:
o Circulation
o Reflexes

Blood creatinine

Blood cholesterol
Education:
o Check self-care & lifestyle
o Self-monitoring
o Foot and eye care

o Smoking habit
o Diet
o Exercise Smoking habit

Common problems in follow up of diabetic patients:


o
o
o
o

Uncontrolled or poor control of DM


Misunderstanding of diabetic diet
False believes about hypoglycemic medication (it is dangerous or it is addective)
Refusing to accept insulin injection

12

The Joint Program of Family and Community Medicine - Jeddah

13

Counseling Newly Diagnosed Diabetic Patient:

Doctor-patient relationship
Encouragement of patient contribution
Patients cues
Exploration of patient ideas, worries and concerns
(What do you know about diabetes?)

Explanation about diabetes:


It is a common, chronic illness, characterized by primary hyperglycemia.

It is two types:
o The first type: Represent 10% of diabetic patients, they are called IDDM (they
always take insulin).
o The second type: Consisted 90% of cases and they are called NIDDM their body
secretes either too little insulin which is not enough or large amount but not
effective.
When the body secretes no insulin sugar level will increase.
So diet and exercise are important to control diabetes.
By some modification in patients life style he can live a normal life.

Diet:
o Eat three meals a day at regular times.
o Eat nothing between meals except a snack (specify hours).
o Eat no sweet for now (sugar, dates, honey)

Exercise:
Start by increasing your daily physical activity, like walking.
Other exercises may by added when you are feeling better.

Glucose Monitoring
o This will help you feel that you have more control over your health.
o Specify time: e.g. before breakfast & supper.

Blood Glucose Level:


o Home blood glucose measurement technique if the patient has glucometer
o It is the best way of monitoring
o Show the patient how to use this monitoring device.

Urine Glucose Level:


o Explain to patient how to use the dipstick.
o It is less effective way of monitoring.

Medication:
Insulin:
o There is no insulin in tablets form.
o They must be taken as subcutaneous injections.
o They are usually taken twice per day: 30 min. before breakfast & 30 min. before
supper.
o Schedule may be changed till we find the best for you.
o The nurse will show you the technique for injecting your self & how to store and
take care of your insulin.
Tablets:
o They are not insulin
o Given to patient whose bodies make insulin.
o They help your insulin to work better.

Low Blood Sugar:


o Your medication may cause sometimes your blood sugar to drop too low.
o When this happen you may feel the following:
o Headache, weakness, sweating, feeling of anxiety...

The Joint Program of Family and Community Medicine - Jeddah

What to do if you develop symptoms of hypoglycemia:


o Quickly eat or drink something sweet.
o (Sugar in water, Orange juice, honey)
o Call your doctor right away.

Medic alert card:


You can buy it from drug stores. Wear it all time.

Care of your feet:


o It is very important for you to always keep your feet dry and clean.
o Wear socks and good shoes.
o Health educator will give you a pamphlet about foot care.

Driving:
It is advisable to stop driving until your sugar is well controlled if you are taking
medication, because risk or low sugar.

Traveling:
Talk with your doctor before you travel to help you to fit your diet and medication into
your travel schedule.

Give patient printed educational material


(Leaflets or booklets).

Arrange appointment with dietitian within two weeks.

Follow up in 4-8 weeks


You can ask the patient to come with care taker e.g. his wife, his mother.

Take continues feedback and answer any questions.

Discuss complications and how to prevent them.

14

The Joint Program of Family and Community Medicine - Jeddah

Approach to Patient With Hypertension


H. Al Hajjar & F. Rayes
The following items may need to be considered in history taking in the first visit:

o
o
o
o
o

o
o
o
o
o

o
o

o
o
o
o

o
o
o
o
o
o

o
o
o
o
o

o
o
o
o
o
o
o

Doctor-patient relationship
Encouragement of patient contribution
Patients cues
Well-being and psychosocial component of patient problem
Date of onset of hypertension, duration
Levels at time of onset
Specific question to rule-out secondary hypertension:
Hairsutism
Easy bruising
Palpitation, sweating
Muscle cramps
Leg claudication
Symptoms suggestive end organ damage:
Chest pain
Breathlessness
Orthopnia
claudication
Transient visual loss
Past medical history :
Renal disease
Obstetric history (pre eclampsia)
Cardiovascular risk factors:
Smoking
Obesity
Hyperlipidemia
Diabetes mellitus
Family history (Assessment of degree of risk) :
Hypertension
Ischemic heart disease
Stroke
Hyperlipidemia
Premature cardiovascular death
Renal disease as autosomal dominant polycystic kidney
Drug history :
For hypertension : efficacy and side effects
Other drugs : OCP, Steroids, Thyroid hormone, NSAID
Over the counter medications
Alternative medicine products
Substance abuse
Social history :
Smoking and alcohol
Diet : salt, fat, weight gain
Caffeine
Leisure activity & exercise
Work environment and stresses
Family and home situation
Education level

15

The Joint Program of Family and Community Medicine - Jeddah

16

The following items may need to be considered in examination of patient with


hypertension the first visit:

Blood pressure measurement:


o Tow or more
o Sitting or supine (after 2 min) plus
o Standing (after 2 min)
o Both arms (take the higher reading)

General examination:
o Height and weight (BMI), waist circumference
o Skin for signs of : mainly secondary causes
o Chronic renal failure
o Xanthelasmata
o Stigmata of cushing, Neurofibromatosis
o Yellowish finger staining
o Fundoscopy : for hypertensive retinopathy

Neck examination :
o thyroid enlargement
o distended vessels
o carotid bruits

Lungs :
o signs of heart failure (basal crepitations)
o sings of bronchospasm (to avoid b-blockers)

Heart :
o left ventricular lift, S3,S4 ( heart failure end organ damage)
o loud AS2, loud systolic murmur in chest and back , delayed femoral pulses
(coarctation of aorta)
o high pitched end diastolic murmur (aortic regurgitation apparent isolated systolic
hypertension)

Abdomen:
o masses or enlarged kidneys
o signs of chronic liver disease
o bruits lateral to midline (renal artery stenosis)
o aortic pulsation

Extremities :
o pulses for radiofemoral delay, diminished or absent
o edema, bruits and neurological assessment

CNS :
o focal deficit (old stroke)
o For elderly patients: nuro-psychiatric assessment ( multi infarct dementia )

o
o
o
o
o
o
o
o
o
o

Guidelines for blood pressure measurements


Patient relaxed in a quiet room
Use od accurate cuff dimensions
Ensure forearm is supported and at heart level
Inflate cuff to 30 mmHg above pulse occlusion
Observe at eye level with mercury column
Deflate cuff at 2mmHg per second
Record systolic blood pressure when sounds disappear (phase 5)
Measure blood pressure to nearest 2mmHg.
Measure at least twice at each consultation
At first visit measure sitting and standing blood pressure.

The Joint Program of Family and Community Medicine - Jeddah

White Coat Hypertension :


o Incidence is 10-20% of hypertensive patients.
o Diagnosis by home monitoring or ambulatory monitoring.
o No drug treatment unless there is end organ damage.
o Follow-up annually using home or ambulatory monitoring

o
o
o
o
o

When do you suspect secondary hypertension?


Renal cause: if the patient has end organ damage.
Onset of hypertension at very young or elderly (> 55 y).
Aldosteronism: if blood k+ < 3.5 mEq /L.
Pheochromocytoma:in labile hypertension with severe headache.
Coarctation: if the patient is young with systolic hypertension.

The following items may need to be considered in every visit and annually:

Well-being
Physical complaints
Any side-effects
Examination in every visit:

Blood pressure

Weight and body mass index

Chest examination
Examination and investigation annually:
o Fundoscopy
o Urinary protein
o Blood creatinine
o Blood cholesterol
Also consider:
o Chest X-ray
o ECG
o Ambulatory blood pressure
Education:

Complications of BP.

Importance of non pharmacological treatment

However medications is also sometime needed

Relative safety of anti-hypertension medications

The effect of medications to decrease morbidity & mortality

Need for good compliance with medication & advice

Need for follow-up


Management:

Shared understanding of problems

Appropriate prescribing

Advise about possible side effects

Follow-up arrangement

Stressing on non-pharmacological treatment

Discussion management or change management

Follow-up arrangements made

17

The Joint Program of Family and Community Medicine - Jeddah

18

Poor Compliance
Poor compliance is a common problem inpatient with a symptomatic chronic disease, e.g.
patient may not respond to doctors advices and he may not take his medication.
The following approach may need to be considered in dealing with poor compliant
patient:

o
o
o
o
o
o

Establishing and maintain effective doctor-patient relationship:


Great the patient, empathetic approach,
Respond to verbal & non verbal cues
Use open ended questions
Respect patient autonomy
Encourage patients contribution
Active listening

o
o
o
o

Exploration of patients health beliefs:


What the patient knows about hypertension?
What the patient knows about medications?
What dose he beliefs about himself?
What does he expect from the doctor?

Explore the social & psychological context of the problem: e.g.


possible life stresses

Assessment of degree of risk (if it is not clearly documented in the patients file)
and use the information to improve patients compliance
o Family history of BP., IHD.CVA.
o Complication of BP / DM.
o Or other significant systemic disease

o
o
o
o
o
o
o
o
o
o

Education and reassurance:


Shared understanding of problems
Discuss possible complications of hypertension.
Stress the importance of non pharmacological treatment
However medications is also sometime needed
Relative safety of anti-hypertension medications
Insurance of accessibility
The effect of medications to decrease morbidity &mortality
Emphasize the need for good compliance with medication & advice
Emphasize the need for follow-up

The Joint Program of Family and Community Medicine - Jeddah

19

Approach to Patient With Arthritis


F. Rayes
The following items may need to be considered in history taking in the first visit:

Doctor-patient relationship
Encouragement of patient contribution
Patients cues
Explore patient believes, ideas, concerns and expectations about his complain
Details of the complain:
o Pain: Onset, duration, severity, distribution, aggravating factors and
relieving factors
o Stiffness: Onset, severity, duration
o Weakness: Degree?
o Swelling
o Deformity: Malalignment, subluxation or dislocation
Occupational history:
o Repetitive overuse
o Biomechanical
o Positioning
Family history: Osteoarthritis (OA), ruematoid arthritis (RA) or gout
Systemic illness: Systemic upset in RA, gout, sepsis
Sleep disturbance or depressed mood
Specific extra-articular feature:
o Alopecia: SLE
o Rash: SLE or Reiters syndrome
o Ocular symptom: Reiters syndrome
o Oral symptom: SLE, Reiters
o Paraesthesias
o GIT or urinary symptom: May indicate inflammatory bowel disease, drug,
or fibromyalgia.
Effect on daily functioning
Shared understanding of the problem with the patient.
Exploration of any relevant continuous problem if any

The Joint Program of Family and Community Medicine - Jeddah

The following items may need to be considered in examination of patient with joint
pain in the first visit:
Inspection:

Attitude: Relax, normal, restricted movement abnormal posture

Skin Changes:
o Scars of skin disease
o Erythema: periarticular inflammation
o Red joint or bursa: sepsis or crystals

Swelling: fluid, soft tissue or bone

Deformity:
o Correctable: soft tissue abnormalities
o Non-correctable: capsular restriction or joint damage
o Muscle wasting

Inspection during active movement


Palpation:

Warmth, swelling and tenderness

Palpation during movement:


o Passive movement: crepitus
o Active movement: muscle power, instability, swelling
General examination:

Nodules: back of the hands, elbow, posterior heel, sacrum

Palmer erythema: RA

Nail changes:
o Clubbing (arthritis with COAD)
o Pitting (psoriasis, Reiters syndrome)
o Splinter: Vasculitis

Mucous membrane lesion: Reiter syndrome / Lupus

Vasculitis

Eye changes:
o Episcleritis (RA)
o Iritis: Ankylosing spondylitis chronic Reiters disease
o Iridocyclitis: juvenile chronic arthritis
o Conjunctivitis: Acute Reiters disease
Assimilation of findings:

Number of joints involved

Distribution

Degree of inflammatory component

Extra-articular features

20

The Joint Program of Family and Community Medicine - Jeddah

Differential Diagnosis of Joint Pain


Ostioartheitis Arthritis
Reuhmatoide Arthritis
Common in elderly patients Common in middle age
More in female
More in female
- Duration months to
- Duration months to
several years
several years
Clinical features:
Clinical features:
- Hand stiffness
- Morning stiffness both
bilaterally, lasting for
hands, lasting for more
less than 30 minutes
then 30 min
- Morning worse
- pain both wrists.
- Fatigues, malais, wt loss
parasthesias & vague
pain both wrists.
- Swelling, redness &
tenderness PIP &
metacarpo-phalangeal
joints
- Bony swelling, DIP
- Nondules over elbow
joints (Heberden
extenso
shoder)
- Swelling in the
- Bony swelling, DIP
metacarpophalangeal
joints (Bouchords
joints.
nodes)
Differential Diagnosis of Joint Pain (Continue)
Gonococcal Septic
Gout
Arthritis
- More in males
- Female 2/3 of patient.
- Monoarticular, first
- One or two joints, wrist,
metatarsophalagel 90%
finger, knee & ankles.
-

Very painful.
Signs of intense
inflammation.
Definitive diagnosis :
urate crystals in
synovial fluid.

Management:
- Colchicine dramatic
improvement NSAID
(indomethacin).

Migratory arthritis
Synovitis
Conococcemia : fever,
polyarthralgias & skin
eruption prior to arthitis
compare to other septic
arthritis.
Management:
- Hospitalization
Ceftriaxime 1 g IV
TDS.

21

Fibromyalgia
Common in middle age
More in female
- Chronic aching
Clinical features:
- Pain : cervical,
shoulders, pectoral
lumbosacral areas.
- Headache sleep
disturbance & fatigue
- Swelling numbness &
morning stiffness

No evidence of joint
swelling
Multiple tender areas

Nongonococcal Septic
Arthritis
- Both genders
- Swelling, pain, warmth
With severe constitutional
symptoms
- Monoarticular synovitis
particularly knee rarely
small joints.
-

75% gram positive


stapheloccocal aureas.

Management:
- Hospitalization
- Drainage & rest
- Antibiotic guided by
culture

The Joint Program of Family and Community Medicine - Jeddah

22

Preliminary Investigations:
o Erythrocyte Sedimentation Rate (ESR)
o Complete Blood Count (CBC)
o Rheumatoid Factor (RA)
o X-Ray of the affected joint

Laboratory Findings in rheumatoid arthritis:


o High ESR
o Normocytic normochomic anemia
o Rheumatoid Factor in 80% of the patient is positive
o Antinuclear antibody in 20-60% of the patient is positive
o Anti DNA is usually negative

Facts about Rheumatoid Factor (RA factor):


o It does not confirm the diagnosis of RA
o It can be of prognostic significance, as patient with high titers tend to have
severe progressive RA.
o All patient with extra-articular manifestation of RA have positive Rh factor.

Other Conditions with Positive RA factor:


o
o
o
o
o
o
o
o
o
o
o
o
o
o
o

5% of healthy persons
10-20% individual over 65 years old
SLE
Sjogrens syndrome
Chronic liver disease
Sarcoidosis
Interstitial pulmonary fibrosis
Hepatitis B
TB
Leprosy
Sub-acute bacterial endocarditis
Syphilis
Malaria
Visceral leishmania
Bilharziasis

The Joint Program of Family and Community Medicine - Jeddah

23

Approach to Patient With Backache


F. Rayes & H. Al Hajjar
The following items may need to be considered in history taking from patient with
backache

Doctor-patient relationship
Encouragement of patient contribution
Patients cues
Duration of pain, onset, quality, characteristic, location, radiation, concurrent
infection.

Risk Assessment:
Symptoms potentially indicative of serious underlying pathology; e.g. fever, progressive
severe neurologic deficits bilateral deficits, bladder dysfunction saddle anesthesia

Aggravating activities & alleviating factors

Morning stiffness relieved by activity suggests ankylosing spondylitis or other


inflammatory conditions. Worsening by standing or walking spinal sclerosis and relief
by bending Check for depression and somatization.

Work situation and situation at home

Consequences in term of daily functioning

Past history of pain

The following items may need to be considered in examination of patient with


backache
Patient Standing:

Inspection:
o Gait (patient without shoes)
o Back for scoliosis, lordosis, swelling, masses, color, & scars.

Palpation:
o Spine land marks: C7, T3 (scapular spine), T7 (inferior angle of
scapula) & L4 (iliac bone).
o Skin for hotness, tenderness (infection, fracture, ) & masses.
o Muscle spasm.
o Sacroiliac joints.
Percussion: for deep tenderness.
Movement:
o Toe-walk S1
o Heal - walk L5
o Squat & rise L4
o Movement: flexion, extenuation, lateral flexion.

Patient Sitting:

Inspection: scoliosis, muscle wasting.

Movement:
o Rotation
o Extend knees role out disc prolapse.
o Knee reflex.

The Joint Program of Family and Community Medicine - Jeddah

24

Patient Supine:
Examine free side first.

o
o
o
o

Movement:
Straight leg raising test (S L R) Active, passive & crossed SLR
Bragard test.
Lasegue test.
Figure of four (sacro-iliac joint)

Power:
o Hip flexion. L1 - L2
o Knee flexion; L5 S1
o Knee extension: L3 L4
o Foot planter flexion. S1
o Foot dorsi flexion. L4 L5
o Big toe dorsi flexion
o Foot inversion: L4 L5
o Foot eversion: L5 S1

Reflexes:
o knee reflex: L3 L4 (if not done while patient is sitting)
o Ankle reflex: S1

Sensation.
o Medial side of foot. L4
o Dorsum of foot. L5
o Lateral side of foot: S1

Patient Prone:
o Femoral nerve stretch. L4
o Compress midline as in CPR
Examination of the abdomen

Differential Etiological Factors in Backache:

Mechanical disturbance
Poor muscle tone / Poor posture / Unstable vertebrae / Severe Scoliosis
Extrinsic disease such as aortic aneurysm, uterine fibroids, prostate disease, hip
disease
Degenerative disc or facet disease
Psychological, this includes hysteria, malingering, and acute remunerative spinal
pain (Green-Poulitice disease).
Inflammatory arthritis - rheumatoid and Marie-Strumpell's disease
Infections, acute and chronic
Trauma: Acute sprain or strain / Chronic sprain or strain / Fractures / Subluxated
facet (facet syndrome) / Spondyfolisthesis with strain.
Toxicities from heavy metals
Congenital asymmetries of facets or transitional vertebrae
Metabolic disorders - osteoporosis or transitional vertebrae
Tumors:
o Benign (such as meningiomas, neuromas, osteoid osteomas, Paget's disease)
o Malignant - primary bone or neural tumors and metastases

The Joint Program of Family and Community Medicine - Jeddah

Counseling Patient with chronic backache:

Explore patients Ideas about backache, concerns & expectations.


Educate patient about backache.
o Multiple etiology &- mechanics.
o Nature.
o Prognosis.
o Why acute pain became chronic? (Social, psychological, stress)

What to do in acute attack?


(Bed rest, heat / cold, posture, analgesia)
Disc herniation may need up to 3 weeks bed rest
However there is little evidence to support prolonged bed rest (EBM)

Prevention:
o Lifting technique
o Standing
o Posture
o Seating
o Demonstrate to the patient
o Bed
o Work: chairs + desks
o Exercise
o Wt. Reduction
o Smoking, personal habits

o
o
o
o
o

Other treatment modalities:


Massage
Traction
Manipulation: osteopathy
Chiropractic
Acupuncture
Leaflets, booklets
Follow up

25

The Joint Program of Family and Community Medicine - Jeddah

26

Preliminary Investigations:

Erythrocyte Sedimentation Rate (ESR)


X-ray of lumbar spine and pelvis

The routine ordering of plan lumbosaeral spine feature is neither cost- effective nor useful
for decision making. Finding normal disc spaces does not rule out disc herniation.

Specific investigation for abdominal or pelvic causes: e.g. urine examination, renal
or pelvic ultrasound

Indications For X-ray:

Suspect malignancy (over 50 years, persistent bone pain)

Compression fracture (prolonged use of corticosteroids severe trauma, focal


tenderness)

Ankylosing spondylitis (young male, limited spinal motion, sacroiliac pain)

Chronic osteomyelitis (low grade fever, ESR)

Major trauma

Major neurological deficit


Indications For Referral:

Rapidly progressive neurologic deficits

Symptoms suggestive of cunda equina syndrome or cord compression

Suspicion of osteomyelitis or epidural abscess

Persistent pain after 4-6 weeks of conservative management

For reassurance if patient insist

The Joint Program of Family and Community Medicine - Jeddah

2.

27

History Taking
N. Dashash, A. Assaggaf, A. Al Harthy & H. Al Hajjar

In history taking stations some time patient presents with typical story, the candidate may
reach the final diagnosis from the first impression and ignore to ask relevant and specific
questions to prove it objectively. Candidates are advised to think loudly to give the
examiner the chance to understand how he think, in order to give him the desirable
evaluation mark
(For more advices see simulated clinic exam)

(1) History Taking from Patient Complaining of Palpitation


A. Al Harthy
The following items may need to be considered:

Introduce yourself and establish doctor-patient relationship


Encourage patient contribution
Respond to patients cues
Clarification of the symptom (what the patient means by palpitation).
Exploration of the details of the main complaint:
Onset, course, frequency, duration, rate, rhythm, pattern (how it end abrupt or
gradual), precipitating factors and relieving factors.
Associated symptoms: e.g. chest pain, shortness of breath, fainting, sweating,
tremor, feeling of tension (fear), any symptoms of thyrotoxcosis.
Presence of mood changes, concentration or memory problems any phobias (e.g.
social phobia).
Past history of cardiac disease.
Drug history or stimulant use: e.g. tea, coffee (its amount).
Social history & any history of stressful life events.
Exploration of patient s ideas, concerns and expectations and the effect of the
problem in patients life.
Family history of.
Exploration of any continuous problems or at risk factors: DM, hypertension,
asthma or smoking (the amount of smoking).

A good history is often diagnostic, particularly with respect to precipitating factors.


Differential Hypotheses
o Anxiety
o Cardiac causes: e.g. sinus ventricular tachycardia, ventricular ectopics, atrial
fibrillation or flatter or sinus tachycardia
o Drug induced arrhythmias
o Thyrotoxicosis or anemia
Management:
o In case of infrequent palpitation or missed beats with no associated symptoms:
reassurance and advice patient to avoid the precipitating factors
o In case of frequent palpitation associated with chest pain or breathlessness:
- Urea and electrolyte
- TFTs and treat if abnormal
- ECG: if normal refer for 24 hours ECG

The Joint Program of Family and Community Medicine - Jeddah

(2) History Taking from Patient Complaining of Tremor


N. Dashash
The following items may need to be considered:

o
o
o
o
o
o
o
o

o
o
o
o

o
o

o
o
o

Introduce yourself and establish doctor-patient relationship


Encourage patient contribution
Respond to patients cues
Exploration of patient Ideas, Concerns, Expectations and feeling.
Problem identification:
What is meant by tremor?
Is it visible or only a sensation of tremor?
Duration of the problem?
Onset gradual or sudden?
Part of the body involved? (Head, voice, trunk...
Type of movement and its spread.
Is it a resting tremor or a movement tremor?
Aggravating and relieving factors.
Other associated symptoms of:
Cerebellar disease
Hyperthyroidism
Anxiety
Parkinsons
The effect of the problem on the patient's life (e.g. work, marital life).
History of drugs: e.g.
Lithium
Tricyclic antidepressants.
History of alcohol drinking and smoking
Past medical history:
Brain disorder or head trauma
Multiple sclerosis
Other diseases, e.g. DM or hypertension.
Family History of similar problems.

Differential Hypotheses:
o
o
o
o
o
o

Alcohol
Medications
Cerebellar disease
Hyperthyroidism
Anxiety
Parkinsons

28

The Joint Program of Family and Community Medicine - Jeddah

29

(3) History Taking from Patient Complaining of Pruritis


N. Dashash & F. Rayes
The following items may need to be considered:

o
o
o
o
o
o

o
o
o
o
o
o
o

Introduce yourself and establish doctor-patient relationship


Encourage patient contribution
Respond to patients cues
Identification of the problem:
What is meant by pruritis?
Which part of the body is involved?
Onset, duration, clinical course.
Precipitating factors.
Severity of pruritis (interfering with sleep and daily activity)
Any associated skin rash or any skin changes (describe it).
Associated symptoms of:
Hyperthyroidism.
Renal failure
Drug reaction,
Iron deficiency anemia
Malignancy
Liver failure
Pregnancy.
Sensitivity to food, soap, perfumes...
Any thing new in the patient's life.
Past medical history
Drug history: opiates, amphetamine, quanidine, aspirin, and vitamin B.
Occupational history.
Family history of similar problem.
Psychological stress.
Exploration of patient ideas, concerns, expectations and feeling.
Exploration of the effect of the problem on the patient's life (work, marital life.

Systemic Causes of Pruritus

Cholestasis:
o Primarily biliary cirrhosis,
pregnancy
o Extrahepatic obstruction, drugs e.g.
Contraception

Endocrine:
o Thyrotoxiosis, myxoedema
o Hyperparathyroidism, DM
Investigation of Pruritus:
With no overt skin disease
o Urine: Glucose, protein, and urine
microscopy
o Stool: Occult blood & parasites
o CBC, differential white blood count &
blood film
o ESR, U & E, serum iron and uric acid
o TFT & LFT
o Chest-X Ray

Haematological / myeloproliferative:
o Iron deficiency, polycythemia
o Hodgkins disease, multiple
myeloma

Chronic Renal Failure

Malignency
o Miscellaneous e.g. gout,
psychological, old age.
Management of Pruritus
o Elimenation of underlying cause
o Emollients as dry skin the most
common cause of itching.
o Calamine lotion
o Crotamiton (Eurax) cream 0.5%
o Systemic antihistamines e.g. Benadry
QID, Atrax, Hisenanal.
o Topical corticosteroid ointment
o Unidazole combination.

The Joint Program of Family and Community Medicine - Jeddah

30

(4) History Taking from patient Complaining of Anxiety


N. Dashash
The majority of patients may present with somatic complaint, e.g. palpitation or dyspnoea
or chest tightness. Candidate needs to suspect the diagnosis from the patients verbal and
nonverbal cues, e.g. excessive hand movement
The following items may need to be considered:

Introduce yourself and establish doctor-patient relationship


Encourage patient contribution
Respond to patients cues
Identify the problem:
o Duration of symptoms, onset: sudden or gradual
o Course: continuous or episodic (to rule out panic attacks and phobia)
o Change in severity
o Precipitating factors e.g. problem at work or home, death of relative

Exploration of patient ideas, concerns, expectations and feelings

Effect of the problem in patient life (physical, social and psychological)

Etiology: It is necessarily to exclude the following diseases:


o CVS diseases: ask about palpitation, chest pain, paroxysmal nocturnal dyspnea,
relation of symptoms to exercise and irregular beats.
o Depression: ask about loss of interest, low mood, change in wt ... etc.
o Hyperthyroid: ask about heat intolerance, excessive sweating...etc.

Symptoms of anxiety disorder (diagnostic criteria):


o Psychological: excessive worrying, nervous mood, apprehension, irritability,
disturbed sleep, and difficulty in concentration. Psychotic features (e.g.
hallucinations).
o Neurological: dizziness, headache, twitching, paraesthesia, blurring of vision
o CVS: palpitation, chest discomfort, flushing
o Respiratory: hyperventilation, difficult breathing, and chest tightness.
o GIT: dry mouth, nausea, difficult swallowing, choking, frequent loose motions,
abdominal discomfort, wt. Loss, and nausea/vomiting
o Urinary: frequency and/or urgency.
o Menstrual disturbances and reduce libido.
o Others: muscle aches and tension, tiredness

Drugs History: e.g. Alcohol, drugs, benzodiazepene withdrawal, caffeine (amount)


and smoking history.

Past History:
o Similar problems, or any psychiatric problem
o Medical or surgical problem (hypertension, DM, hyperthyrodism or asthma.)
Family history of similar problem or any psychiatric problems

The Joint Program of Family and Community Medicine - Jeddah

31

(5) History Taking from Patient Complaining of Depression


A. Assaggaf
The majority of the depressed patients may present to their family doctor complaining of
fatigue or somatic symptoms. Candidate needs to suspect the diagnosis from the patient
verbal and nonverbal cues, e.g. lack of eye contact, self-neglect, or multiple somatic
complains does not fit to any diagnosis
The following items may need to be considered:

Introduce yourself and establish doctor-patient relationship


Encourage patient contribution
Respond to patients cues
Clarification of the symptom
History of the problem:
o Duration
o Mood
o Loss of interest in his usual activities
o Activity level:
Decrease, retarded, slow, loss of energy or increase, restless or agitated.
o Diurnal variation
o Sleep disturbance: (increase or decrease)
o Change in appetite and weight
o Loss of libido
o Decrease ability to concentrate
o Guilt feelings
o Suicidal thoughts and/or attempts
Patient's ideas, concerns, expectations and effects of the problem
Etiology of the problem
Psychosocial history:
o Home environment,
o Emotional problems
o Financial problems
o Loss of job or loss of relative.
Drug History
o Substance abuse
o Antihypertensives or steroids.
Presence of somatic complaint e.g. headache, back pain, shortness of breath, ---etc.
General medical illness
Past history of similar condition (or other psychiatric illnesses)
Family history of similar condition (or other psychiatric illnesses)

The Joint Program of Family and Community Medicine - Jeddah

32

Prescribing of Antidepressants
Fayza Rayes
Associated problems
Depression
Depression with
Psychosocial stress
Depression with
Anxiety symptoms

Depression with
insomnia

Suggested
Treatment
1) SSRIs
2)Tricyclic antidepressants
1) SSRIs
2)Tricyclic antidepressants
1) Imipramine (Tofranil)
2) SSRIs (Prozac)
3) (Tofranil) + Alprazolam
Benzodiazepin (Xanax)
Amitryptylin (Tryptezol)

Dothiapin (Prothadin)
Maprotiline (Ludiomil)
Trazodone(Trazolan)
Depression with DM
Depression with CHF
or IHD
Depression with
Arrhythmia
Depression with
Hypertension
Depression with
Hypotension

SSRIs
SSRIs

Depression with
Urologic disease
Depression with
Parkinson 's Disease
Depression with Stroke

Trazodon (Trazolan)
SSRIs
Nortriptylin (Nortrilen)
Trazodon (Trazolan)
SSRIs
Nortriptylin (Nortrilen)
Amitryptylin (Tryptezol)
Imipramin (Tofranil)

SSRIs
Trazodon (Trazolan)
SSRIs
Trazodon (Trazolan)
Nortriptylin (Nortrilen)
SSRIs

Comments
Consider the cost
Social support
Psychotherapy
(75-150 mg)
(20-60 mg)
(250-500 microgram) TDS for 6Ws
then tapered slowly over 4 Ws
(25-300mg)Strong sedative, dizziness
anticholinirgiceffects:wt.gain,constipa
tion hypotension, cardiotoxic,
tachycardia
(25 250 mg)Less cardiotoxic, less wt
gain, not in pregnancy
(75 150 mg) not in eplipsy
(150-400mg in deveded dose)
hypotension
No Wt. gain
Less cardiotoxic
Less cardiotoxic
(see insomnia)
(see insomnia)
(10-150mg in devided dose)
no hypotension, no sedation,
less anticholinigic, safe in elderly
(see insomnia)
(see Hypotension)
(see insomnia)

(see Hypotension)
Depression with
(see Insomnia)
Migraine headache
(25-300 mg in devided dose) not
sedative constipation, nausea,
headache, sexual disfunction
Nortriptylin (Nortrilen)
(see Hypotension)
Depression with
Amitryptylin (Tryptezol)
(see Insomnia)
Chronic urticarea ,
Imipramine (Tofranil)
(see Migraine)
allergic disease
Nortriptylin (Nortrilen)
(see Hypotension)
Effect after 2 weeks and diagnosis of medication failure after 2 months
Acute treatment for 6-12 months while continuation treatment for 4-6 months
And maintenance for chronic relapsing patients for years

The Joint Program of Family and Community Medicine - Jeddah

33

Choices of Antidepressants
Fayza Rayes
Drug & Dose
Amitriptylin
(Tryptyzol)
25-300 mg
in devided dose
Imepramine
(Tofranil) 25-300 mg
in devided dose
Clomipramine
(Anafranil)
10-250 mg

Significant Side effect


Histaminic blockade / Sedative
Anticholinergic effect
Alpha 2 adrenoreceptor
blokade Hypotension/sexual
dysfunction
Seratonin uptake blokcade /
Constipation, dizziness
Not sedative
Seratonin uptake blokcade /
Constipation, dizziness
Not sedative

Nortriptylin
(Nortrialen)
10-200 mg / day
Maprotiline
(Loduomil)
25- 150 mg
Trazodone
(Trazodam)
150- 400 mg / day

No hypotension, no sedation
Less Anticholinergic effect

Sertralin
(Zoloft)or (Lustral)
25-100 mg
Fluoxetine
(Prozac)
10- 80 mg / day
Citalopram
(Cipram)
20-60mg
Fluvoxamine
(Faverin)
100-300 mg
in devided doses
Parooxetine
(Seroxat)
20-60 mg

Indication
Depression with insomnia
Migraine
Not for hypotensive patient

Depression
GAD
Panic attacks
OCD (Drug of first choice)
Phopias
Panic attacks (Drug of first
choice)
Elderly

Not in epelipsy or risk of


convulsion

Depression with insomnia

Minimum effects
Hypotenssion
Priapism (1/6000 cases)

Depression with insomnia


Strong sedatve / Insomnia (half
tablet for 3 days)

Seratonin uptake blokcade :


GIT / anxiety / tremor /
insomnia / palpitation
/drowsiness / agitation /
hypomania / hypotenssion /
convulsion / movement
disorders & dyskinezia /
neuroleptic malignant
syndrome / violent behavior /
hematological complications

Depression 50-200 mg
Maintenance 50 mg

Cautions: cardiac diseases


Mania / epelipsy / concurrent
ECT / hepatic or renal
impairement / pregnansy /
breast feeding / interactions

Depression 20 mg
Bulimia 60 mg
OCD 20- 60 mg
GAD & Panic attacks
Depression 20-60 mg
Panic disorder 10mg increase
evrey week by 10mg / mux 60
mg
Depression 100-300 mg
OCD (the drug of first choice)
Insomnia

Depression 20mg increse by 10


mg every week / mux 50 mg
OCD / start by 10 mg / mux 60
mg
Effect after 2 weeks and diagnosis of medication failure after 2 months

The Joint Program of Family and Community Medicine - Jeddah

(6) History Taking from Patient Complaining of Dizziness/ Vertigo


H. Al Hajjar & M. Alatta

Doctor-patient relationship
Encouragement of patient contribution
Respond to patients cues
Clarify what patient means exactly by dizziness
(Is it true vertigo or light headedness or disequillibrium)
Severity of symptom: e.g. associated nausea and/or vomiting.
Effect of problem on patient's life, his ideas, worries & expectations.
Course: constant or attacks (duration & frequency)
Onset and timing.
Precipitating factors:
o Change in head position
o Valsalva maneuver
o Standing
o Viral infection
o Auricle manipulation,
o Hyperventilation
o Fatigue
o Explosion
History of pervious attacks.
Ear disease:
o Hearing loss / tinnitus.
o Fullness or stuffiness
o Otalgia / discharge.
o Pervious ear surgery.
Rule out associated brain stem symptoms:
o Double vision.
o Numbness and/or weakness in arm face and leg.
o Difficulty in speech.
o Confusion or loss of consciousness.
o Swallowing problems.
Associated symptoms:
o Valvular disease.
o Palpitation.
o Syncope on exertion.
o Prolonged bed ridden.
o Head & neck trauma
o Seizure
o Symptoms of DM, hypertension, anxiety, depression or panic attacks.
Drugs history

Differential Diagnosis of Vertigo:

Benign paroxysmal vertigo


o Sudden vertigo positional changes (e.g. head turning)
o Recurrent lasting minutes to hours.
o Released if pt is motionless.
o Associated Nystagmus
o Adult uncommon in children.

34

The Joint Program of Family and Community Medicine - Jeddah

35

Menieres disease.
o Sudden onset. Common in adults
o Recurrent similer attack with Adults long free intervals.
o Lasting hours to days
o Associated with tinnituss, hearing loss, ear fullness and
naauseaa+vomiting.nystagmuss presented by menstruation, emotional
stress

Vestibular Neuritis or Labyrithitis.


o Sudden onset after U.R.T.I lasting days to weeks
o Nausea & vomiting
o Tinnitus hearing loss

Acoustic Neuroma
o Gradual onset. Onset in adult
o Persistent
o Progressive. Unilateral hearing deficit. Tinnitus.
o Facial numbers, weakness
o Diplopia, dysarthria, dysphagia, Dysporiea
o Uncordination, Paraesthsias

Vertebro Basillar insufficient.


o Acute onset. Onset in elderly
o Recurrent
o Brainstem Symptoms. No Nausea OR vomiting
o Nystagmus.

Multiple Sclerosis
o Sudden or transient, persistent as or Recurrent
o Lasting days pr wks.
o Other discrete CNS symptoms.

Acoustic Neuroma.
Gradual/ onset in adult:
o Persistent
o Progressive, unibateral hearing deficit, tinnitus.
o Numbness, weakness, diplopia, dysanthria, dysphagia, dysphonia
o Uncordination and paraesthsias.
Acute/ onset in elderly:
o Recurrent
o Brain Stem Symptoms: No Nausea OR vomiting
o Vertebro Basillar insufficiency: Nystagmus

Drugs:
o Antibiotics
o amino glycosides
o Quinines
o Anticonvulsants,
Hypnotics
Disappear when Drug Discontinued.

o
o
o
o

Antidepressent
Diuretics & antihypertensive
Hydrocarbons exposure
Organic -Carbon Monoxide.
Exposure

The Joint Program of Family and Community Medicine - Jeddah

36

(7) History Taking from Male Complaining of Impotence


H. Al Hajjar
The majority of impotent patients do not complain directly from impotence. Candidate
needs to suspect the hidden agenda from the patients verbal and nonverbal cues, e.g.
patient may ask for vitamins or any other tonics, or he may complain of backache or
psychological symptoms
The following items may need to be considered:

o
o
o
o
o
o

o
o
o

Introduce yourself and establish doctor-patient relationship


Encourage patient contribution
Respond to patients cues
Details of the complain: onset & course of the impotence
Degree of dysfunction: chronic, occasional or situational.
Early morning and nocturnal erection.
Is there other wife? Is the problem the same with her?
Precipitating factors:
Is the marriage stable & Happy?
Does the wife contribute to the problem?
History of: pelvic trauma, pelvic surgery, and spinal cord surgery.
History of: diabetes, renal failure, hepatic cirrhosis,
Medications: Diuretics, antihypertensives, H2 blockers
Antidepressant or alcohol.
Any new stressful event
Associated symptoms:
Loss of libido, Gynecomastia.
Presence of visual or neurological symptoms.
Psychosocial history: depressive symptoms
Exploration of patient's ideas, concerns & expectations.
Effect on patient life & relation to wife and family.
Previous treatment modalities

The Joint Program of Family and Community Medicine - Jeddah

(8) History Taking from Female Complaining of Infertility


A. Assaggaf
The following items may need to be considered:

o
o

Introduce yourself and establish doctor-patient relationship


Encourage patient contribution
Respond to patients cues
Detail of the complain:
Nature of the problem. Is it infertility?
Duration
Type of infertility: primary or secondary?
Patients ideas, concerns, expectations & Effect of the problem.
Etiology
Menstrual History
Age of the patient
Age at menarche
Duration of periods
Menstrual irregularities
History of amenorrhea
Obstetric History:
Previous pregnancies
Abortion
Ectopic pregnancy
Complicated deliveries.
Gynecological History
History of PID
Fibroids
Marked weight loss
Excess exercise
Symptoms of general diseases e.g. DM., hypothyroidism or hyperprolactinemia.
Marital relationship
Duration of marriage
Sexual activity: (technique and frequency).

o
o
o
o

Information about the husband:


Age, occupation (exposure to toxin or radiation)
History of previous marriage, children & age of youngest child.
Past medical history
History of mumps, undescended tests, varicocele.

o
o
o

o
o
o
o
o

o
o
o
o

o
o

Family History of infertility, DM or chromosomal abnormalities.

37

The Joint Program of Family and Community Medicine - Jeddah

(9) History Taking from Patient Complaining of


Vaginal Discharge
A. Assaggaf, M. Alatta & F. Rayes
The following items may need to be considered:

o
o
o

o
o
o
o
o

o
o
o
o
o

o
o
o
o
o

Introduce yourself and establish doctor-patient relationship


Encourage patient contribution
Respond to patients cues
Identify the problem:
What does the patient mean by vaginal discharge
Onset
Duration of symptoms
Description of the discharge in relation to:
Amount,
Color & appearance
Odor
Consistency
Any associated blood with the discharge.
Associated symptoms:
Dysurea
Pruritus
Pelvic pain
Dyspareunia
Fever or skin rash.
Menstrual history and LMP.
Relation to menstrual cycle
Relation to intercourse
Symptoms in the partner
Previous history of discharge
Possible etiological factors: e.g.
DM
Concurrent use of medications (steroids or antibiotics),
Use of tampons, pessaries & antiseptics,
Possible exposure to STD
Use of contraception (Pills, IUD).
Exploration of patients ideas, concerns and expectations.
Exploration of patients fear or anxieties related to the discharge.
Exploration of the effect of the problem in patients life
(Physical, social and psychological)
Differential diagnoses for causes of vaginal discharge

Symptom
o Dysuria
o Altered bleeding patternd
contraception-

o Abdominal pain

Alternative Diagnosis
Urinary tract infection
Side-effect of hormonal method of
combined and progesterone-only contraceptive pills,
injectable contraception (Depo-Provera), implants n)
or intrauterine system (Mirena)
Irritable bowel syndrome, constipation,
endometriosis

38

The Joint Program of Family and Community Medicine - Jeddah

o
o
o
o
o
o

Differential diagnoses for causes of vaginal discharge (continue)


Candida:
Most common
Itchy, white, thick, lumpy discharge
Bacterial vaginitis:
Copious greyish & fishy smelling, burning or itching
Trichomonas vaginalis:
Offensive, greenish yellow, thin, bubbly discharge, with pruritis valvae.
Chlamydia:
8% of VD in UK, and only 0.5% of VD in KSA
Gonorrhoea:
Very uncommon cause purulent vaginal discharge.

Microbiological investigations for vaginal discharge.


Investigations
o High vaginal swab

Infection
Candidiasis, bacterial vaginosis,
Trichomonas vaginalis

o Cervical swab

Gonorrhea

o Endocervical swab (culture, direct


fluorescent antibody or enzyme
Immunoassay)

Chlamydia trachomatis

Investigations for no infective causes of vaginal discharge

Investigation
o Urinalysis/blood glucose

Cause
Diabetes

o Midstream urine

Urine infection

o Serum follicle-stimulating
Hormone and oestradiol

Oestrogen deficiency
-perimenoausal
-Inadequate hormone replacement
Treatment

Treatment of vaginal discharge:


Condition
Drug Treatment
Bacterial vaginosis Metronidazole (Flagyl) 400 mg BD X 5 ds
Or Nimorazole (Naxogin 500) 2 g single dose.
Trichomoniasis

Metronidazole 400 mg BD X 5 days or 2 g single dose.

Candidiasis

Clotrimazole (Canesten) single 500 mg pessary


Or Nystatin 2 pessaries at night/ 2 weeks.

39

The Joint Program of Family and Community Medicine - Jeddah

40

(10) History Taking from Patient Suspecting to have


Sexually Transmitted Disease (STD)
A. Al Harthy
Candidate may suspect STD from the patient main complaint, e.g. urethral discharge or
ulcer in genital area. Other helpful cues should also be considered, e.g. travel history,
young age, and single
The following items may need to be considered:

Introduce yourself

Establish good rapport


o Be communicative
o Non-judgmental,
o Confidentiality is essential,

Encouragement of patient contribution

Respond to patients cues

Specific information:
o Age, Job, Travel history
o Marital status and sexual contact,

Main complaint; (e.g. detail of urethral discharge, type, color, associated irritation,
blood staining, odor, duration).

Presence of itching, dysuria, frequency,

Rash: how it progress. Ulcers: how and where the lesions first appear, any
prodromal symptoms suggestive of herpetic infection, any rash involving other parts of the
body (i.e. palms & soles for syphilis, axillae and wrists for scabies).

Associated symptoms: e.g. fever, swelling, joint pain, abdominal pain, pelvic pain,
conjunctivitis, weight loss, cough, diarrhea, fatigue, headache).

Presence of symptoms in husband or wife,

Previous similar attack,

Past history of STD,

Drug abuse or use of alcohol,

Any contraception: e.g. barrier contraception.

Any drug use or self-treatment.

Exploration of patient s ideas, concerns and expectations and the effect of the
problem in patients life.

If the patient is female: ask about her gynecological history:


o LMP
o Dysparunia: superficial or deep
o Dysuria: external or internal

The Joint Program of Family and Community Medicine - Jeddah

41

(11) History Taking from Patient with Epistaxsis


M. Alatta

o
o
o
o

o
o

o
o
o
o

o
o
o
o
o
o
o
o
o

o
o
o
o
o

Introduce yourself and establish doctor-patient relationship


Encourage patient contribution
Respond to patients cues
Identify the problem:
How severe is it?
When did it begin?
How is the patient responding?
Is the patient alone?
Nasal Characteristics:
Is there respiratory difficulty?
Can the patient breath through the mouth without difficulty?
History of possible etiological factors:
Any trauma
Any foreign body
Any drugs taken
Any underling medical condition

Instructions For Nose Bleed


Sit up.
Pinch soft parts of nose to gather between thumb and index finger.
Breath through mouth.
Hold pinched nose for 10 minutes without letting go.
Fill a plastic bag with crushed ice, wrap bag in a towel, and apply to upper nose.
Remain sitting in a quiet environment for 30 minutes.
Avoid blowing or picking nose (this will remove blood clot, and bleeding will
reoccur.
Avoid straining or lifting this may increase pressure and bleeding will reoccur).
Hot, cold, or windy weather may cause drying and crusting of the nasal / mucus
membranes. Heaters and air conditioners add additional drying to the environment.
To prevent nose bleeds:
Use a vaporizer or humidifier, especially at night.
Lubricate the anterior nasal opening with a small amount of Vaseline.
Avoid picking or intense blowing of the nose.
If bleeding reoccurs, nasal / packing or cautery may be necessary.
For frequent nose bleeds, call for an appointment with a health care provider to
evaluate the cause.

The Joint Program of Family and Community Medicine - Jeddah

3.

Physical Examination
H. Al Hajjar, A. Al Harthy & A. H. Hassan

General Instructions:

In joint examination lists, each list is comprehensive and cover all areas

Special tests are not necessary to be done in all patients according to the case

Always great patient and introduce your self by name and specialty

Remember proper exposure of the patient

Always expose and examine both sides for comparison

Ask patient permission

Tell the patient what you are doing

Gentle approach

Be systematic

Thanks the patient at the end

Wrest & Hand Examination


H. Al Hajjar & M. Alatta

o
o
o
o

o
o
o
o
o
o

o
o
o
o
o
o
o

o
o
o

Inspect: dorsum, palmer & sides:


Deformity: Radial or ulnar deviation, RA. (Swan neck,
Boutonniere & Z deformities)
Nerve injury deformities (drop wrist, claw hand)
Wasting: thenar, hypothenar & interossious.
Skin: swelling, scars, color
Palpate:
Skin for temperature.
Wrist joint
Snuffbox
Over nerves: median & ulnar.
Over sheet of abductor pollices longus & extensor
Pollices brevis (dorso lateral aspect of radius)
Move:
Dorsiflexion.
Palmerflexion.
Radial deviation.
Ulnar deviation.
Finger adduction.
Finger abduction.
Opposition.
Special tests:
Power and sensation:
Carpal tunnel syndrome:
Thenar eminence for wasting.
Tinel's test: taping over the median nerve at the carpal tunnel
Causes pain or numbness in median nerve distribution.

42

The Joint Program of Family and Community Medicine - Jeddah

43

o Phalen test: flexing the wrist 90 for one minute causes numbness and parasthesia.
o Flick test: move hands similar to shaking thermometer will relief the pain.
o Abductor pollices breves examination: resisted thumb abduction. -opposition.

Ulnar nerve syndrome:


o Forman's sign: paralysis of adductor pollices.
o Claw hand
o Sensory loss: anterior & posterior aspect of little and lateral half of ring finger.

Management of Carpal tunnel syndrome

o
o
o
o
o
o

None-surgical in non-complicated, short duration less> 1 year;


Wrist support
Palmer wrist splint at night
Advice about work related hazard
NSAI drug
Pyridoxine
Local steroid injection

Surgical
Simple decompression

The Joint Program of Family and Community Medicine - Jeddah


o

(1) Shoulder Examination


H. Al Hajjar

Ask patient to Stand and expose:


o Both shoulders
o Neck
o Upper chest
Look always for patient face (pain)
Tell the examiner what you are doing
Inspection:
In Good light, compare both sides, front, side, back, above &
Axilla, Skin for Scars, redness, bruising or swelling
Front:
o Soft tissue. Contour of shoulder muscle wasting
o Bones & joints:
Prominent sternoclavicular joint
Clavicle deformity
Prominent acromioclavicular joint
Side:
o Glenohumeral joint
Behind:
o Scapula shape & position
o Neck webbing
Above:
o SupraclavicIlar fossa, shoulder,
o Clavicle swelling, deformity, asymmetry
Palpation:
o Skin for hotness
o Bony points:
Anterior & lateral sides of Glenohumeral joint
Upper humeral shaft (from axilla)
Acromioclavicular joint lipping & Crepitation
Clavicle
Greater tuberosity
o Soft tissue.
Deltoid bursa
Biceps tendon
Supraspinatus tendon
Movements:
active, passive if restricted
o Abduction & adduction (observe painful arc)
o Flexion
o Extension
o Apply's scratch test:
Internal rotation
External rotation
o Push against wall (winging of scapula)
Power:
resisted movements
o Long head of biceps speed test
o Supraspinatous resisted can emptying position
o Infraspinatous resisted external rotation
o Subscapularis resisted internal rotation

44

The Joint Program of Family and Community Medicine - Jeddah

o
o
o
o
o

Special tests:
Cross over test (acromioclavicular joint)
Anterior drawing test / apprehension test (dislocation)
Posterior drawer test (dislocation)
Clunk test (labral tear)
Impingement test

Examine the neck.


Aply's scratch test:
o Ask patient to put hands behind neck, elbows backward
(Tests: external rotation & Abduction)
o Ask patient to put hands behind back reaching up as he can
(Tests. Internal rotation & adduction)
Cross over test.
Reach across chest to other shoulder
Tests: acromioclavicular joint
Anterior drawer test:
o Patient supine at bed edge
o Steady scapula with your left thumb on coracoids fingers behind draw anteriorly
the head of humerus by your right hand
o Observe for click, movement or apprehension
Apprehension test:
Patient sitting, doctor behind patient stabilize scapula
Abduct shoulder 90 , elbow flexed 90 , externally rotate
Apprehension Not pain positive test
Posterior drawer test:
o Patient supine shoulder flexed 20 abducted 90
o Place thumb just lateral to coracoid
o Rotate shoulder internally & flex 90
o Observe movement by your thumb
Clunk test:
o Elbow flexed, shoulder abducted
o Rotate while applying axial pressure
Speed test:
o Force downward patient's supinated arm with elbow flexed 90 , shoulder
o adducted, observe biceps swelling.
Impingement test:
o Patient standing, doctor from behind
o Ask patient to flex shoulder forward
o Apply pressure downward over shoulder
o Look for pain

45

The Joint Program of Family and Community Medicine - Jeddah

(2) Knee Examination


H. Al Hajjar
Expose both knees and start by normal knee.

Patient Standing:

Look.
o Gait (observe patient walking)
o Bone deformity. Genovarum / genovulgus.

Patient Supine:
Look:
o Size and shape of patella.
o Quadriceps wasting
o Skin: color, scars & swelling

o
o
o
o
o
o
o
o

Palpate:
Temperature: all sides & compare.
Tenderness: slight flexion.
Attachment of collateral ligaments.
Joint line: menisci.
Tibial tubercle.
Anterior surface of patella.
Synovial membrane thickening.
Effusion:
- Patellar tap.
- Fluid displacement test (most reliable)
- Fluctuation test.

Move:
flexion & extension
o Active
o Passive.
o Against resistance

o
o
o
o
o

Special tests:
Varus & vulgus stress instability (for collateral ligaments)
Anterior & posterior drawer tests for cruciate ligaments.
McMurray manuver for menisci.
Friction test. (For patello-femoral joint.)
Apprehension test. ( For patello-femoral joint.)

Patient Prone: popletial fossa


o Look: mass, deformity, scar and skin color.
o Palpate. Mass, pulsation, tenderness and temperature.

Examine hip & ankle joints quickly (movements)

46

The Joint Program of Family and Community Medicine - Jeddah

(3) Ankle Examination


H. Al Hajjar

o
o
o
o
o
o

o
o
o

o
o
o

o
o
o

Inspection:
Gate
Deformity
Skin
Swelling
Compare with other foot
From behind
Palpation:
Site of pain
Tenderness at ligaments sites, anterior capsule, lateral malleolus and base of 5th
metatarsal
Skin temperature
Movements:
Active planter and dorsiflexion
Passive planter and dorsiflexion and inversion and eversion
Compare both sides
Special movements:
Anterior drawer test
Talar tilt test (inversion stress test)
Comparing two sides
Quick knee examination
Arrangement

Important ligaments around ankle joint:


o Anterior talofibular
o Posterior talofibular
o Calcaneofibular
Most common injuries;
o Strain: does not involve joint instability or ligament tear
o Sprain: stress applied to ankle in an unstable position causing ligaments to
overstretch
Degrees of ankle sprain:
o First degree: stretching of ligamentous fibers
o Second degree: tear of some fibers
o Third degree: complete ligamentous separation
Inversion injury is most common injury

47

The Joint Program of Family and Community Medicine - Jeddah

48

(4) Mental Status Examination


A. Al Harthy

Mental state appearance and behavior:


o Appearance: dress, posture, facial expression, eye contact
o Example: poor eye contact, masked face or crying in depressed patient
Speech and preoccupations:
o Rate of speech:
Example: pressured as in mania
Retarded as in depression
o Tone:
o Example: monotonus tone in depressed patient
(Normal tone in variable)
o Form: Thought block as in schizophrenia
o Content: depressive ideas in depression
hypochondrial ideas as in hypochondriasis, which is a preoccupation with
a fear of having a serious disease.
Abnormal beliefs:
o Delusions: Which is a false belief
Example: persecutory, grandiose, thought broadcasting in schizophrenia
Mood:
Example: anxious in anxiety
Depressed in depression
Phobias and Obsessions:
o Phobic anxiety symptoms and its effect
o Example: avoidance behavior as in social phobia
o Obsess ional ideas or compulsion
These symptoms you need to ask specifically about it patient many times dont
volunteer it
Abnormal experiences
Depersonalization
Hallucinations: which is a false perception arising without external stimulus
Example: auditory hallucination in schizophrenia
Cognition:
o Orientation: to time and place
o Attention and concentration:
o Example: asking patient to mention days of the week in the opposite order.
o Memory: short term
Long term
Example: patient with Alzheimer dementia cant remember what they cat
for lunch. But can remember some events happened 20 years
back.
Insight and Judgment:
Insight: is weather the patient is aware of his illness or not.
Intelligence: more special test is done for that by more specialized personnel.

The Joint Program of Family and Community Medicine - Jeddah

(5) Painful Red Eye examination


A. H. Hassan & A. Al Harthy

Swelling /secretion / redness scars / pallor


Palpation for tenderness and tension.
Size and reaction to light of the pupils.
Visual acuity:
Far vision
o Cover the other eye
o Use the chart and record the finding
Near vision
o Field confrontation method.
o Color vision: using lshibara plates.

Funduscopy:
o Dilate / darken the room
o Check the machine, proper power.
o Red reflex.
Important diagnosis not to be messed
Acute angle closure glaucoma:
It is an emergency, rare to occur, but the patient can lose vision if not managed early.

Symptoms:
o Red painful eye
o Nausea and vomiting
o Blurred vision
o The patient may give a history of similar attack in the past that were aborted by
going to sleep

Signs:
o Red and tender eye
o Hazy cornea
o Fixed, semi dilated pupil to light

Management:
o Emergency treatment is needed and documents the level of vision in each eye.
o Acetazolamide (Diamox) 500 mg oral or I.V.
o Timolal 0.25 0.5% drops
o Rilocarpine 0.5 4% drops
o Then patient referred to the emergency ophthalmology care for
- Continue medical treatment to I.O.P.
- Surgery (iridectomy) or lazer

49

The Joint Program of Family and Community Medicine - Jeddah

Chronic Open-Angle Glaucoma

Symptoms: can be asymptmatic,


o Restriction of visual fields.
o Gradual loss of peripheral vision, (tunnel vision)

Signs:
o Loss of temporal quadrant (visual field).
o Cupping and pallor of the optic disc.

Investigations:
o tonometry: Raised intraocular pressure.
In 3% pressure is in the normal range
o Ophthalmoscopy to determined cup/disc ratio)
o Ccomputerized perimetry.

Treatment,
o Topical beta-blocker, as timolol eye drops 0.25 twice daily.
(Conttraindicated in heart failure and asthma).
o Surgery: if medical treatment fails.

(6) Optic Nerve Examination


A. H. Hassan

Visual Acuity:
o Far vision:
Chart + 6 m., cover the other eye.
o Near vision

o
o

o
o
o
o

Field:
Confrontation method.
One meter apart, cover one eye, Eye fixed, bring the object into the field.
Color Vision
lshibara plates.
Funduscopy:
Dilatation, Check the machine, darken the room, and fix a target.
Right hand for right eye, from the tight side.
Red reflexes
Optic disc.

50

The Joint Program of Family and Community Medicine - Jeddah

4.

51

Counseling and Advice


H. Al Hajjar, A. Al Harthy, M Alatta & N. Dashash

In counseling stations usually patient comes with questions and inquiries, therefore
candidates are advised to give patient chance to ask questions, and it is also advisable when
you give some information to ask for feedback from the patient from time to time, and let
the simulated patient guide you during this station. Candidates who conduct completely
doctor-centered consultation (does not look to patients agenda) may perform badly in
counseling stations.
(1) Epilepsy Counseling
H. Al Hajjar

Topics:
1.
2.
3.
4.
5.
6.
7.
8.
9.
10.
11.
12.
13.
14.
15.
16.
17.
18.
19.
20.
21.

Ask about age, work, and marital status.


What do you know about epilepsy?
Explain what is epilepsy.
Prognosis
What to do during an attack?
Status epileptics
Medications
Avoid precipitating Factors
When to call your doctor.
Home environment
Work
School teachers &. Social contacts
Learning abilities & IQ. Epilepsy is not a mental disease
Activities & sport
Driving
Bracelet / necklace / card
Inheritance & genetics
Marriage
Follow up
Help groups / associations / foundations.
Leaflets & booklets

What to do during an attack?


Grand mal. Don't restrain convulsion movements
Don't place any thing in mouth
Don't cover with blankets. Don't move to other place.
Position on side during clinic phase, wipe away froth
From mouth to help airway. Take, away items that could cause injury.
Allow patient to rest post seizure.
Myoclonal: Remove objects which may cause injury
Absence: Don't try to alert patient (useless)

The Joint Program of Family and Community Medicine - Jeddah

52

Home environment:
Don't be in potentially dangerous state alone (kitchen, fire, bath drain, door locking).

Activities & sport


o Freely encouraged: Football, basketball cricket despite poor control
o No boxing
o Supervised: horse riding, cycling on busy roads, solo fishing or canoeing, climbing
o Swimming: is permissible if supervised on a one-to- one basis (inadvisable in
school groups)

Bracelet / necklace / card:


With details of medication & fits, GP & hospital phone number.

Medications:
o Don't stop suddenly by your self
o Don't miss doses (compliance). Don't or dose by your self.
o Side effects.
o Interaction with other medication

Avoid precipitating Factors:


Sleep deprivation, extreme hunger & fatigue, constipation, flicker (view screen from at
least 2 m in an illuminated room, if nearer cover one eye with palm of hand, Polaroid
sunglasses at seaside).

Status epileptics.
o Major attack does not stop as anticipated
o Duration > 20 minutes
o Recurrent attacks with no consciousness in between
o Call ambulance
o Keep rectal valium at home

Follow up:
o Shared with hospital
o Drug serum levels. Unnecessary when the patient is well controlled.

Driving:
According to local driving regulations in different countries. Ranges from one to ten years
free from seizure also according to type of driven vehicle.

Marriage inheritance & genetics:


You can live normal live and get married,
Inheritance to offspring: There is family history in 15 - 40 % of cases
?? Autosomal dominance mood of inheritance

When to call your doctor:


o If seizure change in number.
o Any time you change your medications or take another medications.

Pregnancy:
o Risk of fetal abnormalities with medications
o Uncontrolled seizures affect the fetus. Balance risk / benefits.

The Joint Program of Family and Community Medicine - Jeddah

53

(2) Counseling parents of a child with febrile convulsion


A. Al Harthy

o
o
o
o
o
o
o
o

Introduce yourself and establish doctor-patient relationship


Explore mothers ideas regarding breast feeding, concerns & expectations
Explain nature of the problem
Reassure about the benign nature.
When the child has fever do; reduce fever by: bath, tape sponges, paracetamole,
and light clothes.
If the child having convulsion:
Protect your child air way.
If you child has any thing in the mouth clear it with a finger to prevent choking
Place the child on the side or abdomen (face down) to help drain secretions.
During seizure, dont try to restrain your child or stop seizure movements.
Dont try to force anything into your child mouth.
In the way to the doctor keep fever down, Dress child lightly, continue sponging.
If your physician decides seizures can be treated at home follow instructions
Keep diazepam 5 mg for rectal use at home.
Prophylactic anticonvulsants can prevent convulsion from occurrence but it has
Side effects, pediatrician will decide when to use.
Fever is common after DPT vaccine, so start paracetamol in the physicians office
and continue it for at least24h.
Avoid covering your child with more than one blanket, it increase temperature 1-2
extra degree.
At the end ask the parents if they have any questions or clarifications
Thank the parents

The Joint Program of Family and Community Medicine - Jeddah

(3) Post-Myocardial Infarction Counseling


M. Alatta
Discuss with the patient the following issues:

o
o
o
o

o
o
o

o
o

o
o

o
o

o
o

o
o
o
o

o
o
o

The nature of coronary artery disease


Common post-infarction symptoms:
Niggling left-side chest pain
Ectopic beat
Light headedness
Fatigue
Risk Factors: weight, avoid stress, and stop smoking.
Exercise: Why its important to him?
What are the kinds suitable for him? E.g. walking, jogging.
Start slowly & gradually 3 times / week 10-30 minutes,
Warm up before exercising.
Diet:
Why it is important?
Types of diet and how to prepare?
Drugs:
Importance of compliance
E.g.: Aspirin, ACE and B-Blockers
Sex:
4 weeks after,
Position?
Driving:
4 weeks after MI
Short distances avoiding heavy traffic
Job counseling:
How strenuous is her/ his work?
Return to work should be post pond until after an exercise test
Patient can go back to work 4 - 12 weeks.
Or return after 4-6 weeks depending on patients recovery and demand of his job
Other issues:
Air traveling after 4-6 weeks, and preferably after review by the doctor
Relaxation therapy and stress management
Any other questions he 1 she would like to ask.

Annual Review:
Evaluate patients life style and coronary risk
Simple assessment for anxiety and depression
Review of drug treatment
Assess patients ability to continue with normal activity and work
Assessment of functional status, .g.: angina, breathlessness, presence of heart
failure
o Identify patients requiring referral for further investigations for possible
revasculization
o Consider ECG
o
o
o
o
o

54

The Joint Program of Family and Community Medicine - Jeddah

55

(4) Advice Mother About Breast Feeding


A. Al Harthy

Introduce yourself and establish doctor-patient relationship


Explore mothers ideas regarding breast feeding, concerns & expectations
Educate mother about importance of breast feeding
o Establish a psychological bonding between baby and mother
o Less cost
o Easier (no need to prepare)
o Increase immunity of the baby
o Decrease the chance of developing allergic diseases and bronchial
asthma
Current situation:
o Attempt of breast-feeding?
o Difficulties?
o Duration?
Social condition
o Facilities at work
Contraception
Technique of breast feeding
o Inform her that breast feeding should be initiated as soon after
delivery as possible
o The mother should sit comfortably with support for her back
(pillows can help), be in a private comfortable place if possible
o Make sure that the mother knows these technique:
- The baby should have a large part of the areola in his mouth
- During first 2 weeks, feed on demand
- Nurse baby 10 minutes on first breast and as long as he want on the
second breast
- Alternate which breast you start with each time
- Feeding less at night is OK; but no more than 5 hours should pass
between feeding
- Dont offer the baby any bottles during first 6 weeks
- Milk supply improve by adequate sleep, fluids, relaxed environment,
reduced stress
How the mother know if the baby is getting enough breast milk:
The baby is getting enough milk if
o The baby has a 6-8 wet diapers /day
o The baby is back up to birth weight by two weeks of age
o You can hear the baby swallowing while breast feeding
o The milk leaks from one breast when you are feeding the baby on
the other
o Baby has 3-4 bowel movements per day

Important points about breast feeding

Counseling a mother about breast-feeding should start early enough, during


prenatal care or even earlier.
The first 4 weeks after delivery is the most important time to establish breastfeeding.
By the start the baby is 3 month old, he need extra feeding and you need to start
introduce him to cereals (see weaning), but still you can continue breast feeding up

The Joint Program of Family and Community Medicine - Jeddah

to age 1-2 years.


For working mothers:
o Still you have enough and excellent time to feed your baby during
your maternity leave, even if you plan to stop after, but the best is to
continue
o By using break hours to feed the baby or
o By pump breast and keep milk in a refrigerator
Counseling the mother about contraception and if she choose the pills, she can use
either progesterone only pill or even combined pills provided she is well motivated
to breast feed.

(5) Advice Mother About Weaning


A. Al Harthy

56

Introduce yourself and establish doctor-patient relationship


Ask about history of current feeding,
Explore mother ideas regarding weaning: her concerns and expectations
Explain the continuous increase in dietary requirement by increase of age,
Explain how to introduce weaning food,
Begin with small amount, one kind of food at a time,
Weaning should be gradual,
Use a spoon and do not give solids in feeding bottle,
Give food before breast feed - gradually food replace feeding,
Problems during weaning,
Explain advantage of early start of cup feeding at 6 months using baby cup,
Home-prepared food is best. Explain how to prepare food at home.
When to begin:
o At 4-5 months: Diluted fruit juice and cereals
o At 5-6 months: Vegetables, soup, egg yolk & soft fruits
o At 7-8 months: Chicken, fish, bread and meet
o At 9-10 months: Continue previous food
o At 11-12 months: At 1 year of age baby can eat regular family diet provided it is
soft, he can swallow.
Continue breast feeding up to age 2 years,
Do not add honey or sugar,
Ensure mother understanding and acceptance of your instruction
Ask mothers if she has any question to clarify.

The Joint Program of Family and Community Medicine - Jeddah

57

(6) Counseling Mothers About Child with Gastroenteritis (GE)


F. Rayes

Introduce yourself and establish doctor-patient relationship


Explore patients ideas, believes, expectations and concerns about the diagnosis.
E.g. can vomiting and diarrhea be dangerous to the child?

Explain the diagnosis (what is gastroenteritis)


o What is dehydration?
o Causes
o Complications
o Prognosis
Advice:

Rehydration solutions:
o Use of ORS, Pedilyte, WHO Packet, Ricelyte
o Home made solution: (One letter water + one tea spoon salt + 8 tea spoon sugar
+ few drops of lemon juice)
o How do you know that ORS is working?
o Breast feeding should continue and extra fluid is given

Advice mother against clear liquids that commonly prescribed inappropriately


(high osmolality, high glucose, low sodium and low potassium)
o Orange juice
o Apple juice
o Coca-Cola and 7Up
o Tea

Educate mother about medications that should be used only for special indications:
o Anti-vomiting medicine: if vomiting is very severe and prevent oral
rehydration
o Anti-diarrhea medicine: not effective and may be harmful
o Antibiotic are not usually indicated

Educate mother about effective home remedial for dehydration?


o (BRAT) feeding in convalescence period (Banana, Rice, Cooked Apple, Toast).

Inform mother about serious symptoms which need hospitalization


o Severe intractable vomiting
o No urine
o Impaired consciousness

Insure patients understanding and acceptance of your advice.

Explain others management options e.g. IV therapy and/or investigations.

Reassurance
o GE is a self limiting
o If properly treated it rarely gives complications

Emphasize the importance hygienic measures to prevent diarrhea in future?


Calculation of fluid deficit:

For mild dehydration:


60 mL of oral rehydration solution per kg (60 mL x wt. in kg)
= mL of oral rehydration solution over 2-4 hours
For moderate dehydration:
80 mL of oral rehydration solution per kg (80 mL x wt. in kg)
= mL of oral rehydration solution over 2-4 hours
A toddler with severe diarrhea loses 100-200 ml of fluid with each stool and will
require half to one glass of oral rehydration fluid for each stool

The Joint Program of Family and Community Medicine - Jeddah

58

(7) Counseling Patient About Obesity


H. Al Hajjar, M.Alatta & N. Dashash

Introduce yourself and establish doctor-patient relationship


Encourage patients contribution:
Sympathetic approach
Discuss patient motives & barriers to change
Provide opportunity for feed back
Patient ideas, concerns & expectations: e.g.
I am not obese I am only over weight
Worried about becoming a diabetic
Family & Social history:
Smoking
DM, HPT& heart diseases, high cholesterol level
Obesity
Obtain enough information:
Previous trials
Heart disease risk factors
Role out secondary causes: hypothyroid, medications, cushings, heart failure
Past medical & drug history
Appropriate examination: to rule out secondary causes
BMI= Wt/ (Ht) 2 in meter
Diagnosis explained
Appropriate management:
General advise: food diary, lifestyle change, gradual
Behavioral: guidelines for healthy diet, avoid cues
Dietary: food groups, food exchanges
Physical activity: daily activity, exercise program
Follow up: 1 week
Written health education material
Share options with the patient
Other modalities according to BMI as medications and surgery
Referral to dietitian

Advice about Diet

o
o

o
o

o
o
o

o
o
o

o
o
o
o
o
o
o
o

o
o
o
o
o
o
o
o
o
o
o
o

Enjoy your food


Eat variety of food
Eat the right amount prescribed for you by your dietetion.
Do not eat too much fat
Do not eat while watching TV
Eat in small plates
Do not shop while you are hungry
Always go shopping with as hoping list
Eat 5-6 times a day (3 meals small healthy snacks
Do not eat sugary food too often eat only what is calculated to you by your dietetion
Look after vitamins &minerals in your food
Ask for support or advice from you doctor if you have any inquiries.

The Joint Program of Family and Community Medicine - Jeddah

59

Body Mass Index (BMI) & Obesity


o BMI used to reflect the presence of excess adipose tissue.
o Calculated by dividing measured body weight in kilograms by the height in meters
squared (BMI= Weight in Kg height in m2)
o Normal BMI is 20-25 kg/ m2.
The National Institutes of Health currently define obesity as: a relative weight over 120%
(BMI > 27.5 kg/ m2).

Classification of Obesity:
o Mild obesity is a relative weight of 120-140 % (BMI 27.5-30 kg/ m2).
o Moderate obesity is a relative weight of 140-200 % (BMI 30-40 kg/ m2).
o Severe or morbid obesity is a relative weight over 200% (BMI > 40 kg/ m2).
o Obese patients with high waist-hip ratios (>1.0 in men; > 0.8 in women) greater
risk of diabetes mellitus, stroke, coronary artery disease, and early death.

Health Consequences of Obesity:


o Hypertension
o Type II diabetes mellitus
o Hyperlipidemia
o Coronary artery disease
o Degenerative joint disease
o Psychosocial disability
o Certain cancers (colon, rectum, and prostate in men; uterus, biliary tract, breast, and
ovary in women)
o Thromboembolic disorders
o Digestive tract diseases (gallstones, reflux esophagitis)
o Skin disorders are also more prevalent in the obese
o Surgical and obstetric risks are greater as well
o Greater risk of pulmonary functional impairment
o Endocrine abnormalities
o Proteinuria
o Increased hemoglobin concentration
o Death rate increases in proportion to the degree of obesity.

The Joint Program of Family and Community Medicine - Jeddah

60

(8) Counseling Patient About Smoking Cessation


H. Al Hajjar & M.Alatta

o
o
o

o
o
o
o

o
o
o

o
o
o
o
o

o
o
o
o

o
o

o
o

Introduce yourself and establish doctor-patient relationship


Assess smoking habits:
Smoking & quitting history (causes of failure).
Interest in cessation.
Potential motivating factors.
Motivate the smoker to attempt to quit:
Emphasize benefits of cessation.
Disadvantages of smoking: medical, social (children, pregnant wife) religious, risk
of fire.
Focus on short term changes.
Tailor to the clinical situation. e.g : asymptotic patient, or patient with acute
respiratory illness, pregnancy or chronic disease (DM, Hypertension, Myocardial
infarction, and/or COPD)..
Explore patients ideas, concerns and expectations
Ask for a commitment to quit (set a quitting date within tow months)
Discuss methods to help the smoker to quit:
Behavioral
Smoking diary.
Positive reinforcement.
Progressive restriction (during preparation)
Find alternatives to oral and hand activity.
Pick a date to stop smoking and tell your family and doctor
Avoid smoking cues.
Remove all ashtrays from you surrounding environment.
Develop social support.
Tell you patient that he must be firm about refusing cigarettes from others.
Avoid situation that will tempt you to smoke (friends, parties).
Tell your family &friends that they should respect your wish of them not smoking
around you.
Find and join a support group.
Pray and ask god to help you.
Self help material.
Learn to do some fun thing that will distract your craving (which will be short
term).
Pharmacological
Nicotine replacement therapy (gum & patch)
Effectivenessand. Side-effects.
Other methods
Smoking cessation programs.
Acupunctures, hypnosis.
Anticipate problems:
Withdrawal symptoms.
Weight. Gain.
Follow up
Put smoking on problem list.
Follow up visits. / Continued monitoring at each visit.

The Joint Program of Family and Community Medicine - Jeddah

(9) Counseling Patient About HRT


A. Al Harty

o
o

o
o
o

o
o

o
o

Introduce yourself and establish doctor-patient relationship


Explore what the patient knows about HRT?
Does the patient have any complaint?
Explore menstrual history
Presence of any complaint:
Hot flushes, Sleep disturbances, Sexual problems, Body ache, others
Diet history (Calcium intake)
Daily activity & exercise
Medical illness, DM, hypertension, Heat disease, DVT, Breast disease, Cancer
Smoking
Family history of Cancer breast
Effect of problem on patient life
Her ideas, concerns, expectations
Discuss advantages and disadvantages of HRT
Possible regime of HRT, for How long will use?
Importance of:
Regular follow-up during HRT
Doing proper investigation
Report of irregular bleeding, leg swelling
Discuss non pharmacological measures for prevention of osteoporosis:
Enough intake of calcium, exercise, other alternative drugs.
Consider the patient willing to undertake risk in exchange with benefits
Examine: Wt, Bp, Breast, abdomen, pelvic exam.
Investigation: Pap smear, mammogram, + lipid profile.
Management:
Give her time to explore any idea or questions
She can take enough time to decide if not today
Refer her to health educators
Prescribe HRT
Give appointment for follow up

Patient Education about Menopause

What is the patient knowledge about menopause?


What is menopause?
Its symptoms (Not necessary all women will have symptoms).
Menstrual irregularity then cessation of menstruation.
Need of contraception in per menopausal stage.
Clarify importance of positive thinking about this stage of life.
Possible occurrence of annoying symptoms & Its management:
Patient ideas, concerns, expectations.
Risk factor for osteoporosis and how to deal with it?
Importance of non pharmacological measures for prevention of osteoporosis:
o Diet (enough intake of calories)
o Exercise (physical activity & sun exposure)

HRT: Its advantage, disadvantage, and possible regimen for use.

61

The Joint Program of Family and Community Medicine - Jeddah

(10) Hepatitis B Chronic Carrier


H. Al Hajjar & M. Alatta

o
o
o
o
o
o
o
o

o
o
o

o
o
o
o
o

o
o
o
o
o
o

o
o
o

o
o
o

Introduce yourself and establish doctor-patient relationship


Skills needed in breaking bad news
Prepare patient that bad news is coming
Show sympathy
Lay out news simply & honestly
Maintain eye contact, proper body language
Discuss results of tests one at a time
Acknowledge your short coming & emotional difficulty
Encourage patient to express feelings
Offer help to tell family, employer
Obtain enough information
Symptoms of liver failure, complications
Past Medical History: e.g. jaundice, contact, blood transfusion
Social history: e.g. smoker, married & how many children, what occupation?
Exploration of patient Ideas, Concern & Expectations
Examples:
How did I get it?
Will this affect my family?
What about my work?
Will I die from if?
What should I eat?
Appropriate counseling
Assess patient understanding
What does chronic HB means? (e.g. drawing of virus)
Mode of transmission: Blood, sexually, body secretions
Symptoms: asymptomatic or malaiseand abdominal discomfort
Prognosis: chronic active hepatitis, cirrhosis, hepatocellular carcinoma or other viral
hepatitis
Treatment :Interferon, cytotoxic medications or surgery (indication & response)
Family protection:
Active & passive immunization
Sexual intercourse & contraception
Use of tooth brush, razor
Respond to patient questions
Health education materials, where to get more information
Appropriate follow up:
Need to refer to specialist for US, liver biopsy, treatment
Shared follow up with specialist, ensure continuity of care
Regular monitoring by LFT, liver enzymes, -fetoprotein

62

The Joint Program of Family and Community Medicine - Jeddah

Guide to Post exposure Immunoprophylaxis for Hepatitis:

Accidental
Percutaneous or permucosal
Household contact
Chronic carrier
Household contact
Acute case with identifiable
Blood exposure
Prenatal
Sexual acute infection
Sexual, chronic carrier

Vaccination + HBIG
Vaccination
Vaccination + HBIG

Vaccination + HBIG
Vaccination + HBIG
Vaccination

63

The Joint Program of Family and Community Medicine - Jeddah

64

(11) Counseling Patient With Insomnia


N. Dashash & F. Rayes

o
o
o
o
o

o
o
o
o

Introduce yourself and establish doctor-patient relationship


Explain the nature of insomnia and reassure.
Identify the cause of insomnia and discuss the management options:
1. Chronic sleep problem: advice patient on sleep hygiene and avoid prescription
2. Sleep difficulties as part of physical or psychological illness: treat the cause (e.g. if
the patient has depression explain the specific treatment)
3. Acute or situational sleep difficulties: consider hypnotic prescription and advice on
sleep hygiene
Prescription of hypnotic Drugs:
- Discuss side effects of drugs.
- Used for a short period of time.
- Should be used only if patient fails to respond to the initial measures.
Sleep Hygiene:
Establish regular bedtime and regular waking time.
Avoid all naps.
Regular exercise but not at night.
Use bed only when ready to sleep and leave bed if sleep is not forthcoming.
Avoid all of caffinated drinks/food, stimulants, cigarettes and alcohol.
Healthy environment in the bedroom:
Dark or weak lighting
Quiet no TV
No books
Comfortable bed ... etc.
Health promotion according to age.
Keep a sleep diary for next visit.
Set up a near appointment.
The help of a psychiatrist can be obtained.
Ask if he has any question?
Offer the patient an educational leaflet.
Offer the patient a referral to a health educator, psychologist and/or psychiatrist if
he wishes.
Thank him for cooperation.

Causes of Insomnia
Psychiatric disorder
Drug & Alcohol abuse
Medical /Surgical problem
Primary sleep disorder

50 %
10-15 %
10 %
10-20 %

The Joint Program of Family and Community Medicine - Jeddah

65

General Therapeutic Recommendations in Insomnia


Depression
Sedative Antidepressant
+ BZDs for the first 10-14 days, then PRN.
Anxiety

Long acting BZDs 2ws, if not responding --> Refere.

Pain

Treat underlying medical problem aggressively


+ Short course BZDs to establish a normal sleep.
Personality Disoder : Refere

Facts About Insomnia and Benzodiazepine Medications (BZDs)


Adiction to BZDs develop after 1-3 months of regular use.
Only 30-45% experienced true withdrawal symptoms.
BZDs are very effective drugs when use appropriatly for short term and low dosage.
No value in long term (>4ws) prescribing of BZDs.
Antidepresant/ Psychotherapyand/or behavioural therapy are more effective than long term
BZDs.
Patient on long term BZDs and review the diagnosis.
Elderly patient happy with small dose BZDs and dont be dogmatic with him.

Prescribing Of BZDs
Short acting
Insomnia
Long acting

Day time anxity

Lorazepam 1-2 mg.


Oxazepam 15-30 mg.
Diazepam 5-10 mg.

The Joint Program of Family and Community Medicine - Jeddah

66

(12) Family Planning Counseling


F. Rayes

Introduce yourself and establish doctor-patient relationship


Explore patients ideas concern and expectation about family planning
Explore factors that influence the choice of method:
o Age and parity
o Frequency of intercourse
o Lactation
o Degree of desirable effectiveness
o Any contraindication to contraception
o Personal preferences & acceptability
o Concern and previous experience
o Risk of STD or PID
o Compliance
o Marital status
o Cost and ethical consideration

Explain to the patient the information needed to help her to make informed choice
of appropriate method:
o Effectiveness of each method.
o Risk & benefits of the various methods (Advantages & Disadvantages).
o The best methods for specific case.
o Instructions for their use.
o The follow-up policy for various methods.

With the patient chose contraceptive method suggested by her type:


Patient factors Contraceptive method
Very young, newly married - Oral contraceptive (mid-strength)
Smoker >35 years of age - Condom, Depo-Provera, diaphragm
Diabetic - Barrier form or Norplant
Lactating - Norplant, Depo-Provera
PID - Barrier form or oral contraceptive
Hypertensive - Barrier form,or IUD if multiparous
Toxic shock syndrome - Condom, oral contraceptive or Norplant
Permanent contraception - Vasectomy or tubal ligation
Long-term reversible method - Norplant or IUD (in monogamous women), Oral
contraceptive
HIV risk - Condom plus any other form of contraception

Discuss and arrange for follow up according to the following schedule:


Contraceptive method Follow-up
Combined oral contraceptive, Three months after initiation, then yearly
Progestin-only pill Three months after initiation; call physician
when discontinuing breast feeding, then yearly
Sponge or Diaphragm Yearly ; some advocate three-week follow-up visit after
first fitting
IUD Yearly for removal and reinsertion
Progestasert Yearly for pap smear
Norplant Nurse visit every three months; physician visit yearly
Depo-Provera Three-weak follow-up visit to check fitting, then Yearly
for pap smear
Cervical cap Three months for Pap smear, then yearly

The Joint Program of Family and Community Medicine - Jeddah

67

Topics for discussion if combined oral contraception is chosen:


o For the first time start at day one of the period
o Then every month stop the pill for 7 days (to allow menstruation)
o If you miss a pill:
- If you missed only one day, take the missed pill and you dont need to do
any extra contraception precaution
- If you missed more than one pill, you dont need to take the missed pill, but
extra contraception precaution is needed for at least one week
o Report to your doctor if you have any warning signs:
E.g.: Severs headache, visual disturbance, severe abdominal or chest pain, leg
pain or leg swelling
o Mention that you are on the pill if you need any medication or hospitalization

(13) Emergency Pill Education


N. Dashash

o
o

o
o
o

o
o
o
o

o
o
o

o
o
o
o

Introduce yourself and establish doctor-patient relationship


Ask about her age and LMP.
When was her unprotected intercourse?
Explain that emergency contraception is riot effective if
>72 hours for combined oral contraceptive pill
>5 days for IUCD
The pill:
Better to give an anti-emetic with the pill
How? A high estrogen (50 g) compind pill e.g. ovran
(2 tablets at once, then 2 tablets after 12 hours).
OR: (50 g estrogen alone BID for 5 days)
N.B. If vomiting started within 2-3 hours, repeat dose with an antiemetic
Exclude absolute contraindications:
Thromboembolic disorders e.g. DVT, Pulmonary embolism, CVA
Active liver disease markedly impaired liver function.
Known or suspected estrogen dependant neoplasm (e.g. breast or endometrial cancer)
Undiagnosed vaginal bleeding.
Ask about relative contraindications
Migraine, epilepsy
DM, HTN, familial hyperlipidemia
Smoking
IUCD: (a more effective alternative)
Exclude contraindications:
Active PID, undiagnosed vaginal bleeding, previous ectopic pregnancy, Valvular heart
disease
Her next period may be early or late.
Should use a barrier method until the next period
She should attend to the physician if she complains of any of the following:
Lower abdominal pain
Heavy vaginal bleeding
Changes in period (heavy, little or missed)
If she is concerned or needs any clarification or help
The failure rate ranges from 2 to 5 %.
She needs to have an arrangement for future contraception.
Ask if she has any questions.

The Joint Program of Family and Community Medicine - Jeddah

68

(14) Travelers Advice


F. Rayes & M. Alatta

Introduce yourself and establish doctor-patient relationship


Information about travel
o When & for how long?
o Where & which area in the country?
o Why is he traveling?

History (medical illnesses, drug history, allergic history, Vaccination history)

Medico-legal certificates

Invite patient to present his complain and/or ask questions

Take feedback frequently and insure that you are answering all his queries
Important information for travelers:

Information about motion sickness:


o No eating fried or fatty food before departure.
o Set between the wings.
o No reading, close your eye, no tight clothes.
o Antihistamine e.g. (Phenergan) 30-60 min before departure.

Endemic diseases in the area the patient is traveling to


E.g. patient traveling to Africa he need to have some information about the following
diseases: Typhoid, Yellow fever, Hepatitis, Malaria, AIDS

Patient needs to know if there is any Specific Prophylaxis

General measures against GIT infections:


o Unless the traveler is sure of the purity of the local water, he should not drink it without
boiling first. This also applies to water for ice cubes and for cleaning teeth.
o Unpasteurised milk should be boiled before use, and care should be taken with local
cheeses and ice cream, which are often made from unpasteurised milk.
o Advise the traveler to eat only cooked vegetables and avoid salads.
o To peel all fruit, including tomatoes.

General measures against mosquito bites:


o To take measures to avoid mosquito bites, especially after sunset.
o If you are out at night wear long-sleeved clothing and long trousers.
o If sleeping in an unscreened room, or out of doors, a mosquito net (which may be
impregnated with insecticide
o Insect repellents that contain deet work the best.
o Wear permethrin-coated clothing and use bed nets while you sleep.)
o Net is a sensible precaution and portable. Lightweight nets are available.

General advices
o Avoid swimming in lakes or rivers
o Use safe traffic and transportations
o Use recognized air lines
o Carry enough of your regular medicines in their original containers along with extra
prescriptions for them.
o Wear a medical bracelet if needed.

How to find a doctor


In under developed countries university hospital is reasonably the best choice to consult if
the traveler develop any health problem

The Travelers Medical Kit


Advice patient to take the essential first aid treatment and the prophylactic drugs, e.g.
Chloroquine, Bactrim, Paracetamol, Oral rehydration, Insulin & Syringe, Phenergan.

The Joint Program of Family and Community Medicine - Jeddah

69

Travelers diarrhea:

Prophylaxis:
o Bactrim double strength tab once/day from the day of arrival.
o Or bismuth subsalicylate 2 tablets or 2 oz of liquid 4 times a day while traveling

Treatment regimens
o Ciprofloxacin 500mg orally 2 times daily for 1 to 3 days. or
o Ofloxacin 400mg orally 2 times daily for 1 to 3 days. or
o Single-does ofloxacin plus loperamide 400mg or
o Ofloxacin and Loperamide (Imodium) 2 mg tab. 2 tablet stat then one tablet after each
losse stool.
Or Diphenoxylate with atropin (Lomotil) 2.5 mg - 2 tab. QID

Specific prophylaxis for travelers:

Typhoid: First dose Now 28 day later booster dose.

Yellow fever: 5 days later from specialist center

Hepatitis A vaccine: May be needed + Precaution.

Malaria: Chloroquine one tab. once a week. 2 ws prior to entering, 4-6 ws after
leaving.

AIDS & Other STD: Avoidance of risky behavior completely.

The Joint Program of Family and Community Medicine - Jeddah

5.

70

Approach to Simulated Clinic Exam


F. Rayes

The main objective of the simulated clinic is to evaluate the candidates skills in
consultation. Accordingly, in preparation for simulated clinic exam, the candidate needs to
improve his/her knowledge and skills in consultation.
Some Important Consultation Models:
Byrne & Long (1976)

Doctor-centered consultation: the doctor was more likely to make decision for the
patient and instruct him to seek some service.

Patient-centered consultation: the doctor was more likely to seek the patients
views and permit him to make his own decision concerning the outcome.

Failure to explore the real reason of patient problem is the main reason of
consultation failure
Patient-Centered
Consultation
Use of patients
Knowledge and experience

Silence
Facilitation

Doctor-Centered
Consultation
Use of doctors
Special knowledge and experience

Clarification
Analyzing
Gathering
Interpretation
Probing Information
Skills used by physician in patient-centered against
Doctor-centered consultation

Scott and Davis (1979) The Expanded Model of Consultation:

Management of Presenting Problem

Management of Continuous Problem

Modification of Help Seeking Behavior

Opportunistic health Promotion


Pendleton 7 Tasks (1982):
1. To define the real reasons for patient attendance;
2. To consider other problems;
3. To choose appropriate action for each problem with the patient;
4. To achieve a share understanding;
5. To involve patient in the management;
6. To use time and resources effectively;
7. To establish and maintain doctor-patient relationship
Neighbour (1992), The Inner Consultation:

Connecting (establishing relationship)

Summarizing (physical, social & psychological diagnosis)

Handing over (management of presenting problem)

Safety netting (Anticipatory care)

The Joint Program of Family and Community Medicine - Jeddah

71

The main skills assessed during simulated clinic exam


Interview and history taking:
1. Introduces self to patients
2. Encourage patients to elaborate presenting problems fully
3. Identifies patients reasons for consultation
4. Listens attentively, Puts patients at ease
5. Recognizes patients verbal and non-verbal cues
6. Uses silence appropriately
7. Phrases questions simply and clearly
8. Considers physical, social and psychological factors as appropriate
9. Seeks clarification of words used by patients as appropriate
10. Elicits relevant and specific information from patients and/or their
records to help distinguish between working diagnoses
11. Exhibits well-organized approach to information gathering
Behavior and relationship with patients:
1. Conveys sensitivity to the needs of patients
2. Demonstrates an awareness that the patients attitude to the doctor (and vice versa)
affects management and achievement of levels of cooperation and compliance
3. Maintains friendly but professional relationship with patients
with due regard to the ethics of medical practice
4. Considers ethical issues in his practice, particularly patient confidentiality, and is
able to offer reasons for his action
Physical Examination:
1. Uses the instruments commonly used in general practice in selective, competent and
sensitively manner
2. Performs examination and elicits physical signs correctly and sensitively
Patient Management:
1. Formulates management plans appropriate to findings and circumstances in
collaboration with patients
2. Checks patients level of understanding
3. Makes discriminating use of investigations, referral and drug therapy
4. Arranges appropriate follow up
5. Demonstrates understanding of the importance of reassurance and
explanation and uses clear and understandable language
6. Is prepared to use time appropriately
7. Attempts to modify help-seeking behavior of patients as appropriate
Problem Solving:
1. Correctly interprets and applies information obtained from patient records, history,
physical examination and investigations
2. Generates appropriate working diagnoses or identifies problem(s) depending on
circumstances
3. Is capable of recognizing limits of personal competence
4. Seeks relevant and discriminating physical signs to help confirm or refute working
diagnoses
5. Is capable of applying knowledge of basic, behavioral and clinical sciences to the
identification, management and solution of patients problems
Anticipatory care:
1. Acts on appropriate opportunities for health promotion and disease prevention
2. Provides sufficient explanation to patients for preventive initiatives taken
3. Sensitively attempts to enlist the cooperation of patients to promote change to
healthier lifestyles

The Joint Program of Family and Community Medicine - Jeddah

72

Possible difficulties and pitfalls in simulated clinic exam:


1) Common difficulties in communications:

Patient with hidden agenda: e.g. patient requesting vitamin or cough syrup or
patient showing certain non verbal cues
Aggressive and demanding patients e.g. patient may till you: give me this
medication now! or he may say: Your colleague Dr. X is very rude
Passive aggressive patient: e.g. patient may say: yes, but!
Poor compliant patient: e.g. patient refusing your medication or investigation or
advice

Common pitfalls:

Use of open-ended question at the start only


Talking continuously and not listening
Forgetting to explore patients health beliefs
Being very anxious and couldnt express any empathy
Being reactive and getting angry
Losing control.

2) Common difficulties in information gathering:

Atypical presentation of common disease: E.g. MI presenting as epigastric pain.


Indirect presentation: E.g. depressed patient present with backache.
Many problems at a time E.g. DM + infections + social problems, and difficulty in
prioritization
Multiple somatic complain E.g. somatization, masked depression or anxiety
Possible serious diagnosis: E.g. elderly patient with palpitation.

Common pitfalls:

Reaching final diagnosis from the first impression and ignorance to ask specific
questions to prove this diagnosis objectively
Disorganization and non-directive interview
No clear objectives
Failure to make use of preliminary information from the patient file
Repeating same questions in the same way
Wasting long time sticking to one issue
Ignorance of patient cues
Doctor-centered consultation
Thinking of one and only one possible diagnosis
Forgetting to ask about patient health beliefs
Forgetting to ask specific questions to rule out the possible differential diagnoses
Ignorance to ask specific questions for risk assessment and continues problem
No summarization of the history and no feedback from the patient.
Forgetting to conduct physical examination
Wasting long time in discussing irrelevant physical examination

The Joint Program of Family and Community Medicine - Jeddah

73

3) Common difficulties in management:

Controversial management issue


Complicated social problem
Complicated diagnosis
Uncertain diagnosis
Risk of complication e.g. ethical dilemma, marital problem, demented patient with
no family support,?? MI. !?? Ca.
Unhealthy life style, e.g. smoker or obese patient needing health education
Risk of complication, e.g. severely depressed patient at risk of suicide

Common Pitfalls:

Forgetting to discuss different management options


Forgetting to make use of other primary health care team members
Forgetting your limitation and to make good use of referral system

Helpful strategies in dealing with difficulties in simulated clinic exam:


1) Read the preliminary information carefully:

Concentrate on the key words, e.g.:


o Infrequent attender or
o DM+ high fasting blood sugar (FBS) or
o Medical student, Follow-up visit, Significant past historyetc

Speculate possible objectives from the given scenario, and at the same time be open
minded and ready to conceder patients objectives
2)

o
o

o
o
o

Have systematic approach to your objectives:


Full focused history
Listen and watch carefully for any verbal or nonverbal cues
Use hypothetical deductive reasoning methods to test your hypotheses
Think loudly to give the examiner the chance to understand how you think, and
give you the desirable evaluation mark
Concentrate on your provisional hypothesis by asking relevant and specific
questions to reach clear and positive diagnosis
Remember: Psychological diagnosis by positive criteria not by exclusion
Eliminate possible deferential hypotheses by asking relevant and specific questions
Use open-ended questions when ever possible
Complete your exploration by asking specific questions
Assess the degree or risk (look for red flags) e.g.:
Suicidal risk factors in depressed patient or
Risk factors in hypertensive patient
Explore continuous problems e.g.:
Chronic illness
Continuous medications
Smoking, obesityetc.

The Joint Program of Family and Community Medicine - Jeddah

3)

Remember the basic skills to obtain information and try to avoid habits which
block communication:
Basic skills to obtain information

General Attitude:
Respect
Empathy.
Touch (if appropriate)
Eye contact.
Body language
Social smile.
Encouraging.

Questioning:
Open-ended questions
Facilitating verbal & non verbal
Reflecting questions.

74

Active listening:
Restatement
Classification and summarizing
Taking feedback
Empathy
Non-verbal awareness
Use of more advanced skills to
push for Resistant information:
Confrontation and probing
Reflection
Use of silence and use of touch
Thinking loudly and acknowledge
uncertainty
Asking for more clarification
Interpretations of...
o Non-verbal communication.
o Paralanguage
o Body language

Habits which block communication

General Attitude:
Patronizing
Tenseness and nervousness
Coldness and unfriendliness
Defensiveness
Appearance of too relax or casual
Appear preoccupied

Questioning:
Direct questions,
Why question,
Suggestive question,
Yes or No questions.
Many questions at a time.

Specific Behavior:
Use of Jargon
Inability to keep quiet
Unawareness of non-verbal cues.
Interrupting the patient
Controlling & inhibition of the
patient.
Lack of purposeful direction in the
interview.
Making assumption.
Giving advice too early.
Allowing personal emotions to get
in the way.
Talking too much continuously.
Inability to take feed back.

4)

Improve your explanation skills:

Ask the patient about what he already knows


Invite patient to ask questions
Continuously ask for feedback to make sure that you and the patient have a shared
understanding of the problem
Use simple language
Use varities of methods, e.g. demonstration or written materials

The Joint Program of Family and Community Medicine - Jeddah

5)

Improve your negotiation skills:

Establish and maintain adult to adult relationship


Show good listening
Show empathy and care
Do logical analysis of the problem
Offer alternative solutions
Deviate the conversation to other issues; examples:
Take more history
Discuss psychosocial component of the problem
Perform physical examination
Give health education
Discuss health promotion issues
Be flexible and respect of patient autonomy
If patient is insisting make a contract of limited agreement

6)

Remember the basic skills for reassurance:

Adult to adult relationship (Respect and honesty)


Appropriate exploration of patients problem:
Physical, social and psychological component of the problem
Exploration of patient health beliefs about the problem
Examination:
Appropriate
May be over doing some extra examination to show how much you care.
Clear and objective explanation:
Summarizing the problem
Naming the diagnosis
Prevalence of the problem (how common is this problem)
Natural history (how rare are the complications)
Management options (how they are safe and acceptable)
Prognosis (how benign, treatable or at least controllable)
Taking feed back:
The patient understands the explanation
The patient accepts the explanation
Assurance of accessibility

o
o
o
o
o

o
o

o
o

o
o
o
o
o
o

o
o

75

The Joint Program of Family and Community Medicine - Jeddah

7)

8)

Remember the comprehensive and holistic style of management in family


medicine:
Shared understanding
Comprehensive diagnosis (Physical, social & psychological)
Reassurance and explanation may be the only treatment
Appropriate use of nonpharmachological treatment
Appropriate prescribing: right drug and right dosage & right frequency
Explanation of effects and precautions of the medication
Modification of help seeking behavior
Awareness of limit of personal competence
Appropriate use of resources
Health promotion
Disease prevention
Appropriate follow-up arrangement

How to break bad news


(Dr. Hana Al Hajjar)

The setting:

Tell the patient when you are certain

No interruption

Comfortable physical setting

Family support
The patient:

Right to know

How much patient knows?

How much patient wants to know?

Encourage feelings expression

Listen to patient concerns

Beliefs & social background


The telling:

Warning shot, simple & honest

Eye contact, body language

Sympathy, encouragement, reassurance

Explain (diagnosis, prevalence, treatment and prognosis)

Reinforce & clarify frequently

Acknowledge your difficulties in breaking the news


Follow up:

See next day

Offer help to tell family & employers

Support groups

Documentation

76

The Joint Program of Family and Community Medicine - Jeddah

77

9) Strategies for dealing with some difficult patients:

Rambling, circumstantial patient: directed interview; closed questions;


permission at outset for frequent interruptions; frequent summarizations.

Threatening, aggressive patient: deflect anger; ally oneself with patient and
alliance position if seated; does not hem patient in; calm voice; reflect feeling of anger.

Violent, berserk patient: prevention: re-channel anger before it becomes


explosive; call for help, plenty of manpower police if necessary; a show of force can be
reassuring to a person terrified of his own lack of control; not too close do not violate
patients territory; interviewer closer to exit than patient; calm, comforting voice; sedative
chemicals, seclusion room, restraints may be needed.

Malingerer: confrontation usually ineffective; diagnosis by inconsistencies in


history and examination.

Seductive patient: deal with issue underlying seductiveness; what does patient
really want; be aware; doctors fantasy or needs for omnipotence.

Mute non-comatose patient: non-verbal communication is necessary (hold


hands); do not talk about mute patient in his or her presence; patient sometimes can
respond by nods or eyelid movements to closed questions.

Psychotic or thought disordered patient: closed questions; directed interview;


simple short sentences; concrete rather than abstract questions; avoid colluding with
patients about delusions or hallucinations (neither deny nor agree, if possible).

Organic brain impairment: as for (g); talk more slowly; give patient plenty of
time to respond.

Migrant: use interpreter; look at patient not at interpreter when talking; do not talk
loudly.

Elderly: if necessary ensure hearing aid or spectacles are available; talk more
slowly wait for replies; allow more time; sit face to face with patient; do not talk loudly; do
not patronize; touch can be reassuring.

Children: stay at some level as child with language and physically do not sit at a
higher level; distraction or mutual task while talking can be helpful.

Doctor as patient /the very important patient (VIP): danger of interviewer not
asking certain questions or assuming the VIP will volunteer essential information; danger
of having strong, positive or negative feelings often unconsciously towards to VIP; danger
of managing VIP differently.

Own family: conscious and unconscious biases preclude the interviewer properly
assessing family members as patients.
Reference : Ken Cox, Christine E. Ewan. The Medical Teacher. Churchill Livingstone;
London 1988.

The Joint Program of Family and Community Medicine - Jeddah

78

10) Organization and time management in simulated clinic exam:


For organization and effective time management in simulated clinic exam, remember the
three stages of the consultation and the tasks you need to fulfill in each stage, and in each
consultation and according to the priorities distribute your time.
Take enough time in stage one (building good relationship), and do not forget to save
enough time for stage three (finishing the interview).
See the table below:
Stages of the consultation and your main tasks in each stage:
Stages

Your Main Tasks

Stage I: Starting the interview

Building effective relationship with the patient

Stage II: Hypothesis formation

Prioritizing between patients problems


Reaching a provisional diagnosis
Excluding the differential diagnoses

Stage III: Finishing the Interview

Identifying factors that affect management and


prognosis
Explaining management options
Closing the encounter

During training identify your difficulties and work on them specifically,


and if possible ask your trainer to help you to over come your difficulties

The Joint Program of Family and Community Medicine - Jeddah

6.

79

Examples of Simulated Clinics


F. Rayes, N. Dashash, H. Hajjar, M Alatta, A. Assaggaf & A. Al Harthy

The following are examples of common simulated patients presentation in exam and the
possible approach to them in the form of checklists. However, candidate should not follow
these checklists strictly, he/she need to be flexible, and always conducts patient-centered
consultation, starting the consultation by exploration of simulated patients ideas, concerns
and expectations, he also should be sensitive to any verbal or nonverbal cues and respond
to them appropriately and immediately.

(1) Approach to Patient with Chest Pain


Dr. Fayza Rayes
Causes include musculoskeletal, gastrointestinal, neurological, functional, cardiac and
pulmonary.
The following items may need to be considered.

Establish good rapport


Encourage patient contribution
Respond to patients cues
Look for recent precipitating event
History of pain: onset, duration and radiation of pain
Characteristics of pain
Aaaociated symptoms: e.g.
o Cough
o Breathlessness or sweating
o Gastrointestinal symptoms
o Palpitations or anxiety

Social and psychological context of the problem

Precipitating factors, e.g. fears or exertion

Relieving factors: rest, medications

Smoking habit
Examination:

Pulse, blood pressure

Cardiovascular system

Chest

Chest wall

Abdomen
Management:

Share diagnosis and share prognosis

Agree management: behavior, drugs or referral

Reassurance and follow-up arrangement if necessary

The Joint Program of Family and Community Medicine - Jeddah

Management of acute MI

Rapid history and physical examination

IV access

Administration of oxygen

Cardiac monitor:

ECG

Blood studies

Aspirin, 1 tablet crushed & swallowed

Morphine sulfate, 2-4mg IV every 15-20 min.

Transfer to hospital.

Indication to Thrombolytic Therapy


o Within 12 hrs. onset of chest pain lasting for at least 30 min.
o ECG changes of ST elevation at least 1 mm in two, Or more contiguous leads of
left bundle branch block.

Contraindications to thrombolytic therapy


o A history of active GIT bleeding within 2 months.
o Uncontrolled hypertension.
o CVA having occured within the last 6 m.
o Recent history of serious injury within 1month.
Non-compressible vascular puncture
o

See Data interpretation: ECG for more details

80

The Joint Program of Family and Community Medicine - Jeddah

(2) Approach to Patient with Cough.


A. Assagga , A. Al Harthy & F. Rayes
Causes include infection (URTI or pneumonia), inflammation (including smoking),
asthma, cardiac failure, chronic chest disease, foreign body, and malignancy.
The following items may need to be considered:

Establish good rapport


Encourage patient contribution
Respond to patients cues
Duration of complaint
Predisposing factors (night-time, exercise)
Clarification of the symptom:
o Is the cough tickle in the throat or from the chest, its onset and course
(Continuous or intermittent, at daytime or at night).

Associated symptoms:
o Wheezing,
o Chest pain
o Shortness of breath, or orthopnea
o Fever, night sweating, weight loss
o Heamoptysis.

Presence of sputum: From throat or chest, quantity, color, relation to position.

Past history of similar problem or T.B.

Family history of T.B. or bronchial asthma.

Continuous problems & at risk factors: bronchial asthma, DM or heart disease

Social history & occupation.

Allergy history

Drug history

Smoking habit

Therapies already tried

Social & psychological context of the problem


Examination:

Examination of respiratory system

Examination of cardiovascular system

Peak flow, before and after Beta agonist

(If the patient is a child exam his throat and ears)


Possible investigations:

Chest X-ray

Sputum culture

Specific investigations according to the differential hypotheses, e.g. TB skin test


Management:

Share diagnosis and share prognosis

Advise against smoking

Use of medication:
Cough suppressant or expectorants, antibiotics, brochodilators or steroids.

Agree referral if indicated

Arrange follow-up if indicated

81

The Joint Program of Family and Community Medicine - Jeddah

82

Important diagnoses not to be missed in a childe presenting with cough:


Differential features of Epiglottitis, Croup & Bronchiolitis

Epiglottitis
3 7 years
Sudden onset, fulminating
Dysphagia, drooling
Fever
Respiratory stridor

Croup
Childhood
URTI problem 1-7 days
No drooling
Low grade fever or
moderate
Biphasic stredor

Muffled voice / cry


Minimal cough
Toxic appearance
H. influenza
Emergency protocol
Avoid exam the pharynx

Hoarseness
Barking spasmodic cough
Nontoxic
Para-influenza 1
Humidification (crouptent)
IV fluid

Cefluroxime (150 mg/kg)

Antibiotic contraversial

Bronchiolitis
0-2-years
May be insidious or acute
or progressive
Fever
Noisy breathing,
Expiratory wheezing,
Inspiratory crackers,
Intercontal retractions.
Cough
May be cyanosis
RSV or parainfluenza
Fluid maintenance
Bronchodilator
Oxygen
For infant: inhaled antiviral

The Joint Program of Family and Community Medicine - Jeddah

(3) Approach to Patient with Diarrhea


F. Rayes
Causes include infection, food intolerance, inflammatory, obstruction, functional.
The following items may need to be considered:

Establish good rapport


Encourage patient contribution
Respond to patients cues
Explore patients ideas, believes, expectations and concerns about the diagnosis,
o E.g.: Worms or food poisoning
o Cholera or dysentery
o Cancer or HIV
o Request for investigations or drug treatment or admission to hospital.

Details of the complain:


o Duration of complaints
o Frequency and consistency of stool
o Associated blood and mucus.
o Associated symptoms: E.g. fever, vomiting, abdominal pain, weight loss, fatigue
nervousness.
o Recent events or foreign travel
o Dietary indiscretion
o Family contact, occupation
o Drug history e.g. laxative, antacid, endomethacin diuretics, theophylline or
colchicin.

Other affected family members

Occupation, e.g. food worker


Examination:

General impression

Signs of dehydration

Examine abdomen

Per rectum examination may be indicated, if serious diagnosis is suspected


Possible Investigations:

Stool analysis

Culture faeces: if specific infection is suspected

Faecal occult bloods: if malignancy is suspected

Blood tests: for evaluation of general well being of the patient

Barium studies or endoscopy: for chronic diarrhea


Management:

Share diagnosis and share prognosis

Advise about diet and fluids

Use of medication:
o Electrolyte replacement (rehydration solution)
o Anti-diarrhea agents?!
o Antibiotics?!

Specific therapies

Referral if indicated

Follow-up arrangements if indicated

* See traveler advice for more details in management of diarrhea

83

The Joint Program of Family and Community Medicine - Jeddah

84

(4) Approach to Patient with Anemia


F. Rayes
Causes include nutritional, hemolytic, chronic GIT bleeding, or chronic diseases.
The following items may need to be considered:

Establish good rapport


Encourage patient contribution
Respond to patients cues
Look for possible complications of anemia:
o Dyspnoea, palpitation, heart failure, or fainting attack (in case of acute internal
bleeding)

Look for possible causes of anemia:


o Family history of anemia, e.g. thalassemia, G6PD or sickle cell anemia
o Drug (NSAIDs, Steroids)
o Blood per rectum / Black stool
o Dyspepsia (bleeding peptic ulcer)
o Hemoptasis, hematuria or menhorragia
o Regular blood donor
o Past history of chronic disease e.g. TB, Chronic UTI, RA, SLE or subacute
bacterial endocarditis
o Alcoholism

Explore patient ideas believes and expectation


Examination

Pallor: (Conjunctive, Lips, Nails)

Nails changes, e.g. Koilonychia (chronic severe anemia)

Evidence of haemoragic talangectasia

If anemia is severe or acute, look for evidence of heart failure.

Abdominal examination:
o Epigastric tenderness
o Renal tenderness
o Mass (cancer)
o Rectal examination:
o Piles or melena
Management and Education:

According to the type and the etiology of the anemia


Explanation and reassurance

Step-care investigations in patient with anemia:


Suspected anemia
CBC Findings
Iron deficiency anemia
Microcytic hypochromic
anemia
Anemia of chronic
disease
Beta thalassemia
B12 or Folate deficiency

Microcytic hypochromic
anemia
Micricytic or normocytic
Hypochromic anemia
Macrocytic anemia

* For more details see (Data Interpretation: Lab Tests)

Confirmatory test
Low serum iron
Low transferin saturation
Low ferritin
Low serum iron
Normal ferritin
Normal serum iron
Haemoglobin electrophorisis
Serum B12 level
And/ or serum Folic acid

The Joint Program of Family and Community Medicine - Jeddah

85

(5) Approach to Patient with Headache


H. Al Hajjar, M Alatta & F Rayes
Causes include tension headache, migraine, referred - pain (e.g. sinus, teeth, cervical
spine). Intracranial pressure (hypertension, tumor, meningitis), temporal arteritis.
The following items may need to be considered:

o
o
o
o
o

Establish good rapport


Encourage patient contribution
Respond to patients cues
Identify the characteristics of pain (Classical history of pain)
Onset & time, duration, site of pain and nature of pain.
Continues or intermittent.
Course (severity &, frequency)
Triggering or aggravating factors and reliving factors.
General health and well-being

Ask specific questions: e.g.


o Prodrome, aura of migraine, e.g. visual or sensory aura,
o Respiratory tract infection in sinus pain

Associated symptoms, e.g. neurological symptoms, fever, eye symptoms, nausea,


vomiting

History of head trauma or history of lumbar puncture.

ENT problem, any dental or vision problem, e.g. acute viral infection, COPD

Drug history:
o For the headache.
o For other medical causes.

Effect of the headache on patients life.

Psychosocial problems:
o New stressful events.
o Marital problems or problems at work.

Family history.

Exploration of any continues problems.

Exploration of patients concerns, worries, ideas and expectations.


Examination:

Blood pressure

Local possible sources of pain:


E.g. sinuses, temporal arteries, teeth, cervical spine, ears

Neurological examination
Management and education

Share diagnosis and share prognosis

Discussion of self-help, e.g. relaxation

Use of medication:
o Analgesics, anti-migraine or anti-depressant
o Specific medication for primary cause

Agree referral if indicated: Counselor or specialist

Follow-up arrangements if indicated


Possible investigations:
o Blood tests, e.g. erythrocyte sedimentation rate
o X-ray chest, cervical spine or CT scan

The Joint Program of Family and Community Medicine - Jeddah

o
o
o
o
o

86

Factors in the development of chronic daily headache from episodic migraine


Analgesic/ergotamine overuse
Abnormal personality profile, including depressive trait
Stress
Traumatic life events
Non-headache medications, including sex hormones

Alarm symptoms pointing to more serious disease headache


o Aura symptoms associate always with the same body side or with acute onset
without spread, or having either very brief (<5min) or unusually long (>60min)
duration
o Sudden change in migraine characteristics or a sudden substantial increase in attack
frequency
o Headache emerging after exercise (may indicate subarachnoid hemorrhage)
o Onset above age 50 (migraine and cluster headaches are not usually late onset)
o Aura without headache
o High fever
o Abdominal pain (could suggest acute ketoacidosis)
o Recurring neurological symptoms between headaches
o Abnormal neurological examination
o Increase intensity after 24 hours from onset.
o Change in cognition, level of consciousness or focal neurological findings.
o Neck rigidity.
o Abnormality in vital signs

Differential Diagnosis of headache


Tension Headache
Migraine
- Young adult and - Common in
middle age
young adult
- More in female
- Recurrent
- Recurrent
- Almost daily
- Once a week
- No significant
- Lasting from 8
associated
to 12 hours
symptoms
- Left side of the
- Triger factors
head.
- Associated with
malaise, nausea,
vomiting and
photophobia.
-Normal
- Normal
examination
examination

Subarchinoid Hge
- Severe headache
(the worst headache
of patients life)
- Associated with
exertion & vomiting

- ECG: similar to
IHD
- CT scan then LP
presence of blood

Cluster headache
- Common in
middle age
- More in male
- Recurrent, may be
every 4 weeks
- Awaken from
sleep
- Every night
- Same time
- Lasting one
hour.
- Deep burning
sensation
- Associated with
lacrimation flushing,
nasal discharge and
conjunctivitis.
- Ptosis & popullary
constriction.

The Joint Program of Family and Community Medicine - Jeddah

87

(6) Approach to Patient with Acne


N. Dashash
The following items may need to be considered:

Establish good rapport


Encourage patient contribution
Respond to patients cues
Identify the present complaint Acne
When has it started? Why now (E.g. preparation for social event)?

Is there any aggravating factor (E.g. stress, exams)

Previous treatments:
What sort of treatment? How long was each one used? Compliance?

Patient ideas: what he/she knows about acne

Patient concerns and fears:


(E.g. losing friends, scars, discolored skin, not getting married.)

Expectations: (E.g. referral to a dermatologist)

Effect of Acne on the patient (E.g. Relationship with friends)

Exploration of continuous problems:


DM, asthma, smoking
Examination:

Inspection of the face, shoulders, back, upper arms and chest looking for acne
Management:

Shared understanding of the problem:

Summary of what the doctor understood

Shared management & health education:


o Acne is a common problem, up to 80% of people had acne sometime in their life
o What is acne? Enlargement of the sebaceous gland (oil producing gland in the
skin), with blocking of its outlet and over growth of bacteria.
o It has no relation with being clean or not
o Chronic problem, needs patience in and tolerating the treatments
o It increases at times of stress such as exams, and is related to hormonal changes
(seen in women)
o Black heads and white heads are not dirt

Appropriate prescribing:
o Discussion of options: e.g. Topical: Retin A and/or Benzoil peroxide and/or
Systemic antibiotics e.g. minocyclin
o Explaining side effects and precautions.

Patient with such mild complain, may present with special communication problem, e.g.
requesting referral to a dermatologist or requesting special medications.

Candidate needs to show skills in dealing with demanding behavior:


o Empathy and caring attitude
o Logical negotiation of advantages and disadvantages of patients demand (referral
or medications)
o Nonjudgmental attitude
o Flexibility and respect of patient autonomy

The Joint Program of Family and Community Medicine - Jeddah


o

(7) Approach to Patient with Urinary Tract Infection


F. Rayes
The following items may need to be considered:

Establish good rapport


Encourage patient contribution
Respond to patients cues
Explore the nature of symptoms:
o dysuria
o frequency and pattern
o Haematuria
o Pain
o Fever

General well-being

Recurrent symptoms?

Symptoms in sexual organs or pain related to sexual activity


Examination:

Palpate kidneys and lower abdomen

Vaginal examination may be indicated


Investigations:

Urine dipstick nitrite

Urine bacteriology (MSU)

Vaginal swabs

Renal x-ray ultrasound

Blood creatinine
Management and education:

Alternative diagnosis
E.g. atrophic vaginitis, urethral syndrome, vaginal discharge

Use of:
o Antibiotics
o Analgesics
o Treatment of associated cause
o Referral
o Prophylaxis

Discuss nature and prognosis of complaint

Discuss management plan

Check self-care and lifestyle


o Adequate fluid intake
o Voiding after intercourse

Follow-up arrangements if necessary

Presentation of UTI in children:


Failure to thrive, fever, enuresis, frequency and dysuria
Management and follow -up:

MSU 2-4 days after starting antibiotic, if positive, patient need urgent referral for
possible obstruction

MSU 2 weeks after antibiotic, if positive repeat the course of antibiotic

MSU 3 months late if positive, patient need maintenance of antibiotic

All proves UTI in children under 5 should be referred for further investigations.
For more details see (Data Interpretation: Lab Tests)

88

The Joint Program of Family and Community Medicine - Jeddah

89

(8) Approach to Patient with Sore Throat


F. Rayes
The following items need to be considered in managing any episode:

Establish good rapport


Encourage patient contribution
Respond to patients cues
Explore the nature of the complaint:
o Duration
o Associated symptoms: fever, malaise, rash
o Prior medication
o Smoking habit
o Immunocompromised?
o Relevant past history or family history of rheumatic fever

Explore the patients concerns, worries, ideas and expectations.

Look for possible hidden agenda

Explore continues problems: e.g.


o DM, asthma or malnutrition vaccination coverage
Examination:

Inspect neck and throat

Palpate cervical glands

Other examinations:
o E.g. rash and spleen (Infectious mononucleosis)
Investigations:

Throat swab rarely indicated

Infectious mononucleosis blood test if it is highly suspected

Complete blood count may be indicated


Management and Education:

Use of:
o Analgesics: use enough dose and right frequency
o Antibiotics if bacterial infection is highly suspected
o Encourage symptomatic home remedies

Discuss disease and its cause

Discuss patients concerns (sick leave, wary about possibility of rheumatic fever)

Discuss management plan

Follow-up arrangements if necessary

Usually simulated patients with minor illness appear in the exam for testing certain skills,
E.g.:

Patient demanding referral for tonsillectomy

Patient with mild pharengitis demanding antibiotic

Simulated patient is a smoker and need counseling

Simulated patient has a hidden agenda, E.g. marital problem or parent may be using
the child as presenting complain

Malingering patient requesting sick leave

The Joint Program of Family and Community Medicine - Jeddah

90

Possible serious differential diagnosis:

Possible Diagnosis
o Epiglottitis
o Meningitis
o Quinsy
o Streptococcal sepsis.
o Rheumatic fever
o Palatal cellulitis

Comments
o Ashencolor, Drooling (children)
o Meningism (child. & young adult)
o Voice change, Trismus (all ages)
o Unstable vital sign (all ages)
o Murmur, Heart failure (Rare)
o Unilateral swelling, Marked tenderness.

Facts about use of antibiotic in tonsillitis:


20-40% of sore throat caused by GABHS
Incidence of rheumatic fever has no correlation with the use of antibiotic
Rheumatic fever runs in family, more in low social class
50% of +ve culture for GABHS have no serological evidence of infection (Carrier)
Treatment shorten the duration of illness by 24 hr & prevent supportive
complications.
o Antibiotic does not prevent development of glomerulonephritis
o
o
o
o
o

o
o
o
o
o

Indications for antibiotics


GABHS more likely
Peritonsillar abscess
Sinusitis
Prophylaxis in case of associated chronic diseases e.g.
DM, Asthma or cystic fibrosis.

Indications for tonsillectomy and admission


Tonsillectomy:
o Grossly enlarged tonsils with sleep apnoea.
o History of peritonsillar abscess.
o Frequent tonsillitis with otitis media.
Admission:
o Airway obstruction

The Joint Program of Family and Community Medicine - Jeddah

91

(9) Approach to Patient with Acute Otitis media


F. Rayes
The following items may be considered in managing any episode:

Establish good rapport


Encourage patient contribution
Respond to patients cues
Explore the nature of complaint:
o Pain , discharge from ear, and/or fever

Recent upper respiratory tract infection

Frequnecy of episodes

Hearing between episodes

At risk factors:
o Age or Downs syndrome,
o Immunocopromised

Explore the patients concerns, worries, ideas and expectations.

Explor continues problems: e.g.


o DM, asthma or malnutrition, vaccination coverage
Examination:

Examine both tympanic membranes

Examine nose and throat for congestion

Assess level of distress


Investigation:

Bacteriology swab if discharge


Management and education:

Prescribe antibiotic and pain killer

Discuss disease and its course

Discuss immediate concerns

Discuss current management

Follow-up arrangements made

Advise lifestyle and self-care: water and swimming

Management of Acute Otitis Media


o Amoxycillin 5-14 days

Review in 48 hours. if symptomatic :


o Insure compliance
o Exclude complications
o Change antibiotic.

If asymptomatic:
o Review in 4 days, in 30% of the patient the tympanic membrane will be normal
o The remaining 70% of the patient, they need to be reviewed every 3 months
o 10% persistent of the patient will continue to have persistant effusion and they will
need referral to ENT
Management of recurrent otitis media

Treat each episode with antibiotics

Use long term low dose antibiotic prophylaxis !

Insert ventilating tubes (grommets) !

Perform adenoidectomy !

The Joint Program of Family and Community Medicine - Jeddah

92

(10 )Approach to Patient with Dyspepsia


F. Rayes
Dyspepsia is a vague term; patient may has upper abdominal pain, heartburn, anorexia,
nausea, vomiting, flatulence and/or dysphagia. It includes a wide spectrum of differential
diagnosis, starting from functional disorders to malignant disorders
History:

Establish doctor-patient relationship

Encourage patient contribution

Respond to patients cues

Explore the nature history of the problem:


o Onset of dyspepsia;
o Chronic: most probably benign etiology
o Site of pain and radiate
o Frequency: cyclic (reflux or ulcer), continuous (dismotility)
o Severity and nature of pain: dull ache, colicky or staping
o Timing: worse at night or hungry (PU)
o After heavy meal or fatty meal: dysmotility or biliary colic.
o Continuous: could be malignancy
o Relieving factors: antacid, rest, strong analgesia, eating

Associated features:
o Reflux: cyclic, retrosternal pain, heartburn, regurgitation, water brash, weight gain
o IBS: change bowel habit, lower abdominal pain
o Dysmotility: ulcer like symptom (epigastric pain associated with meal or hunger
pain,
o Biliary colic: severe require strong analgesia
o Respiratory infection: cough
o Angina: dyspnoea, relieved by rest
o Depression: loss of interest and low mood
o Cancer: weight loss, dysphagia, vomiting

Drugs history: aspirin, steroids, NSAID, antacid or tagamet.

Exploration of patient ideas, concerns, expectations and believes


Examination:
o Abdominal examination: may be mild tenderness
Management and Education:

Work-up strategy based on risk stratification:


o Patient judged to be low risk: start empirical treatment
o Patient judged to be high risk: refer the patient for investigation

Advice in Reflux:
o Stop smoking and
o Life style modification
o Lose of weight if overweight
o Eat small frequent meals and avoid bedtime snacks
o Avoid late night eating
o Raise the head of the bed
o Avoid foods that upset you & avoid tight-fitting clothes
o Elevate head of bed may help

Advice in dysmotility:
o Small frequent meal
o Semi-liquid meals to avoid distension

The Joint Program of Family and Community Medicine - Jeddah

o
o
o
o
o
o
o
o
o
o

93

Advice in peptic ulcers:


Stop smoking
Avoid drugs like NSAIDs (explain)
Cola, coffee & tea with moderation and avoid alcohol
Milk & diary product with moderation
Inform patient about warning sign in PU e.g. black stools
Insure patients acceptance & understanding of your advice
Explain others management options e.g.
(Medications, dosage, frequency, side effect and any relevant precautions).
Reassurance: It is common disease and treatable
Availability of the doctor (you) for any problem or any questions any time.
Arrange for follow up

Drug treatment in patient with peptic ulcer


H2 - antagonist e.g. Cimetidin 800 mg at night, 400 mg BD.
Or
Proton Pump Inhibitors
e.g. Omeprazole 200 mg OD
Or
Sucralfate 1 g before each meal and at night
Antacid 30 - 45 mmol QSD after meals.
Management of dysmotility
8 weeks course lead to healing of 95% peptic ulcer

Drug treatment in patient with dysmotility:


o Metoclopramete: Short term Or Cisapride
o Antiulcer treatment might be tried but for a limited time and not to continue if
symptom fail to resolve

Drug treatment in gastro- esophageal reflux disorder


o Mild disease:
Antacid after meal & at bedtime
H2 - antagonist e.g. Cimetidine 400 mg QID
Or Ramtidine 300 mg BD (3 months)
o Resistant cases:
Omeprazol 20-40 mg OD / 8 weeks
Maintenance treatment H2 - antagonist
o In case of failure of medical treatment, refer patient for surgery

Indication for referral & investigations:


o If diagnosis is in doubt
o If malignancy need to be excluded, e.g. patient has weight loss, dysphagia, vomiting
o Patient age over 45 years
o The patients symptoms change, possibly indicating a new pathology or malignancy.
o Failure of empirical treatment

The Joint Program of Family and Community Medicine - Jeddah

(11) Approach to Patient with Irritable Bowel Syndrome


F. Rayes
History:

Establish doctor-patient relationship

Encourage patient contribution

Respond to patients cues

Explore the nature history of the problem


o Abdominal distention
o Pain eased after bowel movement
o Altered stool frequency, alter stool form and alter stool passage
o Urgency and feeling incomplete evacuation
o Passage of mucus

Risk assessment:
o Pain awaken from sleep or change of pain
o Onset at elderly
o Weight loss
o Rectal bleeding
o Steatorrhea and fever
o History of steadily worsening symptoms

Explore the patients concerns, worries, ideas and expectations.

Explore any continues problems: e.g. psychosocial problem


Examination and Investigation:
o Abdomen and per rectum examination
o Sigmoidscopy may be needed
Management:
o Develop effective Pt-Dr Relationship
o Acknowledgment of pain and treat with empathy

Reassurance:
o prevalence is 10-20% of adult population
o It is not progressive to a serious disease or develop complications
o 30% of the patient became symptomatic over time

Dont overreact & set reasonable treatment goal

Negotiate treatment & know your limitation


Education and counseling:
o Explain the diagnosis:
o The intestine squeeze food too hard or not hard enough to cause food to move too
fast or too slowly.
o Advice patient to increase high-fober foods like vegetables and fruits, whole grain
braed and cereals
o Drink plenty of water
o If gas is a problem to avoid beans, cabbage and some fruits
o Avoid food that increase the symptom, if milk and other dairy product bothers, the
patient may have lactose intolerance
o Stress management

Follow up arrangement

94

The Joint Program of Family and Community Medicine - Jeddah

Differential Diagnosis of IBS

With Diarrhea And/or Constipation:


o Colorectal Cancer
o Polyps
o Inflammatory bowel disease
o Chronic intestinal infection(e.g.
giardiasis)
o Coeliac disease

With Upper Abdominal Pain:


o PU
o Cholelithiasis
o Chronic pancriatitis

Drug Treatment

For diarrhea:
o Cholestyramin, Imodium orlomotil

For pain:
o Antispasmodic e.g. Mebeverin (Colofac)
135 mg TDS 30 min before meal.
o Pepperpment oil ( Colpermin, Mintec) .2 - 0.4 ml TDS 30 min before meal.
o Tricyclic antidepressant. Ametriptyline 25-75 mg.

For constipation:
o Osmotic laxative (Duphalac)10 mg TDS

For bloating:
o Low residue diet (low fiber)
o Peppermint oil. Cisapride 10 mg TDS.)

Risk of Colorectal Cancer


It is the second most common cancer in both males and femals

Risk factors :
o Familial adenomatous polyposis
o IBD > 20 Years
o Family history of colorectal cancer

Risk of colorectal cancer with an affected first -degree relative :


o One relative: risk 1 in 17
o Two relatives: risk 1 in 6
o Three relatives: risk 1 in 2

95

The Joint Program of Family and Community Medicine - Jeddah

7.

96

Data Interpretation: Lab Tests


A. Assaggaf & N. Dashash

Laboratory Findings in Selected Anemias


Thalasemias
Laboratory
Finding
Hemoglobin

Iron
deficiency
Decreased

Folate deficiency
anemia
Normal or
slightly decreased
Normal or
slightly decreased

GRPD

Sickle cell trait

Alpa

Hematlcrit

Decreased

Decreased

Normal or
slightly decrease
Normal or
slightly decrease

Normal or
decreased
Normal or
decreased

Decreased

Mean
corpuscular
volume

Decreased

Increased

Norma (?)

Normal

Normal or
decreased

Slightly
decreased

Mean
corpuscular
Hemoglobin
concentration

Decreased

Normal

Norma (?)

Normal

Normal or
decreased

Slightly
decreased

Transferrin
saturation

Decreases

Normal

Normal

Normal

Normal

Normal

Serum iron
Ferritin
Total iron
binding
capacity
Serum folate

Decreases
Decreases
Increased

Normal
Normal
Normal

Normal
Normal
Normal

Normal
Normal
Normal

Normal
Normal
Normal

Normal
Normal
Normal

Normal

Decreased

Normal

Normal

Normal

Serum B12

Normal

Normal

Normal

Normal

Normal or
decreased
Normal

Sickledx

Positive

Hemoglobin
A2

Normal or
decreased

Normal

Normal

Normal

Normal

Increase

Perihpheral
smear

Microcytosis,
hypochromia

Macrocytosi,
Neutropenia,
thrombocytopenia

Normal

Normal

Microcytosi,
Neutropenia,
poikilocyosis

Microcytosi,
Neutropenia
Target cells

Decreased

Beta

Decreased

Normal

The Joint Program of Family and Community Medicine - Jeddah

97

History and Physical Examination Findings in Macrocytosis Anemia:


History:

o
o
o
o
o

o
o
o
o

o
o

o
o

Medications
Alchol use
Nutrition
Strict vegetarianism (B12
deficiency)
Predominantly carbohydrate diet
(folate deficiency)
Advanced age
Poor dentition
Financial trouble
Neurologic symptoms
Gait disturbance
Parestjesoa
Loss of taste
Tongue paresthesia
Hepatic symptoms
Pruritus
Dark urine
Occupational
Radiation exposure
Chemical exposure

Hypothyroid symptoms
Cold intolerance
Fatigue
Change in voice
Constipation
Change in hair

Surgery
o Total or partial gastrectomy
Physical examination
o
o
o
o
o

o
o
o
o

o
o
o
o

o
o

Signs of liver disease


Jaundice
Spider angiomas
Ascites
Hepatosplenomegaly
Neurologic signs
Ataxia
Loss of vibratory sense
Loss of position sense
Weakness
Other
Glossitis
Vitiligo

Common Causes of Macrocytosis


Cause
o Alcohol abuse
o B12 or folate deficiency
o Chemotherapy or drugs
o Hemolysis or bleeding
o Liver disease
o Primary bone marrow disorders
o Hypothyroidism
o Others

Percentage of cases *
36
21
11
7
6
5
5
12

* Percentages total more than 100 percent because of rounding


Reference: American Family Physician CD

The Joint Program of Family and Community Medicine - Jeddah

98

Causes of Vitamin B12 Deficiency

o
o
o
o
o
o

Malabsorption:
Inadequate intrinsic factor
Pernicious anemia (gastric atrophy, lack of intrinsic factor secretion)
Gastrectomy
Gastric bypass surgery
Ileal disorders (sprue, regional enteritis, surgery, neoplasm)
Competition for B12 (fish tapeworm, blind loop syndrome)

o Drugs (cochicine,neomycin)

Inadequate intake:
o Strict vegetarianism

Other rare causes:


o Congenital absence or dysfunction of intrinsic factor
o Transcobalamin II deficiency

Causes of Folate Deficiency:

Inadequate intake:
o Alcoholism
o Advanced age
o Poverty

Decreased absorption:
o Tropical sprue
o Bacterial overgrowth
o Short-bowel syndrome
o Drugs (phenytoin, phenobarbital, oral
contraceptives)

Increased requirements:
Hemolytic anemia
Pregnancy
Exfoliative dermatitis
Infancy and growth

Loss:
o Dialysis

Chemical interference:
o Methotrexate
o
o
o
o

The Joint Program of Family and Community Medicine - Jeddah

99

Criteria for Diagnosis of Diabetes and ImpairedGlucose Hmoeostasis

Diabetes mellitus:
o Symptoms of diabetes plus causal plasma glucose concentration >or=200mg/dl
(11.1 mmol/l)
o Casual is defined as any time of day without regard to time since last meal. The
classic symptoms of diabetes include polyuria, polydipsia, and unexplained weight
loss
o Or
o FPG > or = 126 mg/dl (7.0 mmol/l). Fasting is defined as no caloric intake for at
least 8 h.
o Or
o 2-h PG > or = 200 mg/dl (11.1 mmol/l) during an OGTT. The test should be
performed as described by WHO, using a glucose load containing the equivalent of
75-g anhydrous glucose dissolved in water

Impaired glucose homeostasis:


o Impaired fasting glucose = FPG from 110 to < 126 mg per dL (6.1 to 7.86 to < 11.1
mmol/l)

Normal glucose homeostasis:


o FPG < 110 mg per dL (6.1mmol/l)
o 2 hr PPG < 140 mg per dL (7.75 mmol/)
FPG = fasting plasma glucose; 2hrPPG = two hour postprandial glucose
Guidelines call for symptoms of diabetes mellitus plus positive findings from any two of the three
tests on different days; symptoms include polyuria, polydipsia, and unexplained weight loss; casual
is defined as any time of day without regard to time since last meal.

Recommendations for Diabetes Screening of Asymptomatic Persons:


o Timing of first test and repeat tests
o Test at age 45; repeat every three years of age or older
o Test before age of 45; repeat more frequently then every three years if patient has
one or more of the following risk factors:
o Obesity: more than or equal 20% of desirable body weight or BMI more than or
equal 27 kg per m
o First-degree relative with diabetes mellitus
o Member of high-risk group (Black, Hispanic, Asian)
o History of gestational diabetes or delivering a baby weighing more than 4.032kg
(9lb)
o Hypertensive (BP > or = 140/90)
o HDL cholesterol level < or = 35 mg per dl (0.90 mmol/l) and/or triglyceride level >
or = 250 mg per dl (2.83 mmol/l)
o History of impaired glucose tolerance (IGT) or impaired fasting glucose (IFG) on
prior testing

The Joint Program of Family and Community Medicine - Jeddah

100

Liver Function Tests


Physical & Laboratory Findings in Selected Jaundice Syndrome
Jaundice
Syndrome

Hepatomegaly

Splenomegaly

Alkaline
Phosphatase

ALT

Bilirubin

Obstruction

__

__

3-8 times
normal

2-10 times
Normal

5-20 times
normal both
type

Viral Hepatitis

__

2-4 times

3-100
times

5-20 times both


type

Infectious
Mononucleosis

__

Normal 5
times

2-20 times

Conjugated <
10 times

Hemolytic

__

__

Normal

Normal

3-5 times
Unconjugated

Alcoholic
cirrhosis

Normal 3
times

Normal 3
times

3-10 times
conjugated

Primary
biliary
cirrhosis

4-30 times

Normal 2
times

< 4 conjugated

The Joint Program of Family and Community Medicine - Jeddah

101

Evaluation of Patient with Jaundice Syndrome:


Primary Conjugated
Hyperbilirubinemia
Liver enzyme evaluation

Elevated

Normal

Clinical evaluation

Suggestive of
Hypatocellular disorder
(aminotransferase
Elevation dominant)

Consider:
Infection
-- (Especially viral
hepatitis)
-- Drug-induced
Hepatitis

Serological
Evaluation

Suggestive of intraor extrahepatic biliary


obstruction
(alkaline phosphatase
elevation dominant;
history of symptomatic
cholelithiasis)

Ultrasonograph
Or
Computed tomography

Depending on
clinical presentation
consider :
-- Recent sepsis or
-- systemic infection
-- Rotor syndrome
--Dubin-Johnson
syndrome

The Joint Program of Family and Community Medicine - Jeddah

102

Hepatitis B makers: Summary of Interpretation:

HBsAg

HBeAg HBsAb

HBsAb HBeAb

Interpretation

Never infected or early incubation.

Early acute hepatitis,


Infectious

Early acute hepatitis,


Very infectious,
Greatest risk.
Acute hepatitis/chronic carrier,
Still infectious,
>3/12 chronic liver disease
Late acute resolution,
Chronic carriers (? Improving).
Infection soon resolving:
Still infectious, Chronic carriers,
Sometimes indicated improvement.

Convalescent, may still be infectious


(Virus not detectable by current
methods)
Recovery phase,
No longer infectious.

Recovery phase,
May last several years.

Usually convalescent,
May be found years after viraemia,
Favorable prognosis.
Infection years ago:
HbcAb lost or artificially immunized
and never had HbcAb.

The Joint Program of Family and Community Medicine - Jeddah

103

Management Strategy for Toxoplasma


Serologic Testing During Pregnancy
1
2a
2b
3a
3b
4a
4b
5a

5b

6a
6b

Serologic testing preferably before conception or the first prenatal visit.


If negative lgG, retest at 18-22 weeks.
If positive lgG, immediately repeat tests including lgG and lgM
If lgG positive, stable, or low titer, IgM negative, suspect old or preconception
exposure. Repeat tests in 2-4 weeks
If lgG positive, any titer, lgM positive, suspect recent exposure.
Repeat tests in 2-4 weeks.
If repeat lgG positive, titer stable, lgM negative, confirm old
exposure . No further testing necessary.
If repeat lgG positive, titer rising, lgM positive, suspect recent exposure.
Repeat tests in 2-4 weeks.
If repeat lgG and lgM positive (Stable or declining titer). Suspect
recent exposure 2-6 months before tests obtained. Treat
according to gestational age at time of presumed infection and according to
patient wishes.
If repeat lgB and lgM positive with rising titers, suspect active
recent infection. Treat according to gestational age and patient
wishes.
If repeat serologic tests negative, repeat tests at 36 weeks.
If repeat serologic tests positive, retest as in (3).

Bruecllosis Treatment

Adults:
Tetracyclin 500 mg QID for 6 wks Or Doxycylin (vibramycin) 100 mg BID for 6 wks
+
Streptomycin 1 gm IM once/day x 2 wks

Children: Treat for 6/52:


- Rifampicin 20 mg/ kg/ day once (OD)
- Septrin (TMP/SMZ) 10 mg /kg/ day in 2 divided doses

Alternative treatment for children:


o Children <9 yrs:
-Septin for 6/52 +Streptomycin 20 mg/ kg/ day/ M (OD) max(1gm/day) 2/52
o Children >9 yrs
-Tetracycline 30-40 mg kg/ day max (2 gms/day) in 4 divided doses 6/52
+
Streptomycin 2/52 OR Septrin 3/52

Relapse:
o Children < 9 yrs Rifampicin for 6/52 + Streptomycin for 2/52
o Children > 9 yrs Tetracycline for 6/52 + Septrin for 3/52

2
6
3
4
5

The Joint Program of Family and Community Medicine - Jeddah

104

Thyroid Function Tests

Thyroid disorders

T3

T4

Subclinical hypothyroidism

Free T4
Index
N

TSH

TRH
test

Hypothyroid

Hyperthyroid

N,

N,

N,

N,

Eutyroid Syndrome
Secondary hypothyroidism
(Pituitary hypothalamic disease need
further investigations of hypothalamic
pituitary function
TSH secreting pituitary tumor or
Thyroid hormone resistance

The Joint Program of Family and Community Medicine - Jeddah

105

Urine Analysis
1.

Pyuria: (Pus cell in urine)


o 5 cells
o Differential Diagnosis:
1. Pyelonephritis
2. UTI
3. Urethritis
4. Prostatitis

2.

UTI:

Urine:
o
o
o
o

Pyuria
Bacteruria > 105 [in symptomatic female > 102] Hematuria in 30 % of patients
Nitrate +ve in 80%
pH alkaline in Proteus and Klebsiella.
Organisms:

o
o
o
o
o
o

E. Coli (80%) (gm ve)


Staph. (gm +ve)
Klebsiella (gm ve)
Proteus (gm ve)
Enterococci
Mixed
Investigations:

o C/S
o Further radiological investigations.

D Dx:
o
o
o
o

Cystitis
Prostatitis
Urethritis
Malignancy

Management of Uncomplicated UTI:

General:
o fluid
o Hygiene advice: washing front to back
o Correct underwear (cotton)

Specific:
o C/S
o Antibiotics
o Septrin DS BID X 3 days
o Cephalexin 250 500 mg Q 60 for 3 days
o Cipro 250 500 mg BID for 3 days
o Amoxil 250 500 mg TDS for 10 14 days

Drug Management of UTI in Children:


o Septrin 6 12 mg / kg / dose BID for 10 14 days
o F/U: for C/S & possible referral for radiological investigations.

The Joint Program of Family and Community Medicine - Jeddah

3.

Oxaluria:
Causes:
1. Ileal dis. (Crohns ileal resection)
2. Hereditary (primary)
3. Food, most common: tea, nuts, chocolate, dark green vegetables
Risk: increase renal stone

4.

Glucosuria:

Causes:
DM
Low renal threshold (< 180 mg/ml)
Pregnancy
Cushings
Hyperthyroidism
Investigations:
FBS
Management:
According to the confirmed diagnosis.

1.
2.
3.
4.
5.

5.

106

Contaminated Urine:

o Epithelial cells > 10 : mainly squamous, transitional cells.


o Pus < 5
o Mixed organism

Advice:
o Reassurance
o Proper MSU (washing first, then allowing a small amount of urine to flow, before
filling the sample tube. Avoid contact to skin)

6.

Protein-urea:
Causes:
(Normal = trace (0 30) mg/ml)
1.
2.
3.
4.
5.
6.

Nephrotic synd. = ++++ (> 2000)


G.N. = 2 + 4 + (100 2000)
Pyelonephritis 1+2+
CRF = 1 + 2 +
False +ve = in pyuria & epithelial cells
Physiological:
Fever
Exercise
Investigations:
o 24 hour collection
o Cholesterol
o U/S abdomen

The Joint Program of Family and Community Medicine - Jeddah

7.

Urine Casts:
o
o
o
o
o
o
o
o

8.

Hyaline Non-specific
Waxy advanced R.F.
RBC G.N.
Epithelial cast acute tubular injury, G.N, Nephrotic synd.
WBC Pyelonephritis, G.N, interstitial nephritis.
Granular Nephritis
Faulty (lipid) Nephritic
Mixed G.N.

Hematuria:
Def.: > 5 cells
Causes:

1.
UTI
2.
Prostatic diseases
3.
Neoplasm in urinary system
4.
Renal stone
5.
Trauma
6.
Renal disease: G.N.
7.
Drugs: Aspirin, Penicillin, and Cephalosporin.
8.
False + ve = exercise, vitamin & food
9.
False ve = vitamin C.
Investigations:
1.
2.

9.

MSU, CIS, UIE, CBC, ESR


IVP, US, Cystoscopy, CT, angiography, biopsy.

Renal Stone:

MSU:
o Hematuria
o pH: Acidic = uric acid & cystine
o Crystals (can be normal).

Investigations:
o KUB
o U/S

Management:
o General:
- Increased fluid
- Pain killers
o Specific:
- Thiazide (in calcium oxalate stones)
- Allopurinol (in uric acid stones)
- Alkalization of the urine by NaHCo3 (uric acid & cystine)

107

The Joint Program of Family and Community Medicine - Jeddah

10.

Sterile Pyuria:

Deferential diagnosis: Pyuria with negative culture (Causes):


1. Chlamydia
2. T.B.
3. Stone
4. Malignancy
5. Viral
6. Partially treated UTI
7. SLE

11.

False Red Urine:


1. Drug: Rifampicin
2. Food: blackberry, beat root
3. Oxidation of crystals (in children).

12.

pH:
o pH (5-8): Most.bacteria : Alkaline
o E.g. Klebsiella, Pseudomonas & Staph. (in UTI).
o TB: Acidic

13.

Specific Gravity:
o Normal =
1.003 1.030
o Osmolality: Normal =100 900
o Both test the ability of kidney to concentrate urine (i.e. renal function)
o Increase in = Dehydration, drugs
o Decrease in = D. insipidus, Primary polydipsia, starvation, exercise, drugs.

Indications for IVP:


o
o
o
o
o
o

Children under 5 year of age


Women with persistent bacteruria or 2 function in one year
Haematuria after eradication of infection
Persistent pyuria after eradication of infection
Infection with less common organism
Asymptomatic bacteruria and pyclonephritis in pregnancy
Risk of IVP:

o
o

1 % may develop anaphylactic reaction


Mortality rate is 1:10,000 - 1: 40,000 %
Interpretation of MSU:

o
o
o
o
o

No significant growth: < 103 + Pyuria + Symptoms


Unusual organism
Anaerobes or viruses
Analgesic nephropathy
Antibiotic is taken before the sample.

108

The Joint Program of Family and Community Medicine - Jeddah

109

Urine culture (Bacteruria)

o Suprapubic:Any number is significant


o MSU: < 10,000 is suggestive
100,000 is an evidence of infection
If the patient is asymptomatic: It is nesecarily to have 2 consecutive colonies >
100,000 of same organism.

MSU in children:
o

Before starting antibiotic


105 /ml pure culture is reliable
< 105 /ml mixed growth is not significant (Becteruria)
Significant Bacteruria:

o
o

> 105 /ml + Pyuria = Infection


> 105 /ml + No pyuria = False positive
Intermediate bacterial counts:
104 - 105 /ml + Pyuria = Infection, if pure culture and fresh specimen
More than one organism + no pyuria = contamination
High mixed count = contamination.

o
o
o

Treatment of Common Infestations

Schistosomiasis:
o S. Mansoni , S. Japonicum and S. and Haematobium (Urinary tract)

Praziquantel

o
o
o
o
o

40 mg / kg / day
2 divide doses BD (4-6 hrs apart) for one day
60 mg / kg / day TD (for S. Japonicum) for one day

Hookworms (N. americanus, A. duodenal) Nematodes = (Round worms)


Mebendazole (Vermox)
100 mg PO BD y 3/7 (children > 2 y)
Pyrantel Pamoate
10 mg/kg (max. 1 gm) single dose (light infection)
Or 10 mg/kg PO OD y 3/7 (heavy infection)
Albendazole
40 mg PO once
Treatment of Anemia
Iron treatment F. Fumarate 200 mg Bid for 6/52
Or F. Gluconate 325 mg Bid for 3/12

Enterobius Vermicularis (Pinworm, threadworm)


o Mebendazole
100 mg PO once
Repeat at 2 weeks one dose (age 2 yrs or more)
o Pyrantel Pamoate
10 mg/kg PO once (tab. = 125 mg)
Repeat at 2/52 and 4/52 (0, 2, 4/52).
Note: * Hygiene, cutting nails, bath after rising in the morning
* Treat whole family
* Continued symptoms. means reinfection
* Adult worm die after 6/52
o Albendazole (Alternative)
400 mg PO once (repeat in 2/52).

The Joint Program of Family and Community Medicine - Jeddah

Giardiasis:
o Metronidazole (250 400 mg)
PO Q 8 hrs x 5/7
o
(Flagyl)
or (2 mg)
PO OD x 3/7
Note:
- Hygiene
- Treat whole family
- If R1 fails check compliance, re-infection
- If diarrhea results avoid milk (lactose intolerance for 6/52)

Ascariasis (Nematodes roundworms)


o Mebendazole 100 mg PO BD x 3/7
o Levamisol
120 150 PO once Drug of choice for mass treatment

Amoebiasis (E. histolotica)


o Metronidazole
800 mg PO Q 8 hr x 10 days
o In case of liver abscess: aspiration.

Trichuriasis (whipworm):
o Mebendazole 100 mg PO BID x 3/7 repeat 2 courses
o Albendazole 400 mg PO once

Strongyloides S.:
o Thiabendazole
22 mg/kg
PO BID x 2/7
o Albendazole
400 mg
PO qd x 3/7
Prednisone 20 40 mg qd x 3-5/7
Re-treatment.

Summary:
Mebendazole (Vermox) treats all parasitic infestations
Except
o Ameobiasis, Giardiasis:
Metronidazole
o Strongyloides:
Thiabendazole
o Schistosomiasis:
Praziquantel

110

The Joint Program of Family and Community Medicine - Jeddah

8.

111

Data Interpretation: Slides


N. Dashash

General Guidelines for Slide & Data Interpretation


1) Read questions carefully.
2) If interpreting Data: Comment on the abnormal as well as the abnormal
3) If you are asked to describe lesion
Describe it fully:
o Shape: e.g.: round, regular or irregular, location and Distribution
o Colour:e.g : erythematous, brownish, pearly
o Ulceration
o Crusting and colour of scales
o Lichnification, Bullae, bleeding pointsetc.
4) Helpful Hints in Studying slides:
Approaching slides on infectious disease:
o What is the caustic organism
o How can you diagnose it?
e.g.-Micros-copy and culture of scrapings
-Woods light examination. color change.
-Special tests for some disorders.
o What is the distribution of the Rash.
o How is it prevented eg: hygiene, eradication of vectoretc.
o What is your management or action?
A good approach is to mention what you would actually do in your practice. eg.
o Explaining the problem to the patient.
o Investigation (if you need further information such as culture).
o Advise : eg: hygiene, treating other members of the family if disease is contagious.
o Preventing and treating complication.
o Prescription of medication, including generic name (best), dose, how to use,
frequency and duration
o Referral

Dermatology & General Medicine

Acute candidiasis of the oral cavity (thrush):

Description: Whitish layer that wipes off easily, leaving a bleeding surface.
Characteristic feature on microscopy:
o Fungal hyphae usually in the mouths of debilitated patients e.g. anaemias.
Precipitating factors:
o Diabetes mellitus
o Hypothyroidism
o Hypoparathyroidism
o Drugs e.g. contraceptive pill, antibiotic therapy, systemic steroids.

The Joint Program of Family and Community Medicine - Jeddah

112

Pitryasis Vesicolor:

Appearance:

o The eruption manifests itself either as pink or brownish patches on a pale


background, or as hypopigmented patches on a dark background
o The organism produces azaieic acid which bleaches melanin.
Causative Organism:

o The yeast malassezia furfur.


Differential diagnosis: Vetiligo
Treatment:

Refer to handout.

o With topical agents e.g. benzoic acid compound ointment BPC (Whitfielfs
ointment), selenium sulphide lotion, creams or lotions of the imidazole group. None
offers a permanent cure.

Scabies:

Description:

o Excoriations due to generalized pruritus. Pruritus starts about six weeks after the
disease is acquired.
o Burrows in finger webs.
Transmission: By close physical contact with an infested person.
Causative organism: Sarcoptes scabiei hominis, scraped out of a burrow.
Treatment:

o By two application of an anti-scabetic lotion (benzyl benzoate or gammabenzene


hexachloride) to the whole skin surface below the chin, on two occasions 24 hours
apart for patient and close contacts. Pruritus can take up to two weeks to settle,
during which time no further anti-scabetic treatment should be used.
o Children and pregnant woman Benzyl benzoate
o Wash the drug off after 24 hours
o After treatment, wash clothes and bed linen in hot water.

The Joint Program of Family and Community Medicine - Jeddah

113

Systemic Lupus Erythematosus (SLE):

Appearance: Butterfly (Malar) rash of face of young females.


Other features of SLE:
- Pleuritis, pericarditis,
- Nephritis, Anemia
Diagnostic test:
o Antibodies to double stranded DNA, are found in over 80 per cent of patients.

Cutaneous Leishmaniasis:

Transmitted by:
The sandfly
Causative organism: The protozoan leishmania tropica
Diagnosis:
Smear demonstrates the organism with Wrights or Giemsa stain.

Pyogenic granuloma:

Often a result of trauma.


Differential diagnosis:
Amelanotic melanoma: The lesion after removal should be sent for
histological examination.
Treatment: - Silver nitrate application.
- Curettage
- Diathermy under local anaesthetic

Kobners Phenomina:

Seen in:

Lichen planus
Proriasis
Vetiligo
Warts
Moluscum contgiosum

Acanthosis nigricans:

Associated with:

Malignancy of an internal organ:


o Occurs in almost 100 per cent of cases of the acquired type in
non-obese adults.
o Commonly adenocarcinomas.
o 85 per cent being intra-abdominal.

The Joint Program of Family and Community Medicine - Jeddah

114

Nail Changes in Psoriasis:

1) Pitting
2) Onycholysis
3) Discoloration
4) Thickening
5) Subungual Hyperkeratosis
Differential Diagnosis:
1. Trauma
2. Eczema
3. Onychomycosis
Treatment of psorisis:

Dithranol, tar, corticosterods

Emoliants

Systemic PUVA, UVB

Methotrexate, etretinate.

Basal cell carcinoma:

Associated with:
Excessive sun exposure
X-ray treatment of the area
Contact or medication with arsenic
Treatment: By curettage, cryotherapy or excision

Insulin induced fat hypertrophy (lipodystrophy):

Dermatological side effects of insulin:


Local reactions:
o Immediate or delayed erythema, keloid formation
General reactions:
o Erythema multiforme, urticaria, purpura.

Palmar Erythema:

Causes: i) Cirrhosis (especially alcoholic)


ii) Normal pregnancy
iii) Rheumatoid arthritis
iv) Thyrotoxicosis
v) Dermatological disorders eczema, psoriasis, pityriasis rubra pilaris.
vi) Others include polycythaemia, diabetes mellitus, mitral valve disease, beriberi.
vii)
May occasionally be inherited

The Joint Program of Family and Community Medicine - Jeddah

Henoch-Schonlein Purpura:

Complications:
Causes:

Bleeding from the gut and nephritis


Usually none is found.
In former years streptococcal infection was thought to be important.
It is now regarded as an immune complex disease but the antigen is
usually difficult to identify.

Xanthelasmata:

Causes:
Essential familial hypercholesterolaemia
Primary biliary cirrhosis
Diabetes mellitus
Usually no underlying cause is found
Treatment: By cautery or trichloracetic acid or excision

Leprosy:

Characteristics:
Hypo-or hyper-pigmented macule
Anaesthesia, neuritis
Causative organism: Mycobacterium leprae
Diagnosis:
Skin scraping and microscopy
Biopsy
Lepromin test (Mitsuda test)

Stevens Johnson Syndrome:

Causes:
Infections: -Herpes simplex
-Mycoplasma
-Streptococcal
-T.B.
Drugs: Barbitarates, penicillin, sulphonamides
Neoplasia: e.g. Hodgkins disease
Connective tissue diseases

115

Malignant Melanoma:

Differential Diagnosis:

Seborrheic keratosis
Compound melanocytic nevus

The Joint Program of Family and Community Medicine - Jeddah

116

Moluscam Contagiosum:

Tiny white (pearly), umbilicated papules


Common in
- Children
- Atopics
- Immune suppressed
Caused by Pox virus.
Complications: secondary infections
Treatment:
Removal:
- By curettage
- Expression of contents
- Phenol or codeine.

Chicken Pox (Varicella):

Complications:
Secondary infection
Thrombocytopenic purpura
Encephalitis
Varicella pneumonia (usually in adults and those with an impaired immune response).

Herpes Zoster:

Commonly affects trunk then cranial nerves

Complications of herpes zoster ophthalmicus:

Conjunctivitis, keratitis, iridocyclitis, optic neuritis (rarely).


Encephalitis.
Secondary streptococcal/staphylococcal infection.
Haemorrhagic zoster (purpura fulminans).
Dissemination (immunosuppressed patients).
Post-herpetic neuralgia.
Treatment:

Acyclovir, vidarabine, or idoxuridine will limit viral replication.


Systemic corticosteroids may reduce the incidence of post herpetic
neuralgia (contraindicated in the immuno-compromised host).
Local corticosteroids and atropine if anterior uveitis has developed.
(immunosuppressed patients).
Post-herpetic neuralgia.

The Joint Program of Family and Community Medicine - Jeddah

117

Lesions on the shin


1)

Erythema nodosum:

A common vasculitis reaction of larger subcutaneous vessels. In many cases no causes is


found.

Causes:

2. Infections
Streptococcus
Tuberculosis
Infectious mononucleosis
Viral
Chlamydia
Leprosy
Fungal infections
Lymphogranuloma
Scratch disease
Blastomyosytis
Coccidomyositis
Yersenia
3. Drugs
Sulphonamides
Oral contraceptives
Salicylates
Bromides/iodides
Gold salts
4. Inflammatory bowel disease, Behchets disease
5. Lymphoma
Clinical features:
Erythematous, tender, nodules appear on shins and occasionall thighs and forearms.

Associated pyrexia, malaise, oedema and aching of legs

Color bright red to purple to brownish

Recurrences may occur

Most attacks settle within 2-12 weeks


Treatment:

Bed-rest

Anti-inflammatory analgesics

Support stockings or bandages

Systemic steroids may be required for severe cases

2)

1. Sarcoidosis

Necrobiosis Lipoidica:
Lesions are sharply marginated, yellow or brownish yellow areas of shiny atrophy
with telangiectasia.
Many but not all cases are diabetic.
Associated with diabetes mellitus.
Complication ulceration.
Differential diagnosis: Erythema nodosum; diabetic dermopathy; pretibial
myxoedema.

The Joint Program of Family and Community Medicine - Jeddah

3)

Pretibial myxoedema:

Caused by Graves disease

Other dermatological signs of throtoxicosis:

Treatment:

118

Vitiligo or diffuse hyperpigmentation


Diffuse alopecia
Clubbing of the fingers and toes
Soft tissue swelling (i.e. thyroid acropachy)
Onycholysis
Urticaria

Corticosteroids:

Under occlusive plastic dressing


Injected intralesionally

Angioedema

Aetiology

Hereditary angioedema is an autosomal dominant condition due to C1 esterase inhibitor


deficiency in the complement cascade.
Angioedema may also occur in urticaria.

Clinical features

Swelling of the lips, periorbital area, neck and joints.


The larynx may be affected and this can be fatal.
Gut involvement may produce abdominal pains.
Investigation

Anyone who suffers recurrent attacks of angioedema or in whom there is a family history
of angioedema should be positively screened for C1 esterase inhibitor deficiency.

Treatment

Specific drugs such as stanozolol, tranexamic acid and danazol are the only
effective prophylactic agents for hereditary angioedema.
Acute episodes:
o require injection of C1 esterase inhibitor
o infusion of fresh frozen plasma.
Angioedema associated with ordinary urticaria may be treated with antibistamines
and / or steroids.
Severe attacks should be treated as for anaphylaxis, with 0.5 ml 1/1000 adrenalin
by subcutaneous injection.

The Joint Program of Family and Community Medicine - Jeddah

119

Oral Conditions

Lichen planus:
Mucous membrane lesions occur in 50% of cases
Differential diagnosis of lichen planus of the buccal mucosa
o Chronic irritation from gum-biting or ill-fitting bridges
o Leukoplakia
o Candidiasis
o Aphthous stomatitis
o Squamous papilloma
o Verruca vulgaris
o Secondary syphilis
Black hairy tongue:
o May follow antibiotics, cytotoxics and excessive smoking
o Elongated papillae (hair) may be yellow-brown or black

Leukoplakia:
o Small, discrete, white patches or more extensive, leathery plaques on an atrophic
erythematous base.
o Predisposed by tobacco smoking and recurrent trauma
o Risk of malignant change

Causes of Mouth Ulcers:

1. Inflammatory:
o Infective: -Bacterial: Vincents angina, T.B, syphilis,cancrum oris
-Viral: herpes
-Fungal: candida
o Non-infective: - Traumatic: ill fitting dentures, Cheek biting, Burns
o
- Radio therapy
2. Haematological: leukaemia, agranulocytosis
3. Neoplastic: malignant ulcer
4. Auto-immune: -Aphthous (commonest)
o Others: Behccets, Reiters, Steven- Johnson syndrome
5. Skin conditions: pemphigoid, pemlphigus vulgaris, Lichen planus
o Vitamin deficiency:
o Vitamin C, Riboflavin, Nicotinic acid

The Joint Program of Family and Community Medicine - Jeddah

120

Ear, Nose and Throat


Interpreting Rinne and Weber Hearing Tests:
Interpretation

Rinne
Tunning fork in front
of Pinna
Positive AC > BC

Normal
Conductive hearing loss
(e.g. perforation, cyromette tube)

Sensorineural hearing loss

N.B.: AC = Air Conduction,

Weber
Tunning fork (512 H2 ) on
skull in midline
No Lateralization

Negative BC > AC

Lateralizes to the abnormal


ear
i.e. Sound perceived louder in
abnormal side.
Positive AC > BC
Lateralizes to the normal ear
i.e. Sound perceived louder in
the normal unaffected ear.
BC = Bone Conduction.

Tympanosclerosis:

White patches chalk patches


Past otitis media
T.M. Trauma or perforation
Complications: T.M. and ossicular immobility conductive deafness.

Appearance:
Causes:

Tympanic Membrane Haematoma (Bleeding):

Causes:

Treatment:

Barotrauma: from common cold or airplane descent injury, e.g. slap


on check.
Usually no treatment is required as it is self limiting.

The Joint Program of Family and Community Medicine - Jeddah

121

Ophthalmology
Diabetes Mellitus:
Although diabetes may have a number of ocular effects (e.g. cataracts, changes in
refractive status), the most important ocular complication is retinopathy.
1. Non-proliferative diabetic retinopathy (NPDR):

The retinal findings of mild and moderate NPDR include-micro-aneurysms (fine


red dots)
o Retinal haemorrhage (blots) Hard exudates (yellow lipid deposits.
o Macular Oedema

Severe NPDR: increased vascular tortuosity


o Venous beading, (widespread intraretinal micro-aneurisms) micro-infarctions of the
nerve fiber layer, or Soft (cotton wool) exudates
o 40% will develop proliferative diabetic retinopathy within 1 year.
2. Proliferative diabetic retinopathy (PDR):
o Responsible for most of profound visual loss from diabetes.
o New blood vessels (neovascularization) in the area of the optic disc and elsewhere
on the retinal surface or elsewhere in the eye; for example, on the surface of the iris
(rubeosis iridis), causing severe glaucoma.
o If not treated, these fragile new vessels bleed into the vitreous. Fibrous tissue in the
new vessels will contract and cause retinal detachment.

Treatment:
o Control of D.M. by diet, exercise and medication
o All patients with retinopathy should be referred to an ophthalmologist for
examination and follow-up.
o Patients with macular edema will require laser photocoagulation.
o For severe complications (massive vitreous hemorrhage or traction retinal
detachment), a vitrectomy may be necessary.

Hypertension Retinopathy:
Keith-Wagener-Barker Classification
Normal
Grade I
Grade II
Grade III

1/3

Present
Present
Present
Present
Slight
Right-angle

Grade IV
A/V ratio*
Fine cords
Flame hemorrhages
Present
Exudates
Present
Papilledema
Present
Copper wiring
Silver wiring
AV nipping
Same as grade
III
*A/V ratio refers to the ratio of the diameter of the arteriole to the venule.

Management of hypertension retinopathy:


o The primary goal is adequate control of the blood pressure General principles
weight reduction,
o Reduced salt intake lipid-lowering
o Direction of smoking reduction of alcohol consumption
o Anti-hypertensive drugs

The Joint Program of Family and Community Medicine - Jeddah

122

Allergic Conjunctivitis

Appearance:
Upper lid: papillae, if severe [giant papillae] leads to cobble stone appearance.

Treatment:
o Sodium cromoglycate (opticrom 2% up to 4%) q 6 hrs. PRN
o If excessive tearing: short course of anti histamine, e.g.
- Livostin eye drops Bid.
- Konjuntival eye drops.
- Naphcon eye drops.

Surgery
Suture Materials
1

o
o
o

o
o
2o
o

o
o

Absorbable sutures
Organic: Catgut is the commonest example
Useful in intestinal anastomosis
closure of peritoneum
stitch7 of fat or subcutaneous tissues.
Synthetic (Dexon, Vicryl):
Synthetic absorb-ables cause less reaction and are superior.
Stronger than catgut handle and tie better than catgut, but take a longer duration
before absorption.
Non-absorbable sutures:
Organic (silk). Tie and handle easily, may precipitate infection (i.e. not preferred)
Synthetic (prolene, nylon). More difficult to handle, little tissue reaction meaning
that infection is less with it ( i.e. preferred for skin closure)
Needle types
Cutting needles used for skin or tendons.
Round-bodied needles for anastomosi of the GI tract and vascular work.
Guidelines for suture removal:

o
o
o
o
o
o

Face and neck: 3-4day


Scalp: 5-7 days
Abdomen and chest: 7-10 days (up to 14 days after aortic surgery)
Limbs. 5-7 days.
Feet: 10-14 days.
N.B. Sutures can be removed earlier than these times for cosmetic reasons.
Steristrip may be used as an alternative especially in children.

The Joint Program of Family and Community Medicine - Jeddah

9.

ECG Interpretation
A. H. Hassan

Contents:

o
o
o
o
o

Part I: Basic ECG Principles.


PART II: Practical Applications:
Ischemic Heart Disease.
Chamber Enlargement.
Conduction Defects.
Digitalis Effects and Electrolyte Disturbances.
Arrhythmias: - Ventricular Fibrillation (VF) and Ventricular Tachycardia (VT).
- Atrial Fibrillation (Afib) and Atrial Flutter (AF).
- A-V Heart Blocks.

Objectives:

Every physician is expected to be able to:


o Appreciate the difficulties in mastering ECG interpretation,
o Differentiate between normal and abnormal ECG tracings,
o Interpret properly abnormal ECG findings,
o Master efficiently the topics covered, and
o Improve in the future his basic knowledge and skills about ECG interpretation
discussed here.

Is it a difficult task:
1) There are 12 leads.
2) There are 5 waves in each lead.
3) There are 3 segments or intervals.
4) In any given tracing, there are a minimum of 14 points to analyze.

PART 1: Basic ECG Principles (14 points to analyze)


1) Standardization
o 1 m Volt = 10 mm.
2) Rate
Regular Rhythm:
o 300 / number of large squares.
o 1500 / number of small squares.
o 300 / 150 / 100 / 75 / 60 / 50.

Irregular rhythm:
o Number of cycles in 6 sec. x 10.

123

The Joint Program of Family and Community Medicine - Jeddah

124

3) Rhythm

o
o
o
o

Normal Sinus Rhythm:


Rate = 60 - 100 / min.
Regular rhythm.
+ ve P II.
- ve PaVR.

4) P-wave

Normal:
o Duration: - < 0.12 sec.
o Amplitude: - <2.5 mm.

Abnormal:
o Wide: - Left Atrial Enlargement (LAE).
o Peaked: - Right Atrial Enlargement (RAE).
5) PR- INTERVAL

Normal:
o 0.12-0.2 sec.

Abnormal:
o Prolonged: - Conduction defects.
o Shortened: - Pre-excitations e.g. Wolff Parkinson White (WPW) syndrome.

6) Mean Electrical Axis Deviation of the QRS Complex

Calculation:
o 1) Midway between the axes of two extremity leads that show tall R of equal
amplitude.
o 2) Right angle (90o) to any extremity lead that shows a biphasic complex.

Normal Axis: (-30o) to (+110o) degrees.


Abnormal Axis Deviation:

Right Axis Deviation (RAD):


o Normal,
o Right Ventricular Hypertrophy (RVH).
o Lateral Myocardial Infarction (MI),
o Chronic Obstructive Pulmonary Diseases (COPD),
o Acute Pulmonary Embolism.
Left Axis Deviation (LAD):
o Normal,
o Left Vevtricular Hypertrophy (LVH),
o Left Bundle Branch Block (LBBB),
o Left anterior hemi-block.

The Joint Program of Family and Community Medicine - Jeddah

7) QRS Duration

o
o
o
o
o

Normal: - 0.1 sec. or less.


Wide QRS complex:
Bundle Branch Blocks (BBBs),
Premature Ventricular Contractions (PVC),
WPW,
Hyperkalemia,
Toxicities: Tricyclic Antidepressants (TCAs), Procainamide, quinidine.
8) QRS amplitude

o
o
o
o
o
o

Short:
Normal,
Obesity,
Pleural Effusion,
Extensive MI,
Emphysema
Myxedema.

Tall:
o Young adults.
o Thin chest wall.
o Ventricular Hypertrophy
9) R wave Progression

o
o
o
o

Poor Progression:
COPD.
LVH.
Anterior MI.
LBBB.

o
o
o
o
o

Prominent RV1
Infants and children.
RVH.
Posterior MI.
RBBB.
WPW.

10) ABNORMAL Q-WAVE


o Width: - > 0.04 sec.
o Amplitude: - > 25% R wave.

125

The Joint Program of Family and Community Medicine - Jeddah

11) QT interval

o
o
o
o
o
o
o

Normal duration: - 0.4 sec; (should be corrected for the heart rate).
Prolonged:
Hypocalcemia.
Hypokalemia.
Ischemia.
Hypothermia.
Subarachnoid Hemorrhage.
Procainamide.
Quinidine.

Shortened:
o Hypercalcemia.
o Digitalis effect.
12) ST segment

Elevation:
o Acute Q wave (transmural) MI.
o Prenzmetal angina (non-infarctional transmural ischemia).
o Pericarditis.

Depression:
o Acute Non- Q (sub-endocardial) MI.
o Classic angina.

13) T-wave

Normal:
o Same direction as the QRS complex.

o
o
o
o
o

Abnormal:
Ischemia.
MI.
Ventricular Strain.
Pericarditis.
BBBs.

14) U-wave
o Hypokalemia.
o Toxicity.

126

The Joint Program of Family and Community Medicine - Jeddah

127

PART II: Practical Applications


1) Ischemic Heart Disease

Transmural MI.:
Acute:
o Persistent ST elevation,

Evolving:
o Q wave.
o Deep T- wave inversion.

Sub-endocardial (non-Q.) MI.:


o Persistent ST depression, &/or
o T-wave inversion.

Transmural Ischemia (Prinzmetal angina):


o Transient ST elevation, or
o Increased T wave positivity.

Sub-endocardial Ischemia (classic angina):


o Transient ST depression.

o
o
o
o
o
o
o
o

Site:
Ant.: - V2-4.
Ant.-septal: V1-3.
Ant.-Lateral: I, aVL, V4-6.
High Lat.: I, aVL.
Extensive Ant.: V1-5.
Inferior: II, III, aVF.
RV: - Right Chest Leads.
Posterior: Prominent RV1 (R>S), ST depression, positive TV1,2.
2) Chamber Enlargement

Right Atrial Enlargement: Peaked P (Pulmonale) > 2.5 mm, or Initial positive PV1
(>1.5mm).
Left Atrial Enlargement: Broad notched P (Mitrale) >0.12 sec. (>3 small squares),
or biphasic PV1 with terminal negative part (>0.04 sec./ >1 mm.).
RVH: RAD, r SR` V1, R V 1 > S V 1, RV5 SV5 or 6, RV1 > 10 mm, qR in V1,
SV1 < 2mm, ST-segment depression, & T-wave inversion V1-3.
LVH: SV1+ RV5 > 35 mm., R a VL > 11 mm., R I+ S III > 25 mm., Rv5 or 6
>26, or Lewis index 17 (RI-SI) + (SIII-RIII), Additional points (Left ventricular
strain pattern, Left atrial abnormalities).

The Joint Program of Family and Community Medicine - Jeddah

128

3) Conduction Defects

o
o
o
o
o

RBBB:
Wide QRS, (if narrow incomplete BBB).
Prominent R V1.
Wide slurred R V1,2.
Wide slurred S I,aVL, and V5 or 6.
ST-segment depression, and T- wave inversion V1-3.

o
o
o
o
o

LBBB:
Wide notched QRS.
Prominent notched R I, aV L, V5,6.
Wide S V1.
ST-segment depression, and T- wave inversion I, aVL, V5-6.
No Q-wave in I and V5,6.

Digitalis Effects and Electrolyte Disturbances:

Digitalis Effects: Bradycardia, Coved concave, or scooped ST-segment


depression, and Short QT interval.
Digitalis Toxicity: 1st., 2 nd., or 3 rd. Degree Heart Block, Marked Sinus
Bradycardia, 1st., AT, AF, Afib, PVCs, VT, and/or VF.
Hyperkalemia: Tall (peaked, slender, tented) T, flattened P wave, wide QRS, STsegment elevation, A-V conduction defects, VT., VF, asystole.
Hypokalemia: U wave, ST- segment depression, flattened or inverted T wave,
increased amplitude and width of P wave, prolonged PR interval.
Hypercalcemia: Markedly short QT interval (even absence of ST- segment).
Hypocalcemia: Prolonged QT interval.

4) Arrhythmias

If There is no Normal Looking QRS Complex:


o Ventricular Fibrillation.
o Ventricular Tachycardia.

If There is no Normal Looking P Wave:


o Atrial Fibrillation.
o Atrial Flutter.

What is the Relationship Between P Wave and QRS Comples?


o Fixed Prolongation of the P-R Interval: 1st Degree HB.
o Progressive Prolongation of the P-R Interval: 2st Degree type I HB.
o No Relation:
- 2nd Degree type II HB: No complex without P-wave before it.
- 3rd Degree HB: Complex appears without P- wave before it.

The Joint Program of Family and Community Medicine - Jeddah

10.

129

Data Interpretation: Radiology


M. Al atta

General Advice on X-ray interpretation


Basic interpretation of the chest X-ray is easy. It is simply a black and white film and any
abnormalities can be classified into:
1.
Too white.
2.
Too black
3.
Too large.
4.
In the wrong place.
To gain the most information from an X-ray, and avoid inevitable panic when you see an
abnormality, adopt the following procedure:
1.
Check the name and the date.
2.
Check the technical quality of the film.
3.
Scan the film thoroughly and mentally list any abnormalities you find.
4.
When you have found the abnormalities, work out where they are:
5.
Always ensure that the film is reported on by a radiologist.
6.
Finally do not forget the patient. It is possible and indeed quite common for a
very sick patient to have a normal chest X-ray.

Exams and the normal X-ray


Spotting the abnormality in an apparently normal chest X-ray is a common question in
postgraduate exams. If confronted with such an X-ray, then think of the following
possibilities:
o Apical shadowing
o Left lower lobe collapse
o Hiatus hernia (fluid level behind the heart)
o Dextrocardia (with the X-ray shown the wrong way around)
o Mastectomy
o Air under the diaphragm

The Joint Program of Family and Community Medicine - Jeddah

130

Chest X-Ray
Causes of a pleural effusion

o
o
o
o
o

Transudate <30 g/l of protein


Heart failure, e.g. congestive cardiac failure, pericardial effusion
Liver failure, e.g. cirrhosis
Protein loss, e.g. nephritic syndrome, protein-loosing enteritis
Reduced protein intake, e.g. malnutrition
Iatrogenic, e.g. peritoneal dialysis

o
o
o
o
o
o

Exudates >30 g/l of protein


Infection, e.g. pneumonia, tuberculosis
Infection
Malignancy, e.g. bronchial carcinoma, mesothelioma, metastasis
Collagen vascular disease, e.g. rheumatoid arthritis, SLE
Abdominal disease, e.g. pancreatitis, subphrenic abscess
Trauma/surgery

The coin lesion


The term coin lesion is used to describe a discrete are of whiteness situated within a lung
field. It is not necessarily strictly circular. The main worry is that it may represent a
carcinoma

Causes of single coin lesions


o Benign tumor, e.g. hamrtoma
o Malignant tumor, e.g. bronchial carcinoma, single secondary
o Infection, e.g. pneumonia, obsess, tuberculosis, hydatid cyst
o Infarction
o Rheumatoid nodule
Fine nodular shadow or
Reticulonodular shadow in a Chest X-Ray:

o
o
o
o
o

Differential Diagnosis
Miliary T.B.
Sarcoidosis
Pneumoconiosis
Haemosiderosis
Miliary carcinomatosis

Cavitating lung Lesion


Some coin lesions may cavitate and if you have identified a coin lesion it is important to
look for features of cavitation

Causes of Cavitating lung lesions


o Abscess
o Neoplasm
o Cavitations around pneumonia
o Infarct
o Rheumatoid nodules (rare)

The Joint Program of Family and Community Medicine - Jeddah

131

Bronchiectasis
Bronchiectasis can be difficult to diagnose on a plain chest X-ray. If you suspect it as a
cause of increased shadowing then look for the following features:
1. Ring shadows. These look like rings and are any size up to 1 cm in diameter.
2. Tramline shadows. Look for these towards the periphery of the lung.
3. Tubular shadows. These are solid thick white shadows up to 8 mm wide.
4. Glove finger shadows. These represent a group of tubular shadows seen head on
and look like the fingers of a glove hence the name!
The presence of any of these features suggests the possibility of bronchiectasis. A
normal chest X-ray does not however exclude the diagnosis and CT scanning is the
most sensitive diagnostic test available.

o
o
o
o

Causes of bronchiectasis
Structural, e.g. Kortagener syndrome, obstruction (carcinoma, foreign body)
Infection, e.g. childhood pertussis or measles, tuberculosis, pneumonia Immune,
e.g. hypogammaglobulinaemia, allergic bronchopulmonary aspergillosis
Metabolic, e.g. cystic fibrosis
Idiopathic to stasis

Fibrosis
Fibrosis is one of the rarer causes of white lung and you need to differentiate it from
consolidation or edema, which are far more common.

Causes of fibrosis:
o Cryptogenic
o External/occupational, e.g. extrinsic allergic alveolitis, asbestosis
o Infection, e.g. tuberculosis, psittacosis, aspiration pneumonia
o Collagen vascular, e.g. rheumatoid arthritis, SLE
o Sarcoid
o Iatrogenic, e.g. amiodarone, busulphan, radiotherapy
Chickenpox Pneumonia
Chickenpox pneumonia in adulthood can cause the development of numerous calcified
nodules. To determine whether this is a likely diagnosis:
1. Look at the distribution of the nodules. In chickenpox pneumonia they tend to be
lower and midzone.
2. Look at the density of the nodules. They are calcified and so should be very white
in appearance.
3. Look at their size. They are usually less than 3 mm in diameter.
4. Look at the number. In chickenpox pneumonia you would expect to see less than
100 nodules. If there are obviously a lot more you should question this as a
diagnosis.

o
o
o
o
o

Causes of numerous calcified nodules


Infection, e.g. TB, histoplasmosis, chickenpox
Inhalation, e.g. silicosis
Chronic renal failure
Lymphoma following radiotherapy
Chronic pulmonary venous hypertension in mitral stenosis

The Joint Program of Family and Community Medicine - Jeddah

132

Pneumothorax

When you see a unilateral black lung you need to:


1.
Check the technical quality of the film.
2.
Determine which side is abnormal

o
o
o
o
o
o
o

Causes of Pneumothorax
Spontaneous
Iatrogenic/trauma, e.g. pleural tap, transbronchial biopsy, central venous line
Insertion, mechanical ventilation
Obstructive lung disease, e.g. asthma, COPD
Infection, e.g. pneumonia, tuberculosis
Cystic fibrosis
Connective tissue disorders, e.g. Mar fans, Ehlers-Donlos

Unilateral Hilar Enlargement


Hilar enlargement always warrants further investigation

Causes of hilar lymphadenopathy


o Neoplastic e.g. spread from bronchial carcinoma, primary lymphoma
o Infective, e.g. tuberculosis
o Sarcoidosis (rarely unilateral)

Causes of hilar vascular enlargement


o Pulmonary artery aneurysm
o Poststenotic dilatation of the pulmonary artery

Bilateral Hilar Enlargement


As with unilateral hilar enlargement bilateral hilar enlargement can be due to enlargement
of pulmonary arteries, veins or lymph nodes.
The commonest causes of bilateral hilar enlargement are pulmonary hypertension and
Sarcoidosis.

o
o
o
o

Causes of bilateral hilar lymphadenopathy


Sarcoid
Tumors, e.g. lymphoma, bronchial carcinoma, metastatic tumors
Infection, e.g. tuberculosis, recurrent chest infections, AIDS
Berylliosis

Causes of pulmonary hypertension:


o Obstructive lung disease, e.g. asthma, COPD
o Left heart disease, e.g. mitral stenosis, left ventricular failure
o Left to right shunts, e.g. ASD, VSD
o Recurrent pulmonary emboli
o Primary pulmonary hypertension

The Joint Program of Family and Community Medicine - Jeddah

133

Causes of pericardial effusions:

Transudate
o Congestive cardiac failure

Exudates
o Post myocardial infarction
o Infection, e.g. tuberculosis, bacterial
o Neoplastic infiltration
o Collagen vascular, e.g. rheumatoid arthritis, SLE
o Iatragenic, e.g. post cardiac surgery
o Endocrine myxoedema

Blood
o Trauma
o Neoplastic infiltration
o Aortic dissection
o Bleeding diathesis, e.g. anticoagulation, leukemiaCauses of pericardial effusions
The Widened Mediastinum
Always look carefully at the mediastinum. If you think that it is widened then relate this
finding to the clinical history. If you suspect an acute aortic aneurysm then you must
follow up your suspicions as quickly as possible with a CT, echocardiogram or MRI.
Important causes of a widened mediastinum are thyroid enlargement, enlargement of
mediastinal lymph nodes, aortic dilatation of the esophagus or thymic tumours.

Metastatic deposits
Your examination of the chest X-ray is not completed until you have looked carefully at
the ribs. They should be of a uniform density with smooth, unbroken edges. The main
abnormalities to look for are old and new fractures and metastases.
1. New fractures. Look along the edges of reach rib. A new fracture will be seen as a
break in the edge.
2. Old fractures. Again look along the edges. The callous formation that follows a
fracture will cause the rib to expand at this point.
3. Metastases. These look like dark holes in the ribs.
4. Look carefully at the other bones, which may contain similar pathology.

Diagnosis of rib fractures


Rib fractures can be missed on a chest X-ray. The diagnosis is therefore clinical and the
chest X-ray is usually performed to look for potential complications.
Surgical emphysema
At first sight surgical emphysema gives a very messy appearance, which is sometimes
confined to the obvious soft issue areas but may spread over the whole X-ray.

Causes of surgical emphysema


o Trauma
o Iatrogenic, e.g. surgery, chest drain insertion
o Obstructive lung disease, e.g. asthma
o Esophageal injury
o Gas gangrene

The Joint Program of Family and Community Medicine - Jeddah

134

Pancoasts tumour
A number of abnormalities can be easily missed. Before dismissing an X-ray as normal:
1.
Look carefully at the apices of both lungs.
2.
Look carefully at the heart shadow
3.
Look carefully at the mediastinum.
4.
Look at the hilum.
5.
Obtain a lateral film.
6.
Read the radiologists report!

Skull X-Ray
Causes of Sull Rdiolucencies

Normal
o Squamous temporal bone
o Pacchionian granulations
o Surgery
Air
o Superficial after scalp injury
o Intracranial seen in open fractures
Outer skull table
o Rodent ulcer
Inner skull table
o Slow growing tumours
o Chronic Subdural hematoma
Diffuse lesions
o Metastases
o Multiple myeloma
o Pagets disease
o Hyperparathyroidism

Generalized Skull Rdiolucencies


o Renal osteodystrophy
o Fibrous dysplasia
o Fluorosis
o Acromegaly
o Drugs-for example,
o Phenytoin
o Hemolytic anemias
Multifocal Skull Radiolucencies
o Sclerotic metastases
o Pagets disease Multifocal
o Pagets disease
Localized Skull Radiolucencies
o Foreign body
o Hyperostosis frontalis interna
o Osteroma
o Meningioma
o Hair bunch

The Joint Program of Family and Community Medicine - Jeddah

Bone and Joints


Hyperparathyroidism / Findings in hand X-Ray:
o
o
o
o

Eroded tufts of terminal phalanges


Subperiosteal cortical resorption
Fine lattice like pattern of bone
Generalized loss of bone density

Supracondylar fracture
Complications

Early:
1. Volkmans Ischemia:
(4 Ps : pain, pallor, pitting edema, pulseless)
o pain, more on extending fingers
o pallor cyanosis of hand
o edema of forearm
o low pulse.
2. Nerve injuries
3. Volkmans contracture.

Later:
1. Myositis ossfficans
2. Elbow stiffness
3. Malunion
X-Ray Findings in Osteoarthritis:
o Subchondral sclerosis
o Cyst
o Osteophytes
o Loose bodies
o interosseous distance (Joint space)
o trabecular thickness
X-Ray Findings in Rheumatoid arthritis:
o Soft tissue swelling
o Periarticular osteoporosis
o Loss of joint space
o Erosions (small bites from bone adjacent to joints)
o Bone destruction & deformity (later features)
X-Ray Findings in Osteoporosis:
(Lateral thoracic & Lumbar spine)
o Vertebral deformation
o in bone density
o Biconcavity
o Anterior wedging
o Compression
Congenital Dislocation of the Hip

Early: difficult to interpret, as so much of the joint is cartilage.

Later:
o Delay in development of ossific centres of acetabulum and femoral head.
o The acetabular roof has an upward slop
o The femoral head is displaced upwards and laterally.

135

The Joint Program of Family and Community Medicine - Jeddah

136

The Joint Program of Family and Community Medicine - Jeddah

11.

136

Data Interpretation: Self-assessment Exercise


Hana Al Hajjar

Question (1)
A 47-year-old male brought to the primary care center at 10 am by a friend. He has
chest pain and shortness of breath for the last 40 min. on arrival he was
semiconscious. His blood pressure was 99/55. His ECG is shown below.

A)

What is the diagnosis ?

B)

What is your management ?

137

The Joint Program of Family and Community Medicine - Jeddah

Question (2)
A 34-year-old female G3P2+0 presents for routine prenatal care. The result of the
50 g one hour glucose screen was 175 mg/dl
100 g oral GTT was done and the result was
Test

Patients value

Normal value

Fasting

94 mg/dl

105 mg/dl

1 hour

180 mg/dl

190 mg/dl

2 hour

178 mg/dl

165 mg/dl

3 hour

150 mg/dl

145 mg/dl

A)

What is your diagnosis ?

B)

Out line your management plan?

The Joint Program of Family and Community Medicine - Jeddah

138

Question (3)
A four-year-old child presents for audiogram and tempanometry testing . his results
are shown as

A)

What is your interpretation ?

B)

List three possible causes ?

139

The Joint Program of Family and Community Medicine - Jeddah

Question (4)
The association between hypertension and myocardial infarction is being investigated
in a study. The findings of a question send to the whole population ( 1000 ), all of
whom responded, are in the table

Whole population
History of hypertension

History of MI
present

Absent

Present

15

185

Absent

795

A)

What is the prevalence of hypertension ?

B)

What is the prevalence of MI ?

C)

Draw 2x2 table

The Joint Program of Family and Community Medicine - Jeddah

Question (5)

ECG

A)

What is the diagnosis?

B)

List two possible causes?

C)

What is the management

140

The Joint Program of Family and Community Medicine - Jeddah

Question (6)
This is the thyroid function test of a 42 year old lady
T4 . 7 g/dl
T3 . 134 ng/dl
T3 uptake . 19 %
TSH . 22 g/dl
FIT .. 4.3

( 5 11.5 )
( 100 215 )
( 25 35 )
( 0.7 7 )
( 6 11.5 )

A)

What is your interpretation?

B)

What is the most common cause?

C)

What further investigation will confirm your diagnosis?

Question (7)
47 year old female complaining of increased weakness and fatigue for the last two
years. She found to have a blood pressure of 155 / 97 mm Hg. Her blood chemistry
result is shown below
BUN . 60 mg/dl
Creatinine .... 4 mg/dl
Na .....146 mmol/l
K .. 5.9 mmol/l
Cl ..... 110 mmol/l
Ca 8 mg/dl
Phosphate 7.6 U/L
T. Protein . 5.6 g/dl
Albumin .. 3 g/dl
Fasting blood sugar 96 mg/dl

( 5 25 )
( 0.5 1.5 )
( 135 153 )
( 3.5 5.3 )
( 95 106 )
( 9 10.5 )
( 0 - 5.5 )
( 6.6 8.7 )
( 3.8 4.8 )
( 70 110 )

A)

Comment about his serum calcium level

B)

What is your interpretation ?

141

The Joint Program of Family and Community Medicine - Jeddah

Question (8)
A 65 year old male presented to you with this urine analysis result. He has past
history of ureteric calculi. He is a known hypertensive for the last 3 years. He is
asymptomatic.

Colour: yellow turbid


Deposit : +
Ph: acidic
Odour: normal
Nitrate: nil
Protein: trace
Glucose: nil
Ketones: nil
Urobilinogen: N. Trace
Bilirubin: n

Blood ( haem ): +++


WBC/HPF: 8 10
RBC/HPF: 15 20 (clumps +++)
Epith cells: ++
Casts: hayaline casts ( + )
Parasite + ova: nil
Mucuos thread: +++
Chemical deeposits: nil
Others: nil

A)

What is your interpretation?

B)

List five most probable causes?

A)

What further investigation this patient needs ?

142

The Joint Program of Family and Community Medicine - Jeddah

Question (9)
52 year old daibetic, smoker presented with the result of his annual cholesterol
screening
T. Cholesterol . 236 mg/dl
HDL 24.6 mg/dl
TG .. 267 mg/dl
FBS 101 mg/dl
A.

A)

Calculate his LDL level.

B.

B)

What is your management?

Question (10)
65 year old male presented for an evaluation after noticing that he was a little bit
yellow colored . he reported no other symptoms.
His initial laboratory results ware as follows.
AST .70 U/L
ALT 90 U/L
ALK P.tase 450 U/L
Bilirubin .. 4.5 mg/dl

A.

A)

What are the two most probable causes of such result?

B.

B)

What further investigation this patient needs?

143

The Joint Program of Family and Community Medicine - Jeddah

Answers to the Self-assessment Exercise


Answer (1)
A)

What is the diagnosis ?


Acute or recent inferior infarct and acute posterior infarct

B)

What is your management ?


Assessment in the first ten min of arrival to primary care:
Vital signs, O2 saturation
Cardiac monitor ( 12 leads ECG )
Start one or two IV lines
Draw blood for cardiac enzymes, electrolytes, coagulation studies
Portable CXR ( if available )
Immediate general treatment: (MONA)
Morphine IV 1-3 mg repeated at 5 min interval as needed
O2 by face mask or nasal canula
Nitroglycerin sublingual
Aspirin PO 160 325 mg
Adjuvant treatment:
B-blockers ( unless SBP< 100, HR< 60, 2nd or 3rd degree heart
block )
ACE inhibitors ( in 12 24 h )
Selected thrombolytic therapy: ( if available )
TPA ( Altepase ) or Streptokinase
Arrange for referral accompanied by doctor or nurse

Answer (2)
A) What is your diagnosis ?
Gestational diabetes
B)

Out line your management plan ?

Explanation to the patient


Shared care with the hospital
Diet therapy
May need insulin therapy if FBS > 95mg/dL or 2PP > 120 mg/ dL
Testing for fetal well being by non stress test twice weekly and
by biophysical profile from 32/34 week gestational age
Monitor for maternal , fetal and neonatal complications

144

145

The Joint Program of Family and Community Medicine - Jeddah

Answer (3)
B)

What is your interpretation ?

Conductive hearing loss left ear


Type B tympanometric curve
B) List three possible causes

Impacted wax
Otitis media
Tympanic membrane perforation
Patent ventilating tube

Answer (4)
A) What is the prevalence of hypertension ?
(15 + 185 ) / 1000 = 200 : 1000 or 20 %
B) What is the prevalence of MI ?
( 15 + 5 ) / 1000 = 20 :1000 or 2 %
C) Draw 2x2 table

History of
hypertension

+ ve
- ve
Total

History of MI
+ ve
- ve
4
36
1
59
5
95

Answer (5)
A)

What is the diagnosis ?


2nd degree heart block, mobitz type 2
Fixed PR interval then missed beat

B)

List two possible causes?


Degeneration
Acute MI

C)

What is the management


Needs ultimate ventricular pacing

Total
40
60
100

The Joint Program of Family and Community Medicine - Jeddah

Answer (6)
A)

What is your interpretation?


hypothyroidism

B)

What is the most common cause?


Hashimotos thyroditis

C)

What further investigation will confirm your diagnosis?


Antimicrosomial antibodies
Antithyroglobuline antibodies

Answer (7)
A)

Comment about his serum calcium level

He is having low apparent calcium coz of his low albumin level


His corrected serum calcium is calculated as Ca + 0.8 ( 4 albumin )
B)

What is your interpretation ?

Chronic renal failure plus


Renal osteodystrophy

Answer (8)
A)

What is your interpretation?


Microscopic hematuria
Possible renal parenchymal disease
Urinary tract infection

B)

List three most probable causes?

C)

Tumor
Calculus
Infection
Drugs
Systemic diseases
Essential

What further investigation this patient needs ?

Referral for biopsy


Urine culture and sensitivity
Ultrasound
Renal function test

146

The Joint Program of Family and Community Medicine - Jeddah

147

Answer (9)
A)

Calculate his LDL level.

T. Cholesterol - HDL - TG/5 = mg/dl


236 24.6 - 267/5 = 158 mg/dl
B)

What is your management?

This patient is high risk for cardiac disease, primary prevention


( DM, smoker, age 52, male, loww HDL, high LDL )
Aim to decrease LDL to < 130 mg/dl
Diet
Medications ( pravastatin or lovastatine )
Exercise
Smoking cessation
Start daily aspirin
Non pharmacological ( garlic, antioxidants, psylium and omega 3 fatty acids )
Follow up
Answer (10)
Raised AST & ALT hepatocellular injury
Raised ALK phosphatase obstruction
GTT is very sensitive to liver insult but is not specific and may increase in : alchohol,
antiepileptic medication , DM and history of MI
A.

A)

What are the two most probable causes of such result?

Obstructive or infeltrative disease


Consider malignancy
B.

B)

What further investigation this patient needs?

Drug history
Ultrasound

The Joint Program of Family and Community Medicine - Jeddah

12.

149

Approach to the Oral Exam


F. Rayes

Introduction:

The broad goals of this examination are to provide evidence of satisfactory


completion of the Family Medicine Board Program for Family and Community
Medicine, and to provide evidence of competence to practice.
The oral exam assesses areas of competence not tested in other parts of the
examination
Since the MCQ-type questions test factual knowledge reliably and extensively,
hard facts and figures are not part of oral assessment
During oral examination the examiner will be looking at your problem solving
skills and attitude and your consideration of illness in physical, psychological and
social terms
In the face of critical challenge from the examiners, the examiner will be looking at
your abilities to make decisions and to justify the conclusion they reach.
In general, the type of problem set in Modified Essay Questions type component of
the written paper will not be repeated in oral exam as the oral has its own focus

The range of oral exam questions:

Problem definition: this covers the candidates clinical thinking skills, his ability
to think broadly, logically and to have a high index of suspension (safe doctor)
Management (Problem solving): this covers the decision-making process. A
situation will be set or clinical problem raised and the candidate will be asked to
critically think about it and discuss his management options.
Prevention: this covers the basic knowledge of epidemiology, which is needed by
family physician.
Practice organization: this covers the candidates awareness about the system and
the regulation of health care service in his country, the obstacles and challenge
towards primary health care (PHC). It also includes practice management issues
and the PHC team
Communication: this encompasses verbal and non-verbal communication
technique, skills for effective information transfer and principles of communication
and consultation
Professional values: this covers general moral and ethical issues, patient
autonomy, medico-legal issues, flexibility and tolerance, implications of style of
practice, role of health professionals, cultural and social factors
Personal and professional growth: this focuses on the candidates personal
approach to continuing professional development, self-appraisal and evaluation,
stress awareness and burnout management

The Joint Program of Family and Community Medicine - Jeddah

150

General Approach to Oral Exam Questions

o
o

o
o
o

o
o
o

Listen to every single word in the question, and think broadly for possible issues
the examiners try to raise with you.
Analyze of the problem and remember the family medicine dimensions (physical,
social, psychological, ethical problems, or management problemetc) and
describe to the examiners your analysis of the problem.
This step is very important for the following reasons:
It helps you to give comprehensive answer
It demonstrates to the examiner your holistic approach in thinking
Organize your answer according to the different dimensions of the problems e.g.
You can say for the physical part of the problem I need to do ..etc.
For the ethical part of the problem I need to do . etc.
See next table (Systematic Thinking)
The examiner may give you a challenging question:
Remember the examiner is not looking for a definite answer (yes or no). He/she
will be looking at your abilities to make decisions
Be flexible and discuss options, advantages and disadvantages and avoid strong
statements.
When a preferred course of action is chosen, justify it in a rational and coherent
manner.
Systematic Thinking

The examiner may present to you a short case scenario, and ask you an open-ended
question, e.g. how would you proceed?
Suggested Systematic Thinking:
o Remember the consultation models
o You may need to ask for more history
o History, include patients ideas, concerns and expectations regarding his illness,
important past history and risk factors.
o Be specific and ask for relevant information only
o Think loudly
o Think broadly
o Common things first
o Have high index of suspicioun for serious possibilities
o If you are asked about physical examination, the examiner generally wants to know
your objectives from conducting a physical examination, he will not be interested
to examine your skills in conducting physical examination

The examiner may present to you a short case scenario, and ask you about
differential diagnosis
Suggested Systematic Thinking:
o Infection,
o Auto-immune
o Endocrine
o Psychological
o Malignancy
o Miscellaneous
o When you discuss your differential hypotheses, be ready to demonstrate to the
examiner how the differential hypotheses were proven or refuted
o Avoid giving number of problems or causes you are sure you will fill this number,
e.g. dont say this is caused by three cause, rather say: it can be caused by.. and ..

The Joint Program of Family and Community Medicine - Jeddah

151

The examiner may present to you a short case scenario, and ask you about the
etiological factors of this situation

Suggested Systematic Thinking:


o Physical
o Social
o Psychological

The examiner may present to you a short case scenario, and ask you about your
management of this situation
Suggested Systematic Thinking
o In your management plan, consider the patients physical, psychological and social
factors contributing to his illness.
Remember CRAPRIOP:
o C = Clarification
o R = Reassurance
o A = Advice & counseling
o P = Prescribing: If you mention a drug, be willing to specify dose, side effects,
precautions and contraindications
o R = Referral: Dont forget to make use of primary health care team e.g., social
workers. If you decide to refer the patient, be able to discuss
advantages and disadvantages and outcome expected
o I = Investigation: When you think a problem needs investigation, dont forget to
take into account the financial cost, effect on patient and benefit
expected
o O = Observation: Continuity of care is one of your important role as a family
physician.
o P = Prevention of complications
The examiner may present to you a short case scenario, and ask you what is the
effect of the problem
Suggested Systematic Thinking
o The effect on the patient,
o The effect on the family,
o The effect on PHC team,
o The effect on the community

The examiner may present to you a short case scenario, and ask you what is your
role in managing a health problem?
Suggested Systematic Thinking
o Patient management,
o Disease management,
o Practice management

The examiner may present to you a short case scenario, and ask you How do you
explain this behavior?
Suggested Systematic Thinking
o Patient factors,
o Doctor factors or PHC team,
o Disease factors,
o Others.

The Joint Program of Family and Community Medicine - Jeddah

152

Tips and Pitfalls in Oral Exam

Misunderstanding of the Question:


Formulate the question the way you understand it and ask the examiner: Is this what
you mean by your question?

Thought Block:
o Think of alternative solution
E.g. I cant remember the dose of the drug, but I can look for it in BNF
Or I may use the practice protocol or the emergency room protocol.
o Think broadly & answer broadly.
You do not need to be very specific e.g. guidelines in management instead of
drug treatment.
o Review the list of problem definition: Some time specific questions need specific
answers. Be ready for it.

o
o
o
o

Very short answers and long silence:


Having system of thinking will help you to be fluent
Remember it is your stage, express yourself as much as you can
Instead of keeping silent, think loudly and express your uncertainty
Proceed in talking unless the examiner stops you.

Rigid Answers:
o Instead of saying I should do it is preferable to say I may do so
o Instead of saying I always do it is preferable to say I prefer to do so
o Instead of saying I never do it is preferable to say I do not prefer to do so.

No Clear Answer:
o Go around the question
E.g. Q1.
What is your policy?
Q2.
What are you going to do now?
Q3.
What are you going to tell the patient?
Answer: Honestly I have no definite answer, However;
My Objectives are so and so, I may do so, I prefer to do so.

Odd answer for strange question:


o Reformulate the question in a more logical way, e.g.:
Q1.
Health education is not very effective. Why we do health education?
A1.
You mean obstacles to health education
Q2.
Family doctor is not a social worker. Why you keep asking about
social history?
A2.
You mean the role of family doctor towards social problems.
o Always and always remember there is no (yes or no) answer
o Think of probabilities, advantages & disadvantages.

No Answer at all:
o CME might be an acceptable solution
o Referral might be the perfect answer, e.g.:
Q1. What is a cellular DPT vaccine?
Q2. You suspect pheochromocytoma. What are you going to do?

Disorganized Answer:
o Think before you talk
o Have enough training and develop your own system of thinking

The Joint Program of Family and Community Medicine - Jeddah

153

If a difficult problem is presented:


o Acknowledge the difficulty of the situation and give reasons for it
o Express your feelings about the situation, e.g. This is a difficult situation because
I want to maintain a good relationship with the patient and keep his confidentiality,
but on the other hand his wife need to be protected

Self-Training For Oral Exam


Answer each of the following questions by writing a short notes, and then rehearsing
presenting your answer to a colleague or even to yourself
1.

What is your plan after graduation?

2.

How are you going to maintain your CME?

3. Tell me a bout your research (The research question, the objectives, your
methodology, your final recommendations and how can your result help in improving
health care?)
4. What might you do if you are assigned as a general directorate of PHC in the
Kingdom?
5.

What do you understand by EBM?

6. Tell me about an important article you have read during the last year & how it
affected your practice?
7. (Audit) what is it? What are the advantages & disadvantages of Audit? How you do
it?
8. (Practice Formulary) What do you understand by this term? How do you make your
own formulary? What are the advantages & disadvantages of the formulary?
9.

How do you deal with job stresses

10. A drug Rep. offers you an invitation to conference in Italy, how do you respond?
11. How would you deal with a patient requesting a sick note although you feel he is
healthy?
12. A 28 year old woman presents with a vaginal discharge which was proven to be
chlamydia infection. She asks if she caught it from her husband. What is your
reply?
13. A businessman returns from a trip abroad and present with a penile discharge.
He refuses referral to STD clinic and demand that you do not tell his wife. What are
the implications?
14. A 2-year-old child is a frequent attender to your PHC center with his mother
complaining of minor symptoms, what is your approach to frequent attenders?

The Joint Program of Family and Community Medicine - Jeddah

154

15. You suspect sexual abuse in a 7-year-old girl. What is your management plan?
16. (We should treat all menopausal pt. with HRT). Discuss.
17. A 70-year-old woman has noticed some vaginal spotting. Examination reveals an
atrophic epithelium. How do you treat her?

18. 35-year-old healthy lady presents to you saying: (Can I have some Betnovate
ointment for that rash on my face? How do you respond?
19. Can dietary restriction be helpful in the management of skin disease?
20. There is debate as to whether good control reduces the long-term complications
of DM. What is your opinion?
21. What problems might face a new diabetic? Consider the case of a15 year old and
70 year old.
22. Despite advances in treatment of asthma, the mortality rate from asthma has not
dramatically fallen. What reasons may account for these facts?
23. What are the likely explanations of a sudden dramatic increase in consultation
rate in your practice? How would you determine the cause?
24. How may bad records result in bad medical care?
25. Discuss the advantages & disadvantages of the practice owning its own ECG
machine?
26. How could you modify your medical records to improve preventive care?
27. What do you know about the current public health problem in KSA or/and current
public health problem in the world?
(E.g. Rift Valley Fever / Mad Cow Disease..)

The Joint Program of Family and Community Medicine - Jeddah

155

Risk Assessment Exercise


Q1

List the possible serious complications and/or the possible serious differential
diagnoses for the following problems.

Q2

List the most specific symptoms or signs that would make you suspect these serious
diagnoses or complications
1. A 45-year-old patient with dyspepsia
2. A 50-year-old patient with severe headache
3. A 48-year-old patient with symptoms suggestive of irritable bowel syndrome
for 2 months duration
4. A 25-year-old patient complaining of weight loss
5. A 65-year-old patient with chronic cough
6. A 6-year-old boy with fever for three weeks
7. A 45-year-old illiterate male from badia (rural area) presented with fever for
one month.
8. A 5-day-old infant with jaundice
9. A 9-year-old boy complaining of short stature
10. A 10-month old baby unable to sit independently
11. A 3-year-old child with gastro enteritis
12. A 35-year-old lady with a breast mass
13. A 44-year-old lady with thyroid enlargement
14. A 58-year-old lady with vaginal bleeding
15. A 60-year-old gentle man with prostatic hypertrophy
16. You discover a systolic murmur in a healthy pregnant lady
17. You discover a systolic murmur in a child
18. A 60-year-old lady severely depressed
19. A 25-year old male with first degree hypertension

The Joint Program of Family and Community Medicine - Jeddah

o
o
o

156

Important Topics in Oral Exam


Vital statistics,
Patient scenario testing your index of
Sensitivity & Specificity,
suspicion - e.g.
Survey and Surveillance,
o Depression and risk of suicide,
P. Value, CI, NNT
o Child abuse,
Quarantine,
o Drug abuse,
Incidence and prevalence,
o Dyspepsia in a 50 years old patient
Immunization schedule and specific
Challenging questions, e.g.
immunization,
o Rationale of health education, Is it
Investigation of an epidemic,
effective? Why to do it?
Alma Alta Declaration
Barriers to compliance with advice or
Rationale of Well Baby Clinic,
medication,
Rationale of routine antenatal visits,
Models of consultation,
Primary Care Tasks and Principles,
Patient-Doctor Relationship,
Screening principles / Periodic health
Counseling,
examination,
Reassurance,
Family Life Cycle & Family Genogram, Compliance,
Differences between Family Medicine
Referral,
& Other specialties,
Dealing with difficult patients,
Endemic diseases in KSA e.g.
Breaking bad news,
Brucillosis, Bilharziasis, TB, Malaria.
Advice and Instructions,
Management of chronic diseases,
Illness behavior &
Diagnosis of asthma in children,
Health behavior.
Diagnosis of hypertension / white coat
Audit,
hypertension,
Team work, and Relationship with
Elderly hypertensive & Role of anticolleagues
hypertension drugs,
Plan of action,
DM: e.g. decision to start insulin in
Management cycle,
NIDDM,
Organizing your practice,
Dealing with emergency,
Time management
Use of referral system and discussion
Medical records,
with specialist,
Registers.
Management of common diseases,
CME.
URTI and Role of antibiotics,
How to read an article?
Otitis media & Glue ear,
Burn out.
Menopause and Role of HRT,
Audit
PU and eradication of H. pylori,
Research.
Rationale of empirical treatment
Ethics:
(therapeutic trial) e.g.
o Confidentiality
o Dyspepsia,
o Patient autonomy
o UTI,
o Doctor obligations
o Night cough in children.
o Doctor loyalty
Rationale of some preventive care
protocol - e.g.
Cholesterol screening,
Pap smear,
Breast self-examination & Breast
mammography.

The Joint Program of Family and Community Medicine - Jeddah

13.

Community Medicine Review


A.H. Hassan

Objectives of this review


To perform well in the OSCE examination,
To satisfy the examiner in the final oral examination,
To master efficiently the topics covered in the course,
To master other areas and topics using similar basic principles of thinking and
organization, (apply the same model in other areas),
o To improve the basic knowledge regarding the topics discussed here.
o
o
o
o

Important topics
Primary health care.
Management.
o management cycle.
o audit cycle.
o planning health care activities.
o planning health education program.
Epidemiology.
Others:
o Screening.
o Periodic health assessment.
o Evidence based medicine.
o Immunization (vaccinology).
o Polio. Eradication.
o Tetanus neonatorum elimination.
o New epidemics.
o You the family doctor.
o Up-date.

157

158

The Joint Program of Family and Community Medicine - Jeddah

Management Cycle

Planning
Define objectives
(General & specific)
Prioritization of Objectives

Organization
Who to do what
Resuorce needed:
Personnel
Equipments
Time
Budget

Monitoring
(Audit):
Structure
Process
Outcome

Implementation
Effective Communications
Effective delegation
Teamwork

The Joint Program of Family and Community Medicine - Jeddah

Planning health education program


Preparation
1) Identify the community health problems.
2) Identify the at risk group (target population).
3) Identify resources.
4) Identify possible barriers.
5) Prioritize health problems.
Basic program planning questions
1) What are the dimensions of the health problem?
Condition specific?
People specific?
2) What information, skills, and behaviors must be acquired (learning objectives)?
3) What services needed to enable people to change behaviors?
4) What resources are needed?
5) What changes are expected?
6) Which behavior changes can and should be measured?
7) What support services are needed (training, forms , administrative,..etc.)?
Organization
1) Identify the message (learning experience)
2) Identify the method of communication
3) Schedule an operational plan
4) Identify monitoring and impact measurement tools

Epidemiology

Control of Communicable diseases,


Control of Non-communicable diseases,
Occupational hazards.
Investigation of an epidemic.
Disaster plan.
Field survey.
Surveillance.

159

The Joint Program of Family and Community Medicine - Jeddah

Epidemiology
Control of Communicable Disease
1) Identification of cases.
2) Causative Agent.
3) Occurrence.
4) Reservoir.
5) Mode of transmission.
6) Incubation period.
7) Period of Communicability.
8) Susceptibility and resistance.
9) Control:
Preventive measures:
o Health education
o Prophylaxis.
Control of patient, contact, and environment.
o Notification, isolation, disinfection, quarantine, investigations, treatment.
Epidemic measures.
Disaster implications.
International measures

Control of communicable diseases


A) agent :o Eliminate the agent.
o Prevent multiplication.
o Eliminate reservoir.
B) reservoir :o Human.
o Animal.
o Soil.
C) portal of exit : D) mode of transmission :o Direct
o Common vehicle
o Air born
o Arthropods
E) susceptible host :o Health habits
o Immunity
o Chemoprophylaxis

160

The Joint Program of Family and Community Medicine - Jeddah

Control of communicable and non-communicable diseases


A) primary prevention:o Health promotion
o Specific protection:- (ig., vaccines, chemoprophylaxis)
B) secondary prevention:o Early detection (screening)
o Prompt treatment.
C) tertiary prevention:o Limit disability.
o Rehabilitation.

Control of occupational hazards


o
o
o
o
o
o
o
o
o
o
o
o
o

Identification of occupational hazards.


Substitution.
Enclosure.
Removal of the source.
Segregation.
Ventilation.
Exhaust system.
Sound protective equipment.
Good house keeping.
Environmental monitoring.
Worker education.
Legislation.
Screening.

Physician role in prevention of occupational hazards

Visiting the work place.


Establishing surveillance (reporting) system.
Monitoring new materials.
Educational programs.
Establishing team:
o Hygienist
o Nurse
o Safety officer.

161

The Joint Program of Family and Community Medicine - Jeddah

Investigation of an epidemic
1) verify the diagnosis.
2) establish the existence of an epidemic.
3) characterize the distribution of cases.
4) develop the hypothesis.
5) test the hypothesis.
6) formulate a conclusion.
7) institute control measures.

Disaster management
Preparation of relief plan:
1) rescue of victims.
2) provision of emergency medical care.
3) elimination of physical dangers.
4) evacuation of population.
5) provision of preventive and routine medical care.
6) provision of water, food, clothing, and shelter.
7) disposal of human and solid wastes, and human bodies.
8) control of vector-borne diseases.

Field survey
Definition:
Detailed field study in a period of time to determine magnitude and
epidemiological factors
To help in planning and evaluation of preventive and control program
A) preparation: personnel, finance, equipment, forms,etc.
B) steps:
1) define area and population.
2) examination and/or investigation.
3) existing program.
4) analysis of data.
5) report and recommendations.

162

The Joint Program of Family and Community Medicine - Jeddah

163

Surveillance
Definition:
Regular collection and summarization and analysis of data on newly diagnosed
cases of any infectious disease for the purpose of identifying high risk groups in
the population, understanding the mode(s) of transmission, and reducing or
eliminating its transmission.
Types:
Active: collection of data (usually on a specific disease) for a limited period of
time by regular outsearch on the part of health department personnel.
Passive (reporting): data generated without solicitation, intervention, or contact by
the health agency carrying out the surveillance.

Screening
Definition:
The presumptive identification of unrecognized disease or defect or individuals at high
risk by the application of tests (question, physical examination, investigation, or other
procedures), which can be applied rapidly to sort out apparently well people who
probably have a disease from those who probably do not.
Prerequisites:
Problem:
o Important.
o Treatable.
o Available treatment.
o Understandable natural history.
o Asymptomatic stage.
Test:
o Suitable.
o Acceptable.
o Reasonable cost.
o Reliable.
o Valid.

The Joint Program of Family and Community Medicine - Jeddah

164

Periodic health assessment


Strength of recommendations:
A: good evidence to support the recommendation to include the condition in
a periodic heath examination
B: fair evidence to support the recommendation to include the condition in
a periodic heath examination
C: insufficient evidence to recommend for or against the inclusion of the
condition in a periodic health examination, but recommendations may be made
on other rounds.
D: fair evidence to support the recommendation to exclude the condition from a
periodic health examination.
E: good evidence to support the recommendation to exclude the condition from a
periodic health examination.

Evidence Based Medicine


Quality of evidence:
I: at least one properly randomized controlled trial.
II-1: well-designed controlled trials without randomization.
II-2: well-designed cohort or case-control analytic studies.
II-3: multiple time series with or without the intervention; or
dramatic results in uncontrolled experiments.
III: opinions of respected authorities.

The Joint Program of Family and Community Medicine - Jeddah

165

General Rules of Vaccinology


Rule Number 1:
o The more similar a vaccine is to the natural disease, the better the immune response
to the vaccine.
Rule Number 2:
o Live attenuated vaccines are usually affected by circulating antibody to the antigen.
o Inactivated vaccines are generally not affected by circulating antibody to the antigen.
Rule Number 3:
o There are no contraindications to simultaneous administration of any vaccines.
o Except cholera and yellow fever.
Rule Number 4:
o Increasing the interval between doses of a multi-dose vaccine does not diminish the
effectiveness of the vaccine.
o Decreasing the interval between doses of a multi-dose vaccine may interfere with
antibody response and protection.
Rule Number 5:
o Live attenuated vaccines generally produce long lasting immunity with a single dose.
Inactivated vaccines require multiple doses and often require periodic boosting to
maintain immunity, except OPV and measles
Rule Number 6:
o Adverse events following live attenuated vaccines are similar to a mild form of the
natural illness.
o Adverse events following inactivated vaccines are mostly local with or without
fever, except DPT
Rule Number 7:
There is only one universal absolute contraindication to vaccination:
o Severe allergy to a vaccine component, or
o Following a prior dose of a vaccine

The Joint Program of Family and Community Medicine - Jeddah

166

Missed opportunity
A child in need of immunization seeks health care but receives either no immunizations
or does not receive all the needed immunizations.
o Child seeks immunization but; not offered the needed vaccines.
o Child seeks other health care services but; not his immunization status not
assessed.
o Child is enrolled in assistance program for vaccination but; missed.
The most important, immediately implemented actions for reducing missed
opportunities for vaccination are:
o Assess vaccination status of all < 5 at all visits.
o Elimination of invalid contraindications to vaccination.
o The use of simultaneous administration role whenever needed.
Invalid contraindications to vaccination:
1) minor illness:
Low-grade fever, upper respiratory tract infection, common cold, otitis media and
mild diarrhea.
2) antibiotic therapy
3) disease exposure or convalescence:o If the child is not severely ill
o It will not affect the response
o It will not increase the likelihood of an adverse event.
4) pregnancy in household:o The risk is similar to non-pregnant household
o Measles & mumps produce a non-communicable infection.
5) breast feeding
6) prematurity
7) Allergies:o Nonspecific allergies
o Non-severe egg allergies
o Allergies to antibiotics not in vaccine
o Allergies to duck antigens
o Family history of allergy
8) family history of adverse events to a vaccine.
9) tuberculin testing:
o All vaccines can be given with PPD or any time after testing and .except
MMR wait for 4-6 weeks after MMR before PPD testing.

The Joint Program of Family and Community Medicine - Jeddah

167

Up date Knowledge in Vaccinology


New vaccines: 7 valent pneumococcal vaccine for infants.
Renewed old vaccines (DTaP, Hep. A 2 doses only).
Vaccine combinations
o Quadri-valent.
o Penta-valent.
o Hexa-valent.
Debate
o Decision making: To include or not (Hib, Hep A, VZ, 7 valent pneumococcal)
o OPV Vs IPV.
o MMR and Inflammatory Bowel Disease.
Poliomyelitis Eradication Vs Tetaneous neonatorum eminationel

New epidemics
RTA, CVD, HIV, Substance abuse

The Joint Program of Family and Community Medicine - Jeddah

14.

Basic Statestical Principles for Physicians


A.H. Hassan

Objectives of the Review


1) To perform well in the OSCE examination.
2) To satisfy the examiners in the ORAL examination.
3) To master efficiently topics covered.
4) To deal efficiently with similar problems using the same models.

Important Definitions
o
o
o
o
o
o
o
o
o
o
o
o
o
o

Sensitivity.
Specificity.
A positive predictive value.
A negative predictive value.
Likelihood ratio of a positive test.
Type 1 error.
Type 2 error.
P value.
Incidence.
Prevalence. (Point prevalence and Period prevalence)
Odds ratio.
Relative risk.
Absolute risk.
Attributable risk.

Sensitivity
o Proportion of diseased persons with positive test result.
o Measures the true positive.
o Positivity in disease.
o Ability to include person with a disease.

168

The Joint Program of Family and Community Medicine - Jeddah

Specivity
o Proportion of healthy persons with negative test result.
o Measures the true negative.
o Negativity in health.
o Ability to exclude person without a disease

Highly Sensitive
o
o
o
o
o
o
o
o
o

Best for screening.


Believe negative test.
Rarely miss a patient.
Best for common diseases.
Best for serious diseases.
Used to rule out a disease.
Negative test excludes the disease.
Many false positive.
Few false negative.

o
o
o
o
o
o
o
o
o

Best for conformation.


Believe positive test.
Rarely include a healthy.
Best for rare diseases.
Best for stigma diseases.
Used to rule in a disease.
Positive test conforms the disease.
Few false positive.
Many false negative.

Highly Specific

169

The Joint Program of Family and Community Medicine - Jeddah

170

Definitions
Positive predictive value: probability of a disease in a person with a positive test.
Negative predictive value: probability of absence of disease in a person with a
negative test.
Likelihood ratio: the ability of a test to discriminate between diseased and nondiseased persons.
i.e. Probability of a positive result in a diseased person / Probability of a positive
result in a healthy person.
Type 1 error: true null hypothesis rejected.
Type 2 error: false null hypothesis accepted.
P value: the probability of the result of the study occurring by chance, if the null
hypothesis were true.
Incidence: the number of new cases of a disease in a population over a period of
time.
Prevalence: the proportion of individuals in a population who have the disease at a
given time.
o Point prevalence: prevalence at one point of time.
o Period prevalence: prevalence at a point time, plus new cases in a time period.
Odds ratio: the odds that a disease will occur among exposed compared to
occurrence among nonexposed. (the probability of contracting a disease divided by
the probability of not contracting the disease).
Relative risk: the ratio of the incidence among exposed to the incidence among
nonexposed.
Absolute risk: the probability of occurrence due to exposure.
Attributable risk: the excess risk of a disease that can be ascribed to the exposure to
a risk factor.
Population attributable risk: the excess risk of a disease in a population that can be
solely attributed to the exposure to the risk factor.

The Joint Program of Family and Community Medicine - Jeddah

171

Two by Two Tables


o In a trial to calculate the power of urine testing for glucose to diagnose DM, a family
physician included 38 person in a pilot study. Of them 18 were known diabetics.
Eighteen persons tested positive of them 11 were diabetics.
o Construct two by two table for the urine test.
o Calculate the following: Sensitivity and Specificity.
Positive and negative predictive values.
The likelihood ratios of positive and negative test results.
o Of the 231 patients admitted to the CCU with professional diagnosis of acute MI, and
with positive CK test; 16 patients were found not to have MI.
o Whereas, of the 129 patients with negative CK test admitted to the same unit, 15
patients were found to have acute MI.
o Construct the two by two table for the CK test.
o Calculate the following: Sensitivity. Spacivity, positive predictive value, negative Predictive value and
Likelihood ratio of the test.

Example of Two by Two Table


o The prevalence of tuberculosis in a 100,000 population to be screened is
approximately 4%.
o The tuberculin skin test is used for screening. The sensitivity and specificity
associated with this screening are 40.8% and 99.2% respectively.
o Construct a (22 table).
o A person in this population was found to have a positive result, What is the
probability that he has TB?
o Calculate the percentage of false positive and negative test results.

The Joint Program of Family and Community Medicine - Jeddah

172

Vital Rates
Definition:
The expression of the probability of occurrence of a particular event in a defined
population during a specified period of time. Mathematically, a rate is expressed as
X
/Y X K.
Where
X= the number of events or Cases,
Y= the total population at risk, and
K is a round number, or base, chosen to express the rate as a number greater
than one.

Natality Rates
Crude Birth Rate:
number of live births reported during a given time interval
---------------------------------------------------------- x 1000
Estimated midinterval population
The crude birth rate is expressed per 1000 population. (Note that the total population
is used in the denominator even though many individuals are not at risk of becoming
pregnant).
Fertility Rate:
Number of live births reported during a given time interval
--------------------------------------------------- x 1000
Estimated number of women age 15-44 years at midinterval
Fertility rate is expressed per 1000 population.

Morbidity Rates
Incidence Rate:
Number of new cases of a specific disease during a given time interval
--------------------------------------------------------------- x 100
Estimated midinterval population at risk

A high incidence rate means a high occurrence of disease; a low incidence rate
means a low occurrence of disease.
1) Because incidence rate is a measure of the rate at which healthy people develop
disease during a specific time period, it is a statement of probability.
2) Since incidence rates are affected by any factor that affects the development of a
disease, they can be used to detect etiologic factors.

The Joint Program of Family and Community Medicine - Jeddah

173

Morbidity Rates
Prevalence Rate:
Number of current cases [old (i.e. people who contracted disease before time period
began and who still have the disease) and new] of a specified disease during a
specified time period
----------------------------------------------------- x 100
Estimated midinterval population at risk
1) Point prevalence refers to a specific point in time
2) Period prevalence refers to a given time interval

Morbidity Rates
Prevalence:
o Since prevalence rate contains all known cases in the numerator, it is used primarily
to measure the amount of illness in a community and, thus, can be used to determine
the health care needs of that community.
o Prevalence rates are influenced by both the incidence of disease and by the duration
of illness.

Morbidity Rates
Attack rate:
Number of new cases of a specific disease during a specific time interval
---------------------------------------------------------------------------- x 100
Total population at risk during the same time interval
Attack rate normally is expressed as a percentage. It is an incidence rate that is
calculated in an epidemic situation using a particular population observed for a
limited period of time.

The Joint Program of Family and Community Medicine - Jeddah

174

Morbidity Rates
Secondary Attack Rate:
(number of new cases in a group) (index case or cases during a specified time period)
---------------------------------------------------------------------------- x 100
(number of susceptible individuals in the group) (index case or cases)
Note that the index case or cases that introduced the disease into the group are
removed from both the numerator and denominator. The secondary attack rate
measures spread within an epidemiologic unit. Coindex cases are two or more cases
that, based on the incubation period of the disease, were infected by someone
outside the group.
Crude Death Rate:
Total number of deaths reported during a given interval
------------------------------------------------ x 1000
Estimated midinterval population
Crude death rate is expressed per 1000 population. (Note that the total population is
used even though the risk of death is different for different age-groups.)
Cause-Specific Death Rate:
number of deaths assigned to a specified cause during a given time interval
---------------------------------------------------- x 100,000
Estimated midinterval population
Cause-specific death rate is expressed per 100,000 population.
Maternal Mortality Rate:
Number of deaths related to pregnancy during a given interval
--------------------------------------------------------------------- x 1000
Number of live births reported during the same time interval
Although the true population at risk should be the number of pregnant women, this is
an impossible figure to determine. The number of live births is chosen because it
reflects the number of pregnant women; thus, this is a pseudorate, or index.
Case-Fatality Rate:
Number of deaths assigned to a specific disease
------------------------------------------- x 100
Number of cases of the disease
Case-fatality rate is frequently expressed as a percentage. It predicts the risk of dying
if the disease is contracted. Tetanus has a case-fatality rate of 30-90%, depending on
the age of the host, the length of the incubation period, and the type and length of
therapy.

The Joint Program of Family and Community Medicine - Jeddah

175

Mortality Rate
Proportionate Mortality Ratio (PMR):
Number of deaths from a given cause in a specified time period
--------------------------------------- x 100
Total deaths in the same time period
PMR usually is expressed as a percentage.
o The PMR is not a rate and does not measure the probability of dying from a particular
cause.
o The PMR is primarily used to determine the relative importance of a specific cause of
death in relation to all causes of death within a population.

Exercise:
In a small town the estimated population on 6/98 was 500,000 persons ; of them
300,000 were Nationals. At the same time, it was estimated that the total live birth for
Nationals is 9000 & for Non-Nationals 6000. Number of deaths for all ages from all
causes was 2700 for Nationals & 2400 for Non-Nationals. The infant death numbers
were 180 & 360 for Nationals & Non-Nationals; while deaths due to IHD were 1200
& 600 respectively.
Define Infant mortality rate (IMR).
Calculate IMR for Nationals.
Define Crude Death Rate (CDR).
Calculate CDR for Nationals.
Define Cause Specific Proportional Mortality Rate (CSPMR) for Nationals & NonNationals due to IHD.

Types Of Epidemiological Studies


A) Your own study.
B) Types of studies.
C) The power of the study.

Your Own Study

o
o

Objectives (Aim).
Design.
Sampling:
Technique
size.
Conclusion.
Recommendations.

The Joint Program of Family and Community Medicine - Jeddah

Types Of Studies

o
o

Observational
Descriptive
Analytic
Experimental

Observational Studies
Descriptive studies:
o Ecological (correlational).
o Cross sectional (prevalence).
o Longitudinal.
o Case seires or report.

Descriptive study
o
o
o
o

Is designed to give a clear picture of a particular situation


Describes diseases in relation to person, place, and time.
It constitutes an important first step to determine risk factors
It uses the available sources of information

Descriptive study
Descriptive study can be:
Qualitative research concerned with opinions, perceptions and attitudes towards a
topic, for example family panning.
Quantitative research aiming at quantifying the distribution of certain variables
among the study population e.g. prevalence of certain disease.

Observational Studies
Aanalytic studies:
o Case control (case reference).
o Cohort (interventional):
- Prospective.
- Retrospective.
Case-control studies
o Begin after individuals developed or failed to develop the disease.
o Incidence cases should be selected.
o Useful in early stages of development of knowledge about a particular disease.

176

The Joint Program of Family and Community Medicine - Jeddah

Case-control study design


The design is guided by
o The type of the disease to be investigated
o The appropriateness of controls
o The basic assumptions underling the analysis such as:
- Cases are representatives of persons who are being investigated for the disease.
- The control represent the entire non-diseased population.

Advantages and disadvantages


of case-control study
Advantages
o Cheep.
o Quick.
o Small sample size.
o Good for rare diseases.
o Can test several risk factors.
o Odds ratio can be calculated.

Advantages and disadvantages


of case-control study
Disadvantages
Tests only one disease.
Determination of casual relationship is difficult (can only estimate the risk of
developing the disease indirectly).
Incidence rate, and relative and absolute risk cannot be calculated.
Bias is common: o Recall bias.
o Selection (of control) bias.
o Bias by knowing the diagnosis.

Selection of controls
o
o
o
o

From the population


Persons working in the same factory or attending the same school
Hospital patients
Close associate of the cases

177

The Joint Program of Family and Community Medicine - Jeddah

178

Cohort study
Individuals are selected on the basis of the presence or the absence of exposure to
suspected risk factors

Definition of cohort
Cohort: is a group of persons who share a common experience within a defined time
period. A birth cohort and survivors of myocardial infarction in one particular year are
examples of cohort

Advantages and disadvantages


of cohort study
Advantages:
Good for assessing rare exposure
Several out-comes can be measured (investigate multiple effects of a single
exposure).
minimizes selection bias.
Incidence rate, and relative and absolute risk can be calculated.
The methods are standardized and the results are trusted (temporal sequence can be
more clearly established).
Efficacy of the drug or procedure in treatment of certain disease.
Disadvantages:
Longer time.
Expensive.
Larger sample size.
Difficult: o Drop-out.
o Changes in diagnostic criteria.
o Changes in other variables.

The Joint Program of Family and Community Medicine - Jeddah

179

Experimental Studies
1) Randomized control trial (clinical trial)
2) Field trial
3) Community trial

Experimental study
Experimental study is the strongest possible type of study to prove causation.
The unique feature of experimental study is the method for assigning individuals to
study and control groups.
Experimental study
Used to evaluate: o Prophylactic agent, such as vaccine.
o Public health procedure, such as screening test.

Strength of Evidence for Causality


1)
2)
3)
4)
5)
6)
7)

Experimental: The strongest


Prospective cohort:
Historical cohort:
Case-control:
Cross-sectional:
Case series:
Case report: The Weakest

The Joint Program of Family and Community Medicine - Jeddah

15.

Some Important Topics for Oral Exam


F. Rayes, A. H Hassan, M. Alatta, N. Dashash & A. Assaggaf

Primary Health Care


A. H. Hassan

Definition of PHC
o It is the first level of contact of individuals with the health system.
o It is the provision of essential health care based on practical,
scientifically sound, and socially accepted methods and technology
made universally accessible to individuals, families, and community;
through their full participation and within a reasonable cost.

Elements of PHC
1. Health education.
2. Proper nutrition.
3. Maternal and child health.
4. Immunization.
5. Safe water supply.
6. Control of endemic diseases, and environmental health.
7. Referral.
8. Treatment of common health problems.
9. Treatment of acute respiratory tract infections.
10. Provision of essential drugs.
11. Mental health.
12. Dental health.
13. Geriatric care.

Principles of PHC
o
o
o
o

Equity of distribution.
Appropriate technology.
Multisectorial approach.
Community participation.

180

The Joint Program of Family and Community Medicine - Jeddah

Instruments of PHC
o
o
o
o
o

Community survey and diagnosis.


Family health records.
Plan of action.
Team approach.
Information system.

Strategies for PHC implementation


o Expansion of services.
o Better relation with community.
o Encourage comprehensive care (curative, preventive, promotive,
rehabilitative).
o Integration preventive and curative services.
o Promotion of health awareness.
o Coordination with secondary care.
o Multisectorial approach.
o At risk approach.

181

The Joint Program of Family and Community Medicine - Jeddah

182

The Team Work


Dr. Fayza Rayes
As the effective multidisciplinary teamwork is an essential ingredient of good patient
care, the importance of the teamwork was recognized in the Alma Alta Declaration in
1978 and was further emphasized in recent statement by World Health Organization.
Aspects of Teamwork:
o Pooling of resources
o Delegation of work
o Specialization of function
o Every team need a leader
Common Problems in Teamwork:
Difficulties in communications
Teamwork takes time;
o Meetings take time
o Infrequent or two frequent meetings
3.
Confidentiality
4.
Common Problems in meetings:
o Interruption
o The setting (the place and time)
o Decision Yes, Action No.
The Members of the Primary Health Care Team:
o The patient is very important member in the team
o Doctors
o Nurses
o Receptionists
o Social workers
o Dietitians
Who is the Leader?
o The successful leader will have the following criteria:
o The most responsible one in the team
o The most respectable one in the team
o The one with the highest income
o The one with the highest qualification
o The most stable one in the center to insuring continuity of care
Approach to Leadership:
o Initiatives: The leader person is usually the most initiative person in the team,
bringing new ideas and encouraging creativity
o Arrange social activities: Usually he /she makes use of every social occasion
in order to maintain good relationship between team members
o Take care of the new members: Usually he /she takes special care of the new
members, he/she will discuss with him his job description and help him to be
oriented to work environment and to be familial with other team members in
the practice

The Joint Program of Family and Community Medicine - Jeddah

183

Cholesterol Screening
F. Rayes
Cholesterol level mmol / L Action

Follow-up

Less than 5.2

None

Re-check in 3 years

5.2 - 6.5

Dietary advice

Re-check in 1 year

Above 6.5

Dietary advice
Drug history ?!
TFTs, LFTs
Or referral if
Creatinine is high or FBS
is high

Re-check in 3 ms

o
o
o
o
o
o
o
o
o
o
o
o
o
o
o

Indications of screening for hyperlipidemia


Family history of CHD
Family history of hyperlipidemia
Premature vascular disease
Stigmata of hyperlipidemia
Hypertension
Diabetes mellitus
Obesity
All adults aged over 20 years (Future).
IHD / Referral for coronary angiography
Angina despite full medical therapy
Unstable angina
Angina with features of heart failure
Acute MI with recurrent pain between day two and discharge
Post-infarction angina during the next three months
Valvular heart disease

The Joint Program of Family and Community Medicine - Jeddah

184

Poliomyelitis
F. Rayes

Poliomyelitis

Epidemiology of poliomyelitis
o Infectious agent:
- Polio virus I, II & III.
o Occurrence: World - wide.
o Reservoir: Man
o Mode of Transmission: Food, water and droplet
o Incubation Period:
- 7 - 14 days commonly
- 3 - 35 days range
o Period of Communicability:3 - 6 weeks.
- Case are most infectious during the first few days before and
after onset of symptom.
o Susceptibility & Resistance :
- Susceptibility is common but paralysis is rare.
o Factors increase risk of paralysis:
- IM injection
- Surgery
- Muscular exhaustion
- Pregnancy

Mass immunization campaign.


o Size of the problem:
- 250,000 cases continue to occur each year.
- Crowding, poor sanitation, inadequate hygiene other enteric
pathogens are believed to facilitate transmission.
- In USA 1955 >450 million doses were administer to children
and adults during 5 years
- Incidence declined from 18 cases / 100,000 total population to
2 cases / 100,000.
- 1961 the incidence decrease 1 case / 10 million populate by
1970.
- By 1979 NO cases Eradication in USA.

o Vaccine development and use:


- Salk (Inactivated Poliovirus Vaccine) IPV 1954
- Sabin (Live, Attenicated Vaccine ) OPV 1960

The Joint Program of Family and Community Medicine - Jeddah

185

Global Eradication :
WHO called for Global Eradication of poliomyelitis by year 2000.
Achievement and maintenance of high immunization level.
Vaccine quality control and availability.
Training and supervision of in-country personnel.
Acute Flacid Paralysis ( AFP ) surveillence.
National Immunization Days ( NID ).
Research and development to increase the effect of vaccine.
No more than 14 cases / year, average 8 cases/ year since 1980.
One case / 2.6 million doses of OPV, the relative frequency of paralysis
- with first dose
( 1/520,000 doses)
- with subsequent doses
( 1 / 12.3 million dose ).
Note: Preparations are now available in USA which incorporate killed poliomyelitis
with DPT.

o
o
o
o
o
o
o
o
o

o
o
o
o
o
o
o
o
o

Immediate Treatment for Suspected Meningococcal Infections


Adult and children older that 10 years --> 1200 mg Benzylpenicillin. IM
Children aged 1-9 years ---> 600 mg Benzylpenicillin. IM
Infants aged less than 1 year ---> 300 mg Benzylpenicillin. IM
Pt. with Rash / Warning Presentation
Associated symptoms suggestive of serious illness.
Purpuric or petechial rash
Generalized pustular rash
Infection in dangerous area eg. eyes, dangerous area of the face.
Very toxic patient

The Joint Program of Family and Community Medicine - Jeddah

186

Tuberculosis
F. Rayes & N Dashash

TB in Developed & Developing Countries in comparison to KSA:

Prevalence of infection in the age


group 0-14 y.
Case rate / 100,000
Mortality rate / 100,000
Percent of TB deaths to total deaths

Developed
2-3%

Developing
60 - 80 %

KSA
7%

12 - 20
1-2
0.1 %

250-500
60 - 100
3 - 10 %

30 - 40
-----

TB / Approach to Prevention
o Countries of high prevalence: Vaccination at birth (60-80% efficacy).
o Country of low prevalence: Case finding and INH prophylaxis (80% efficacy).

o
o
o
o
o

o
o
o
o
o
o
o

TB / Facts
BCG efficacy 60-80%.
BCG cause +ve skin test for 6-7 years after immunization.
False -ve PPD in 20% of active TB.
False -ve PPD in 50% of HIV.
Chest x-ray is normal in 50% of extrapulmonary TB.
TB / Diagnosis
Clinically:
High index of suspicion
Screening Tests
PPD skin test: if positive test = infection has occurred.
X-ray chest: it is less sensitive than skin test.
Confirmatory test
Sputum

T.B. skin test


o 1 in 3 persons affected in the world.
o 20-30 % increase in cases in 1991. In USA 13% increased rate in 1992.
o 8 million new cases annually, 3 million deaths/year from T.B.
New CDC Recommendations

5 mm indurations is considered positive in:


o HIV or high risk for HIV
o Close recent contact TB patient
o CXR consistent with old TB

o
o
o
o
o
o

10 mm is considered positive in:


Prevalent areas
IV drug abusers
Low socio-economic class and homeless
Patients in long term care institutes
Other medical conditions: gastrectomy , chronic renal failure, DM, steroid
Health care employees, day care, schools

The Joint Program of Family and Community Medicine - Jeddah

Child with Short Stature


F. Rayes

o
o
o
o
o
o

o
o
o
o
o
o

Criteria of familial short stature:


Short parents
Normal birth history and gestational weight
Short stature but with normal growth velocity
Bone age equal to choronologic age
Normal age for the onset of puberty
Absence of organic or psychogenic disease
Criteria for Constitutional Short Stature
50% of cases have history of delayed growth and development is either parent.
Normal birth history and growth for the first months of life.
Absence of organic or psychogenic disease
Bone age less than choronoligic age
Delayed onset of puberty.
Final height is appropriate for genetic potential.

Initial investigation of A child with short stature


X-ray to determine bone age:
o AP & lateral -knee & hand (< 2 ys age).
o AP & lateral - left hand (> 2 ys age).
Bio-chemistry:
o CBC
o Urinalysis
o ESR
o U&E
o LFTs
o Iron concentration
o Calcium, Phosphorus & Alkaline phosphate

187

The Joint Program of Family and Community Medicine - Jeddah

188

Appendix
Studying advice
Nisreen Jastaniah
Preparing your self for the exam:
For the written part:
You need to do more reading hours with good use of time.
Try to cover as much as you can of the MCQ books; you are advised to read with
their explanations. While studying keep some notes of things that you think were
difficult for you,
MCQs always need revision in order to remember them later. (Use your long-term
memory).
For MEQs do more practice, try to write at least one MEQ / day 3-4 weeks before
the exam. (Consider the time, review the answer & you may need to re-write them
later if you could not gather most points)
For CRQ you must have good basic knowledge by now. Tray to answer a paper
/week at least 5 papers. (Consider the time).
For the oral:
Your performance most of the time will depend on your knowledge & you real
practice (peer help can be useful).
Try to be exposed to more cases.
Practice consultation skills.
Prepare you self for the data.
Try to do more oral exams before the real one.
Remember always follow a written plan & modify it according to your need.
Best wishes

The Joint Program of Family and Community Medicine - Jeddah

189

Studying advice
Manal Khursheed
In the beginning, I thank God and then the Family & Community medicine program
in Jeddah for my graduation in family medicine specialty.
My advice to our colleges for succeeding their exam:
Dont leave any day without reading any thing related to our requirement
Be updated ,always search for new knowledge in journals, internet ,courses & so
on
MCQs, need a lot of concentration and repetition, read them in quiet place, at least
three months before your final exam, it is better to read them alone not with
company, & if you have a mistake put a sign beside it and review it again before
you finish that MCQ book.
SIMULATED CLINIC, needs practice & practice, don't escape from your clinic.
At least six months before exam, you need to be trained on them, not alone but as
a group (at least two per group) & use an alarm clock to be on time.
MEQ, needs training by writing & also try to use an alarm clock to be on time.
SLIDES, need practice and practice, read from atlases and related books that are
available in the program library
DATA INTERPRETATIONS, need practice in the clinic and also read the books
of data available in the program library.
Listen to the advises of your trainer staff & dont ignore them.
Finally I ask God to help you to succeed your exams.

The Joint Program of Family and Community Medicine - Jeddah

190

Internet Resources
H. Hajjar

Site name

Address

Sites of Interest to Family


Physicians
Internet resources for
family doctors

http://www.uwo.ca/fammed/clfm/sites.html
http://www.wonca.org/resources/journals/wonca_journals.htm

Journals
American Family
Physician
Family practice
management
Royal Collage of General
Practitioners
British Journal of General
Practice
Electronic BMJ
Jama
Royal Australian Collage
of General Practice
The Collage of Family
Physicians of Canada

http://www.aafp.org/afp/index.html
http://www.aafp.org/fpm/index.html
http://www.rcgp.org.uk/
http://www.rcgp.org.uk/rcgp/webmaster/bjgp_sub.asp
http://www.bmj.com/
http://jama.ama-assn.org/
http://www.racgp.org.au/publications/
http://www.cfpc.ca/index.htm

MEDLINE

Pub med

http://www.ncbi.nlm.nih.gov/entrez/query.fcgi?SUBMIT=y

Infotrieve

http://www3.infotrieve.com/medline/infotrieve/

American Academy of
Family Physician
American Medical
Association
American Diabetes
Association
British Hypertension
Society
British Thoracic Society
Center of Disease Control
and Prevention
Canadian Medical
Association
National , Heart, Lung and
Blood Institute
WHO

http://www.aafp.org/
http://www.ama-assn.org/
http://www.diabetes.org/
http://www.hyp.ac.uk/bhs/
http://www.brit-thoracic.org.uk/
http://www.cdc.gov/
http://www.cma.ca/eng-index.htm
http://www.nhlbi.nih.gov/index.htm
http://www.who.int/home-page/
http://www.wonca.org/

The Joint Program of Family and Community Medicine - Jeddah

Site name

191

Address
Evidence Based Medicine

National Guideline
Clearinghouse
Primary Care Clinical
Practice Guidelines

http://www.guideline.gov/body_home_nf.asp?view=home

Evidence-Based Medicine
(Journal)

http://hiru.mcmaster.ca/ebmj/default.htm

The Cochrane Library

http://hiru.mcmaster.ca/cochrane/cochrane/cdsr.htm

http://medicine.ucsf.edu/resources/guidelines/

http://www.mayohealth.org/home
Medscape
The Emergency
Medicine and Primary
Auscultation Assistant

http://www.medscape.com/
http://www.embbs.com/
http://www.wilkes.med.ucla.edu/intro.html
http://www.arabcom.net/chp/scfhs/

194

Index

Acne: Approach to patient with acne


Anemia: Approach to patient with anemia
- Investigations
- Macrocytic anemia
Ankle Examination
Antidepressant prescribing
Anxiety History
Arthritis: Approach to patient with arthritis
- History
- Examination
- Differential diagnosis and investigations
- X-Ray
Asthma: Approach to patient with asthma
- Advice
- Inhaler technique
- Peak Flow Meter
- Patient education
- Asthma traffic light zone
Audit
Backache: Approach to patient with backache
- Back examination
- Counseling patient with backache
Body mass index
Breast Feeding Counseling
Carpal Tunnel Syndrome
Chest Pain: Approach to patient with chest Pain
- Management of acute MI
- ECG findings
Cholesterol Screening
Community Medicine Review
Cough: Approach to patient with cough
- Differential diagnosis of child with cough
Contraception counseling
Contraception: Emergency Pill Counseling
Control of Communicable Disease
Control of Non-Communicable Disease
Data Interpretation: Lab Tests
Data Interpretation: Slides
Data Interpretation: ECG
Data Interpretation: Radiology
Data Interpretation: Self Assessment Exercise

87
84
84, 96
97
47
32
30
19
19
20
21
135
8
9
9
10
10
11
183
23
23
25
59
55
42
81
80
127
184
157
81
82
66
67
160
161
96
111
123
129
136

195

F
G

J
K
M

Diabetes Mellitus: Approach to patient with DM


- Counseling newly diagnosed DM
- Diabetes Investigations
Dermatology (see Data interpretation: slides)
Depression
Diarrhea: Approach to patient with diarrhea
Disastermanagement
Dizziness Vertigo History
- Deferential diagnosis of vertigo
Dyspepsia
- Management of dyspepsia
Ear, Nose & Troat
Epilepsy Counseling
Epidemiology
Evidence Based Medicine
Febrile Convulsion Counseling
Field Survey
Gastroenteritis: Approach to patient with gastroenteritis
Glaucoma
Glasgow Scale
Glucose Tolerance Test
Headache: Approach to patient with headache
- Headache alarming symptoms
- Headache deferential diagnosis
Hepatitis B Chronic Carrier Counseling
- Hepatitis investigation
Hormone Replacement Therapy (see menopause)
Hypertension: Approach to patient with hypertension
- History
- Examination
- Poor Compliance
Impotence history
Infertility Female History
Infestations (see data interpretation: lab tests)
Insomnia Counseling (sleep hygiene)
INTERNET Resources
Investigations of epidemics
Irritable Powel Syndrome: Approach to patient with IBS
Jaundice Syndromes (see data interpretation: lab tests)
Joint Examination
Knee Examination
Macrocytic anemia investigation
Management Cycle
Menopause counseling
Mental Status Examination
MI Counseling (Post MI)

12
13
99
111
31
83
162
34
34
92
93
120
51
160
164
53
162
57
49
190, 192
99
85
86
86
62
100
61
15
15
16
18
36
37
109
64
196
162
94
100
42
46
97
158
61
48
54

196

N
O

R
S

U
V

W
X

Nasal Bleeding: History


Obesity Counseling
- Body Mass Index
Ophthalmology (see data interpretation: slides)
Optic Nerve Examination
Oral Exam
Otitis media: Approach to patient with OM
Palpitation History
Planning Health Education Program
Periodic Health Examination
Pruritis History
Red Eye Examination
Rheumatoid Arthritis
Saudi Board Examination
Screening: Cholesterol
Screening: Colorectal Cancer
Screening: DM
Shoulder Examination
Short Stature
Simulated Clinic Exam
Statistics
Surgery
Surveillance
Smoking Counseling
Sore Throat: Approach to patient with sore throat
Studying Advice
STD History
TB
Time Management
Teamwork
Toxpolasma management
Tremor History
Thyroid Function Tests
Traveler Diarrhea
Tetenus Prophylaxis
Urinary Tract Infection: Approach to patient with UTI
Urine Analysis and MSU
Vaccination
- Missed opportunity
Vaccinology General Roles
Vaginal Discharge History
- Deferential diagnosis and treatment
Vertigo (see dizziness)
Weaning Counseling
Wrist Examination (Carpal Tunnel Syndrome)
X-Ray (see data interpretation: radiology)

41
58
59
121
50
148
91
27
159
164
29
49
21
1
184
95
99
44
188
70
168
122
163
60
89
194,195
40
187
197
182
103
28
104
68
193
88
105
166
166
165
38
39
34
56
42
129

197

Você também pode gostar